PAeasy Musculoskeletal

Réussis tes devoirs et examens dès maintenant avec Quizwiz!

A 23-year-old patient who has recently been on a ski trip presents with pain to the right hand after sustaining a fall. It is difficult to move, and there is pain on flexion of the digit. Based on this history what ligament would the patient most likely have injured? A 1st MCP joint ulnar collateral ligament B 2nd MCP joint ulnar collateral ligament C 3rd MCP joint ulnar collateral ligament D 4th MCP joint ulnar collateral ligament E 5th MCP joint ulnar collateral ligament

1st MCP joint ulnar collateral ligament The ulnar collateral ligament at the base of the thumb, or 1st MCP joint, is often injured in forced abduction, such as a fall while skiing or during other sporting activities. An injury to this ligament has traditionally been called Gamekeeper's Thumb, but the origin of this term referred to a more chronic injury sustained by English gamekeepers as a result of the way they killed rabbits using their hands. Any of the MCP joint ulnar collateral ligaments could be injured in a fall if the mechanism of injury creates significant forces on the ligaments, but the 1st MCP joint is far more commonly injured than the others mentioned above.

A 30-year-old man presents with acute posterior ankle pain following a 6-foot-fall from a ladder while painting his residence. He states that he landed on his right foot during the fall, and immediately felt a severe, throbbing pain accompanied by a loud "snap" sound. He is unable to ambulate. His physical exam is notable for a palpable bulging defect of the posterior right ankle, swelling, ecchymosis, and severe tenderness upon palpation of the involved area. Right ankle plantar flexion is markedly weak compared with the left side. An urgent MRI of the ankle is performed, and it demonstrates the following image. Question What is the most likely expected clinical manifestation in this patient? Answer Choices 1 Positive McMurray test 2 Positive Homan's sign 3 Pes cavus deformity 4 Positive Thompson test 5 Positive Cotton test

+ thompson test Explanation This patient is presenting with signs and symptoms consistent with an Achilles tendon rupture. A positive Thompson test is diagnostic of complete Achilles tendon rupture. It is performed with the patient prone and the affected knee bent 90 degrees. Squeezing the calf causes plantar flexion of the foot if the Achilles tendon is intact or partially torn but not if there is complete rupture of the tendon. McMurray's test is useful in the evaluation of a knee meniscus injury. A palpable "click" is appreciated while the examiner applies a valgus stress to flexed knee while externally rotating the leg and slowly extending the knee while still in valgus. Pes cavus refers to a descriptive term for a type of foot deformity with an abnormally high longitudinal arch of the foot (caved in foot). Pain in the leg upon dorsiflexion of the foot in the setting of a deep venous thrombosis is referred to as Homan's sign. The side-to-side (Cotton) test is performed with the examining hand on plantar aspect of the foot/ankle, with the clinician's thumb under 1 malleolus and the middle finger under the other malleolus. A medial and lateral-directed force is then applied to the ankle. A few mm of motion indicates a syndesmosis sprain.

What are the joints MC affected by RA?

MCP and PIP joints

Sicca Syndrome

Sicca syndrome: An autoimmune disease, also known as Sjogren syndrome, that classically combines dry eyes, dry mouth, and another disease of connective tissue such as rheumatoid arthritis (most common), lupus, scleroderma or polymyositis.

A 4-year-old girl presents after a fall from a 4-foot slide 30 minutes ago. She fell with her right dominant arm outstretched and her elbow fully extended. There was no loss of consciousness, but there was extreme pain. She immediately grabbed her right forearm and her right elbow. There is an obvious deformity at the elbow. What is the most likely diagnosis? Answer Choices 1 Proximal humerus fracture 2 Distal radius fracture 3 Supracondylar humerus fracture 4 Lateral epicondyle humerus fracture 5 Medial condyle humerus fracture

Supracondylar humerus fracture Explanation Supracondylar fractures of the distal humerus are the most common elbow fracture in children between the ages of 2 and 12. The typical mechanism of injury for this type of fracture is a fall on an outstretched arm, usually from a height. Proximal humerus and distal radius fractures will not show deformity at the elbow unless also associated with injury at the elbow. Fracture of the lateral epicondyle of the humerus accounts for 1/5th of elbow fractures in children. Fracture of the medial condyle of the humerus is also uncommon.

A 45-year-old Caucasian woman presents with a 3-month history of hot flashes. She complains of occasional night sweats, extreme heat sensitivity, and mood swings. Her last menstrual period was 12 months prior to presentation; it followed an 8-month period of oligomenorrhea. She denies weight loss and fevers. She has no significant medical or surgical history. She denies any history of bone fractures. She does not take any medications. The patient is married and has 2 children. She works as a dental assistant. She denies nicotine, alcohol, or any illicit drugs. Her family history is significant for a sister who had pre-menopausal breast cancer at age 43 and a mother with severe osteoporosis, with a resultant Colles' fracture, femoral neck fracture, and multiple vertebral compression fractures. Physical examination shows a slightly overweight woman. Vital signs are unremarkable. Breast exam is normal. Gynecologic exam shows slightly dry and atrophic vaginal mucosa, with otherwise normal findings. Stool sample is negative for occult blood. The remainder of her physical examination is unremarkable. Remarkable laboratory studies include normal liver function tests and normal cholesterol profile. She underwent a dual-energy X-ray absorptiometry (DEXA scan) with the following results: Bone Mass Density (BMD) T Score Z Score Lumbar Spine (L2-L4) 1.036 -1.4 -0.2 Hip (Femoral Neck) 0.898 -.70 +0.4 The T score is an index of the patient's BMD compared to the average young normal female BMD. According to the World Health Organization, low bone mass (osteopenia) is a T score between -1 and -2.5 SD. Osteoporosis is a T score at or below -2.5. The Z score is a measure of how a patient's BMD compares with age-matched females. Question Based on these results and the patient's history, what treatment program would be most appropriate? Answer Choices 1 Advise a weight-bearing aerobic exercise program, calcium carbonate (1500 mg/day) and vitamin D supplementation 2 Advise medroxyprogesterone to relieve hot flashes 3 Combination therapy (estrogen and progestin) 4 Estradiol therapy 5 Alendronate (Fosamax) therapy

The answer is Alendronate (Fosamax), it is a bisphosphonate used for the treatment and prevention of osteoporosis. This medicine is taken on an empty stomach and requires the person to be sitting or standing (without bending) at least 30 minutes to 1 hour after its intake in order to prevent esophagitis. Because bisphosphonates may form insoluble complexes with food or drink, this patient should limit her ingestion to water while taking alendronate. This is the best treatment option for the patient in this question due to her medical and family history and her ability to comply with the manner of taking this medication. This patient is experiencing the effects of reduced estrogen levels because of menopause. The patient's DEXA scan results show a low bone density (osteopenia) in the lumbar spine, but she does not have osteoporosis. She has several risk factors for osteoporosis, including being a Caucasian woman, having a family history of osteoporosis, and undergoing menopause at age 45. Other risk factors for osteoporosis include: Caucasian or Asian race Cigarette smoking High caffeine or alcohol intake High protein diet Lean Body Frame Sedentary Lifestyle Because her sister had pre-menopausal breast cancer, she has a relative contraindication to taking estrogen therapy. Postmenopausal hormone replacement therapy (PHT) has been shown to have benefits in preventing osteoporotic fractures and colorectal cancer1. According to the American Heart Association, women should not take postmenopausal hormone therapy (PHT), formerly called HRT, or selective estrogen receptor modulators (SERM) for the purpose of reducing the risk of coronary heart disease or stroke4. According to the Heart and Estrogen/Progestin Replacement Study (HERS), a large study conducted by Dr. Deborah Grady at the University of California San Francisco, there was no reduction in the risk of heart attack or stroke among women who took the hormones for 4 years. Estrogen raises the high-density lipoprotein (HDL) cholesterol and lowers the LDL (low-density lipoprotein) cholesterol. Progesterone has the opposite effect on cholesterol levels; however, the overall protective effect of estrogen has been linked to its inhibition of vascular constriction and direct action on the arterial walls. Women who use unopposed estrogen therapy were at increased risk of breast cancer after longer-term use2. Another study showed that women using estrogen and testosterone therapies have a significantly higher risk of invasive breast cancer3. Endometrial cancer incidence is significantly elevated when unopposed estrogen is used, as compared with nonusers. There is increased incidence of thromboembolic events, including deep vein thrombosis and pulmonary embolism among estrogen users, with the highest incidence in the 1st year of use. Among the benefits of PHT is the relief of genital atrophy, vaginitis, relief of hot flashes, and dyspareunia. The HRT administration regimen may involve continuous estrogen and progestin (most commonly oral medroxyprogesterone), thus avoiding the withdrawal bleeding, or continuous estrogen with progestin 14 days a month. However, studies have shown that women receiving PMH (postmenopausal hormone) therapy containing estrogen and testosterone therapies have a significantly increased risk of invasive breast cancer. Weight-bearing exercise, calcium supplementation, and vitamin D are all preventive measures for osteoporosis.

A 38-year-old man presents with a 2-day history of a mass and severe pain in his scrotum. Physical examination reveals that his right testicle appears much larger than his left. On palpation, you note a small hole in his inguinal canal, and you are unable to place the contents into the canal. The contents of the hernia appear ischemic. What is the best description of the hernia? Answer Choices 1 Reducible 2 Recurrent 3 Irreducible 4 Strangulated 5 Incarcerated

The clinical picture is suggestive of an inguinal hernia. The hernia has become ischemic, which is better known as a strangulated hernia. In a reducible hernia, the contents would be able to be placed back into the abdominal cavity with simple manipulation. A recurrent hernia is a hernia that has been previously repaired surgically and has now returned. There is no history of a previous hernia in this patient. An irreducible (also known as incarcerated) hernia is when the hernial contents cannot be returned to their normal site with simple manipulation. This type of hernia would cause edema and entrapment due to impaired venous return. An irreducible (also known as incarcerated) hernia is when the hernial contents cannot be returned to their normal site with simple manipulation. This type of hernia would cause edema and entrapment due to impaired venous return.

A 17-year-old football player (punter) was tackled and forcefully slammed onto his right dominant shoulder 2 hours ago during a game. He had immediate pain, but it has subsided; he now experiences the same level of pain when he carries a heavy object. He wants to continue punting. He has full active and passive range of motion, but some pain (4/10) with abduction. There is no obvious deformity, and the skin over the shoulder is intact and not tented. There is no crepitus on palpation while moving the arm. Most of his tenderness seems to be on the superolateral right shoulder. Question What recommendation(s) should be made to this patient? Answer Choices 1 You should continue playing because this injury is only minor 2 You need to go see an orthopedic surgeon immediately 3 You can continue if the pain allows it, but further injury might make it worse 4 You should continue to play only if the team really needs you 5 You can play after I inject your shoulder with cortisone

You can continue if the pain allows it, but further injury might make it worse Patients with a Type 1 Acromioclavicular separation may resume playing as pain allows, but typically that takes 4 weeks. The patient will run a risk of further injury if he continues to play through the pain and convert a Type 1 into a Type 2 or worse. The patient playing because the coach or teammates really need him should not be advised. This injury, although considered minor, can be made worse if the player were to be hit or land on his shoulder again. Never inject cortisone into an AC joint with a ligament injury. This only weakens the ligament further, and it can increase the chance of converting a Type 1 into a Type 2 or worse. Since this is most likely a Type 1 injury, surgery is not necessary.

A 73-year-old man presents with an inability to actively raise his left arm,. This started 1 month ago, prior to which his shoulder pain had improved. He has had shoulder pain for more than 6 months and it has kept him from sleeping on his left side and has often woken him up. There was no specific injury that he can recall, although he felt a pop in the shoulder a month ago while taking out the trash. On a physical exam, what findings would you expect to observe? Answer Choices 1 Swelling of the shoulder girdle muscles 2 Good strength of all shoulder muscles 3 No tenderness 4 A positive (+) drop arm test 5 Paresthesia of the median, radial, and ulnar nerves

a positive drop arm test Explanation This is a torn rotator cuff and drop arm test is positive in this condition. Typically history includes pain for several months, with difficulty sleeping on the affected side and waking from the pain. Active range of motion is limited, but passive range of motion is usually full (unless associated with a frozen shoulder prior). There is usually tenderness over the greater tuberosity. Nerve damage is rare with cuff tears.

A 45 yo obese male comes in to the ER in the middle of the night screaming and holding his left toe. He tell you that he thinks he has an "acute vessel blockage" in his left toe. His history is significant for essential HTN, for which he has been treated with a thiazide (HCTZ) for the past 5 years. He has never had these symptoms before. On examination the patients temperature is 38 C and his BP is 170.110, he has an inflamed, tender, swollen left great toe. There is extensive swelling and erythema of the left foot. No other joints are swollen. What is the most likely dx? a. acute cellulitis b. acute gouty arthritis c. acute RA d. acute septic arthritis

b. acute gouty arthritis Podagra= attack of great toe (Metatarsal phalangeal joint) is seen in >50% of first time gout attacks Biological hallmark= hyperuricemia; it results from overproduction or excretion of Uric acid or a combo of the two. Painful, monoarthritis (may be polyarthritis), and accompanied by fever. Joint is tender, swollen, warm, and tense and shiny Attack subsides spontaneously in days to weeks NSAIDs are the DOC for acute gouty attack for symptomatic relief. Interarticular methylprednisone acetate will provide this patient with significant relief. Only 5% of hyperuricemia patients will develop gout.

In your new practice, you have inherited 3 adolescent patients who had different early childhood malignancies that were treated in different ways according to the type and grade of cancer. They have been in remission now for over 5 years. In reviewing their charts, you consider the need to monitor for late effects of childhood cancer. What secondary malignancy is commonly associated with radiation therapy? Answer Choices 1 Bladder cancer 2 Hodgkins disease 3 Retinoblastoma 4 Bone and soft tissue sarcoma 5 Familial polyposis coli

bone and soft tissue sarcoma Explanation Radiation is associated with the development of cancer within the radiation field, notably, breast cancer, thyroid cancer, skin cancers, brain tumors, and most commonly bone and soft tissue sarcomas. The cumulative risk for secondary malignancy at 20 years of age in pediatric cancer survivors is 3% to 10%, which is 5 to 20 times greater than for the general population. Survivors should be evaluated yearly for the development of secondary malignancies. Hepatoblastoma has been associated with familial polyposis coli; any survivors should be screened for colon cancer. Patients that were treated with cyclophosphamide have a higher risk of developing bladder cancer, with a cumulative dose directly related to increased risk. The occurrence of non-glomerular hematuria identifies this group. It is not known which major cyclophosphamide metabolite drives bladder carcinogenesis. Survivors of Hodgkin's disease are likely to develop secondary malignancies such as acute myeloid leukemia, non-Hodgkin's lymphoma, breast cancer, and other solid tumors. The risk for some of these malignancies increases with time. Children with a genetic form of retinoblastoma, which includes all who have bilateral disease and about 10% who have unilateral disease, have a 50% chance of developing a secondary malignancy.

You are evaluating a 28-year-old man who is brought in by ambulance for a right knee injury sustained during an MVA. The patient is in obvious distress, holding his right knee and screaming. You are told by EMS that the patient was the restrained driver of a car that was rear-ended at high speed. You are unable to obtain any additional history. Examination already shows the development of a right knee effusion. The skin is intact, and there does not appear to be any deformity. Examination of laxity is limited, secondary to the patient's pain. It appears that distal sensation is intact but you cannot palpate a dorsals pedis or posterior tibial pulse. You order an emergent MRI for further evaluation. Question In the meantime, what test would be best to help confirm the patient's diagnosis? Answer Choices 1 Apley grind test 2 Bilateral ankle-brachial indices 3 Lachman's test 4 Valgus stress 5 McMurray test

b/l ABI test This patients history and exam are concerning for a knee dislocation. Anterior knee dislocation is the most common type, and often due to a high velocity injury such as an MVA.1 While rare, dislocations of the knee have significant complications involving both structural damage to the knee (disruption of tendons, ligaments, and cartilage), as well as neurovascular injuries (injury to the popliteal and/or tibial arteries, and damage to the peroneal nerve most commonly).1 The finding of absent, or weak, distal pulses is concerning for an injury to the popliteal artery. While an MRI will be the definitive test, the finding of an ankle-brachial index of <0.8 (1.0-1.4 being normal) is very concerning and warrants immediate involvement of an orthopedic or vascular surgeon.2 The Apley grind test is used to evaluate a meniscus injury. It is done by having the patient lay prone with their knee bent at 90 degrees. The examiner then applies downward pressure by pushing down on the bottom of the patient's foot while rotating the foot. The test is positive if pain is reproduced.3 The Lachman test is performed by having the patient supine and the knee flexed at 30 degrees. One hand firmly holds the distal femur, and the second hand pulls the proximal tibia anteriorly. Excessive anterior movement of the tibia indicates an injury to the anterior cruciate ligament.3 Valgus stress is done with the patient in the supine position. One hand supports the lateral aspect of the knee while the other hand grasps the patient's ankle and applies valgus stress (move the ankle joint to the lateral side). This test is used to assess if there is laxity of the medial collateral ligament.3 The McMurray test is used to evaluate for a meniscus injury. It is performed on a supine patient by bringing the knee up to 90 degrees of flexion. The examiners hand is then placed of the medial aspect of the knee, and the other hand externally rotates the foot and then extends the leg. If a "click" is appreciated, the test is positive for a medial meniscus injury. The opposite is done to assess for a lateral meniscus injury.3

The patient is a 40-year-old woman who is G5 P4. On ultrasound at 24 weeks, the fetus is diagnosed as small for gestational age (SGA). Normal heart, stomach, situs, and cord vessels are identified. The fetus is identified as having a protruding myelomeningocele during the sonographic examination. During the examination, the sonographer noted reduced movement in the lower limbs. This male fetus has associated malformations that are common with this malformation. What is associated with a lumbar myelomeningocele? Answer Choices 1 Clubfoot 2 Clubhand 3 Reduced movement in the limbs 4 Dacryocystocele 5 Amniotic band syndrome

clubfeet Clubfoot is often seen with a lumbar myelomeningocele, possibly due to the lack of nerve stimulation to the muscles. The basic deformity is a subluxation at the talocalcaneonavicular joint. Several factors, including family history and intrauterine crowding, have been blamed as the causative factors. It is associated with meningomyelocele, arthrogryposis multiplex congenita, and tibial hemimelia. Initially conservative therapy is tried; failure paves way for surgical correction. A clubhand could be seen with a higher defect. Spinal defects may or may not lead to reduced movement in the limbs. Amniotic band syndrome is a combination of many disfiguring manifestations, such as congenital distal ring constrictions, intrauterine amputations, and acrosyndactyly, which are the most common manifestations. The basic pathology is the intrauterine entrapment of the fetal parts by the mesodermal fibrous bands of the ruptured amnion.

4 cardinal signs of inflammation

erythema, warmth, tenderness, and swelling

A 63-year-old man with long standing right shoulder pain from impingement syndrome returns to your office saying now he cannot lift his arm after pulling the starter cord on his lawnmower a month ago. You order an AP view of his right shoulder. Question What finding on the plain films may indicate rotator cuff arthropathy? Answer Choices 1 Greater than 2 cm widening of the acromioclavicular joint 2 Cystic changes in the head of the humerus 3 Narrowed glenohumeral joint space 4 High riding humeral head (subluxation) 5 A Bankhardt lesion on the glenoid

high riding hummer head (subluxation) Explanation With large, long-standing tears of the rotator cuff, AP radiographs may reveal a high-riding humerus relative to the glenoid (subluxed), indicative of rotator cuff arthropathy. Widening of the acromioclavicular (AC) joint is indicative of AC joint injury (separation grade 2). Cystic changes in the humeral head are indicative of long-standing rotator cuff tendonitis or bursitis. Narrowed glenohumeral joint space is seen with shoulder joint arthritis. A Bankhardt lesion of the glenoid is indicative of previous traumatic glenohumeral dislocation.

A 68-year-old man presents with progressive onset of lower back pain, unsteady gait, and numbness and occasional weakness in his lower extremities. His symptoms seem worse after a day at work, where he stands for hours at a time. He finds that resting, especially sitting, helps alleviate the symptoms. He has not had any other treatment. Diffuse narrowing of the spinal canal is seen on recent MRI, and the patient now wishes to discuss management of this issue. Question What first-line management should be discussed with this patient? Answer Choices 1 Lumbar epidural steroid injections 2 Anti-inflammatory medications 3 Spinal manipulation 4 Decompressive laminectomy 5 Implement regular exercise

implement regular exercise The correct answer is to implement a regular exercise program. Patients with diffuse spinal canal narrowing have what is known as spinal stenosis and are not restricted in their activity, but they often limit their own activity due to intolerance and pain. As long as there is no evidence of fracture or gross instability, then activity and exercise should be encouraged. This will continue to keep muscles and joints healthy, and will continue to aid the patient in performing activities of daily living. In addition, spinal stenosis tends to be a progressive disease, so maintaining a healthy body is important to slow the progression. As spinal stenosis is a function of nerve compression due to spinal canal narrowing, it is also important to keep the patient's weight under control in order to reduce the added stress that extra weight puts on joints, bones, etc. Lumbar epidural steroid injections is not the correct answer. Steroids, either oral or injectable, should be reserved for severe cases where more conservative measures have not been successful. In addition, there is very little evidence that long-term efficacy of epidural steroid injections exists. Anti-inflammatory medications is not the correct answer. Medications, such as nonsteroidal anti-inflammatories and acetaminophen, can be used for pain in patients with spinal stenosis. However, these medications have side effects when used long-term and could be avoided for a time period or altogether if a patient maintains a good activity level and keeps their weight under control. The gastrointestinal side effects of anti-inflammatories made them a poor long-term management choice. Spinal manipulation is not the correct answer. Spinal manipulation plays no role in the treatment of spinal stenosis because it cannot widen the spinal canal. Decompressive laminectomy is not the correct answer. This answer describes the type of surgical treatment that is reserved for patients who fail more conservative treatment and have pain secondary to spinal stenosis that impedes their lifestyle. This would not be considered first-line treatment.

A 37-year-old man presents with pain in his left shoulder after a motor vehicle accident. The ER doctor suspects that the left shoulder pain is secondary to splenic rupture. What is this sign called? Answer Choices 1 Kehr's sign 2 Cullen's sign 3 Grey Turner's sign 4 Lisker's sign 5 Blumberg's sign

kehr's sign Explanation Kehr's sign is pain in the left shoulder secondary to splenic rupture. Cullen's sign is ecchymosis around the umbilicus that can sometimes be seen with acute pancreatitis. Grey Turner's sign is ecchymosis of the flanks that can sometimes be seen with acute pancreatitis. Lisker's sign is tibial bone tenderness that can sometimes be elicited with deep vein thrombosis. Blumberg's sign is abdominal rebound tenderness. This is an indication of peritoneal irritation.

A 37-year-old man presents with pain in his left shoulder after a motor vehicle accident. The ER doctor suspects that the left shoulder pain is secondary to splenic rupture. What is this sign called? Answer Choices 1 Kehr's sign 2 Cullen's sign 3 Grey Turner's sign 4 Lisker's sign 5 Blumberg's sign

kehr's sign Kehr's sign is pain in the left shoulder secondary to splenic rupture. Cullen's sign is ecchymosis around the umbilicus that can sometimes be seen with acute pancreatitis. Grey Turner's sign is ecchymosis of the flanks that can sometimes be seen with acute pancreatitis. Lisker's sign is tibial bone tenderness that can sometimes be elicited with deep vein thrombosis. Blumberg's sign is abdominal rebound tenderness. This is an indication of peritoneal irritation.

A 49-year-old Caucasian man well known to your practice has come to see you today with pain in his left lower extremity. Upon further questioning, the patient describes doing construction work 3 months prior when he jumped onto his feet from an elevated height of about 5 ft. Since this episode, he has noted increasing issues of left-sided hip and knee pain. He describes the pain as radiating into the left groin and front middle thigh area. The pain is relieved with sitting and aggravated by walking and climbing up stairs. The patient denies any paresthesias, numbness, bowel/bladder dysfunction, fever, night sweats, or chills. Radiographs reveal marked irregularity of the left femoral head with sclerosis, subchondral lucency, and presence of mild collapse. Question What would be the most beneficial intervention in halting the progression of his disease state? Answer Choices 1 Bed rest 2 Initiation of daily use of nonsteroidal anti-inflammatory (NSAID) therapy 3 Joint-preserving procedures 4 Joint replacement 5 Initiation of daily use of ergoloids

joint preserving procedures Explanation Avascular necrosis, sometimes also referred to as osteonecrosis, is osseous cell death resulting from vascular compromise. Common sites that are affected include the proximal or distal femoral head or even the ankle, shoulder, or elbow. Common causes include but are not limited to corticosteroid use, alcoholism, trauma, systemic lupus erythematosus, pancreatitis, gout, or even sickle cell disease. Radiographic findings will include mild density changes early on, sclerosis, and more progressed disease state will lead to the pathognomonic crescent sign, which is subchondral collapse. Later disease states will show joint-space narrowing and degenerative changes in the acetabulum. At this point, the best option for this patient in halting the actual disease state of avascular necrosis is having a joint preservation procedure. Procedures such as core decompression, vascularized fibular graft, bone marrow grafting, or non-vascularized bone grafting are just a few procedures used to help preserve the natural joint of the patient. Total joint replacements in patients with osteonecrosis have been found to have extremely poor long-term survival rates of the prosthetic devices used. More investigation is being done to help increase the long-term endurance of this option. All non-operative options, such as bed rest, partial weight-bearing with crutches, weight-bearing as tolerated, or the use of NSAIDs, have not been found to be effective in halting the disease. The use of ergoloids and other pharmaceutical agents has also been investigated but has not produced promising success.

A 3-year-old child suddenly dies while playing. According to the parents, the child did not have any serious illnesses. 8 weeks prior to death, the child suffered from a fever accompanied by skin rashes. The child's symptoms subsided after treatment. The autopsy reveals a massive aneurysmal dilatation of the long length of the proximal coronary arteries with an occluding thrombus. Histologically, the arteries show an intense transmural arteritis. The arteries in the other organs are normal. What is the probable diagnosis? Answer Choices 1 Takayasu's arteritis 2 Giant cell arteritis 3 Kawasaki disease 4 Microscopic polyangiitis 5 Polyarteritis nodosa

kawasaki disease Kawasaki disease is an acute arteritis involving large, medium sized, and small arteries (often the coronary arteries) and is associated with mucocutaneous lymph node syndrome. The mucocutaneous syndrome is characterized by fever, conjunctival and oral erythema, edema of the hands, feet, erythema of the palms and soles, a skin rash (often with desquamation), and enlargement of the cervical lymph nodes. It is usually self-limited. Approximately 20% develop cardiovascular sequelae with the range of severity from asymptomatic vasculitis of the coronary arteries, coronary artery ectasia, or aneurysm to giant coronary artery aneurysm (7 to 8 mm) with rupture or thrombosis, myocardial infarction, or sudden death. Acute fatalities occur in 1% of the cases due to coronary artery thrombosis or rupture of coronary artery aneurysm. Vasculitis resembles poly arteritis nodosa with necrosis and pronounced inflammation, affecting the entire thickness of the vessel wall. Takayasu's arteritis is a granulomatous vasculitis of medium sized and larger arteries. Takayasu first described it in 1908 as a clinical syndrome characterized by ocular disturbances and marked weakening of the pulses in the upper extremities (Pulseless disease). This is related to the fibrous thickening of the aortic arch with narrowing, or virtual obliteration, of the origins or more distal portions of the great vessels arising in the arch. Microscopically, there is adventitial mononuclear infiltrate with perivascular cuffing of the vasa vasorum (initially). Later, there is intense mononuclear inflammation in the media, in some cases accompanied by granulomatous changes replete with giant cells and patchy necrosis of the media. In addition, when it heals, the inflammation is replaced by marked collagenous fibrosis involving all the layers of the vessel wall accompanied by lymphocytic infiltration. Giant cell arteritis, or temporal arteritis, is the most common of the vasculitides. It is an acute, and chronic, granulomatous inflammation of the medium and small arteries. It affects mainly the temporal, vertebral, and ophthalmic arteries. The short segments of 1 or more arteries develop nodular thickening with the reduction of the lumen, which may become thrombosed. Histologically, there is granulomatous inflammation of the inner half of the media, centered on the internal elastic membrane. It is marked by mononuclear infiltrate, multinucleated giant cells of both foreign body and Langhans type, and fragmentation of the internal elastic lamina. Sometimes the granulomas may be absent, and there is only nonspecific polyarteritis without giant cells. The healed stage of both will reveal only collagenous thickening of the vessel wall. Polyarteritis nodosa is systemic vasculitis characterized by necrotizing inflammation of the small or medium sized vessels, typically involving renal arteries and visceral vessels but sparing the pulmonary circulation. There is neither glomerulonephritis nor vasculitis of the arterioles, capillaries, and venules. It particularly involves the branching points, also known as the points of bifurcation. The involvement is segmental and may involve only a portion of the circumference. It causes segmental erosion with weakening of the arterial wall with aneurysmal dilatation or localized rupture. Initially, there is transmural inflammation of the vessel consisting of neutrophils, eosinophils, and mononuclear infiltrate with fibrinoid necrosis. Later, the inflammation is replaced by collagenous fibrosis. Within the same vessel, various stages of inflammation may be seen. Microscopic polyangiitis is also called hypersensitivity, or leukocytoclastic vasculitis. It generally affects arterioles, capillaries, and venules. The lesions are thought to represent a hypersensitivity reaction that involves the skin, mucous membranes, lungs, brain, heart, gastrointestinal tract, kidneys, and muscle. Necrotizing glomerulonephritis and pulmonary capillaritis is common. The lesions are histologically similar to polyarteritis nodosa, but muscular and large arteries are spared. Histologically, segmental fibrinoid necrosis of the media may be present; however, in some, the change is limited to infiltration with neutrophils. Greater than 80% of the patients have ANCA (anti-neutrophil cytoplasmic antibodies), most often p-ANCA. In many cases, reaction to an antigen such as drugs (e.g. penicillin), microorganisms (e.g. streptococci), heterologous proteins, or tumor antigens can be traced as the precipitating cause, but there are few or no immune deposits in this type of vasculitis. Simple removal of the offending agent may help most patients with cutaneous vasculitis, but those with systemic disease may develop organ failure unless treated. Refer to the table for additional information.

A 33-year-old female presents for routine follow-up with an abnormal Computed Tomography (CT). Her past medical history includes a leg amputation four years ago for osteogenic sarcoma treated with neoadjunctive chemotherapy. She is married and a non-smoker. Her physical exam includes a BP-111/67 mm Hg, pulse-70/min, and a healed laparotomy incision for a colon resection from perforated diverticulitis. A chest CT demonstrates a new solitary, irregular, non-calcified 3 cm nodule. A CT guided biopsy demonstrates metastatic osteogenic sarcoma. Which is a negative prognostic indicator? Answer Choices 1 Long disease-free interval 2 Lymph node involvement 3 Complete surgical resection 4 Long tumor doubling time 5 Low number of metastasis

lymph node involvement Explanation Pulmonary metastases may present with symptoms similar to primary lung cancer. These include chest pain, cough, hemoptysis, and dyspnea. Usually metastatic lesions are detected with chest X-ray or CT. CT can characterize suspicious findings on chest X-ray and detail the number of metastasis. CT detects approximately 80% of pulmonary metastases detected at surgical exploration. Positron Emission Tomography (PET) scanning is currently being investigated as a modality to access for pulmonary metastases. In patients with a previous history of sarcoma, a new pulmonary nodule has a 92% probability of being a metastatic nodule. Patients who are candidates for resection with pulmonary metastasis include those who have the a controllable or controlled primary tumor, no extrapulmonary tumor, no better method of treatment available, and adequate medical status. Tumor pathology that benefits from resection under these circumstances includes sarcoma, germ cell tumors, colorectal tumors, breast tumors, hypernephroma, head and neck tumors, melanoma, and hepatocellular carcinoma. Approximate five year survival for colorectal is 37%, breast 37%, kidney 41%, sarcoma 31%, melanoma 21%, and germ cell tumors 68%. Surgical approaches include unilateral or bilateral thoracotomy, sternotomy, and video-assisted thoracic surgery. Prognostic factors include tumor doubling time, disease-free interval, number of metastasis, lymph node metastases, and complete resection. Primary pulmonary thymoma is rare. Treatment includes surgical resection and possibly post-operative radiation.

What is the most common bacterial pathogen isolated in otitis externa? A Group A streptococcus B Strep pneumonae C Pseudomonas aeruginosa D Hemophilus influenza E Aspergillus

pseudomonas Staphylococcus aureus and pseudomonas are the most common pathogens isolated in otitis externa. Proteus, diphtheroids, and E. coli are also causes.

The fracture of a patient's lower humerus results in the inability to extend the wrist. What nerve was damaged in the injury? Answer Choices 1 Median 2 Ulnar 3 Radial 4 Musculocutaneous 5 Can not be determined from the information given

radial Explanation Damage to the median and ulnar nerves would limit the ability to flex the wrist and fingers. Damage to the musculocutaneous nerve would limit the ability to flex the forearm. Therefore, because the radial nerve innervates the extensor (posterior) forearm muscles, radial nerve must be the correct answer.

A 27-year-old man is admitted to the hospital following a motor vehicle accident. He sustained lacerations to his arms bilaterally and has fractures of the right tibia and fibula. A cast is placed and the patient is scheduled for surgery the following day. A few hours after the cast is placed, he develops severe pain; the pain is unresponsive to several doses of intravenous morphine. His pain increases when he extends his right leg. Peripheral pulses are weak, but present. Question Based on the history and physical, what should be done next? Answer Choices 1 X-ray of the tibia and fibula 2 MRI of the right leg 3 Remove cast and check compartment pressure 4 Ultrasound of the lower extremity 5 Monitor and continue giving narcotics

remove cast and check compartment pressure The patient should have the cast removed and compartment pressure measured. This patient is showing signs of compartment syndrome. Compartment syndrome develops when there is accumulation of pressure within a muscle compartment. It typically occurs following trauma to a limb. Patients typically present with pain unrelieved by analgesia as well as pain with extension of the involved limb. As the swelling within the compartment worsens, patients develop absence of pulse, paresthesias, pallor, and poikilothermia. The compartment pressure should be checked, and if it is elevated, the patient should be taken for fasciotomy. An X-ray of the tibia and fibula would not demonstrate compartment syndrome. An MRI of the right leg would not be useful in this patient. An ultrasound of the lower extremity would be useful in diagnosing a lower extremity DVT. However, the patient's presentation is highly suggestive of compartment syndrome, and it should be ruled out before pursuing any other diagnoses. Monitor and continue giving narcotics is incorrect. If compartment syndrome is left untreated, it can lead to tissue death.

A 65-year-old woman presents to the office with decreased hearing, and pain over her sternum, pelvis, and her right tibial tubercle. On x-ray, the involved bones are noted to be expanded and denser than normal. Her serum calcium and phosphorus levels are normal, but serum alkaline phosphatase level is markedly elevated. Which of the following would be the appropriate initial treatment for this patient? A ibuprofen 600 mg po every 6 hours B indomethacin 25 mg po tid C meclizine 25 mg po tid D methotrexate 7.5 mg po qd E tiludronate 400 mg po qd

tiludronate 400 mg po qd This patient's signs and symptoms are consistent with Paget disease of bone. Biphosphates have become the treatment of choice for this disease. Tiludronate, taken orally for 3 months, is very effective in treatment of this disease.

A 49-year-old Caucasian man presents with pain in his left lower extremity. During questioning, the patient states that while doing construction work 3 months prior, he jumped from an elevated height of 6 feet; ever since this episode, he has noted increased issues with left-sided hip and knee pain. He describes the pain as radiating into the left groin and front middle thigh area. The pain is relieved with sitting and aggravated by walking and climbing up stairs. The patient denies any paresthesias, numbness, bowel or bladder dysfunction, fever, night sweats, or chills. Pertinent medical history includes a 20-year extensive history of alcohol. A radiograph interpretation shows the presence of a crescent sign, as well as marked irregularity of the left femoral head with sclerosis. Question Considering the most likely diagnosis, what would be the ultimate clinical intervention necessary for this patient? Answer Choices 1 Total hip replacement 2 Vascularized bone grafting 3 Daily oral bisphosphonates 4 Daily oral corticosteroids 5 Nonvascularized bone grafting

total hip replacement Avascular necrosis, sometimes also referred to as osteonecrosis, is osseous cell death resulting from vascular compromise. Common sites that are affected include the proximal or distal femoral head or even the ankle, shoulder, or elbow. Causes include, but are not limited to corticosteroid use, alcoholism, trauma, systemic lupus erythematosus, pancreatitis, gout, or even sickle cell disease. Radiographic findings will include mild density changes early on and sclerosis, while a more progressed disease state will lead to the pathognomonic crescent sign, which is subchondral collapse. Later disease will show joint-space narrowing and degenerative changes in the acetabulum. The age of this patient would make the treating healthcare provider highly pursue conservative efforts to help preserve the affected joint. Unfortunately, with progressive process of this disease, most likely the patient in the above scenario will need to have a total hip replacement. The initial treatment plan usually includes avoidance of weight bearing of the affected joint for several weeks. Vascularized or nonvascularized bone grafting is sometimes used to help preserve the femoral head but only delays the ultimate clinical intervention of a total hip arthroplasty. Daily oral corticosteroids and bisphosphonates are both incorrect and in fact on occasion may lead to the development of osteonecrosis.

A 40-year-old woman presents occasional constipation that she relates to her diet; She is married with 2 sons. Aside from mild anxiety, she is dealing appropriately with her duties as a librarian. She does not smoke; she consumes alcohol only recreationally, and she has a body weight of 187 lbs. She is well-nourished and communicates well. Lung, cardiac, abdominal, and neurological examinations are unremarkable. Thyroid is normal in size and consistency. Menses are normal, and drug history is negative except for "hormone pills" for contraception. Family history is remarkable for a mother with hypothyroidism and osteoporosis. Question What strategy is recommended for screening osteoporosis in this patient? Answer Choices 1 Bone densitometry after menopause 2 Hip and spinal X-ray after menopause 3 Bone densitometry at 65 years of age 4 Hip and spinal X-ray at 65 years of age 5 Bone densitometry at 55 years of age

3 Bone densitometry at 65 years of age The U.S. Preventive Services Task Force (USPSTF, 2002) recommends screening for osteoporosis among women aged 65 or older and women aged 60-64 with risk factors for osteoporotic fractures. This patient is 40 years old and is not within the age range to be at risk for osteoporsis which is 50+ years old. Bone densitometry or dual-energy X-ray absorptiometry (DEXA scan) from the femoral neck is currently the best screening measure used (Maghraoui and Roux, 2008). Body weight less than 154 lbs. and no current use of estrogen therapy are major factors associated with an increased risk for osteoporosis and fracture (USPSTF, 2002). In the presented case, an appropriate decision on the timing of initial bone densitometry should be based on the patient's osteoporosis risk factors at the age of 65. However, the most appropriate choice among others is bone densitometry at 65 years of age.

A 45-year-old man presents to his orthopedic office for a 1-month history of worsening pain in his left groin and buttock. He had a surgical hip fracture repair approximately 4 months ago, 2 days after a fall in a bowling alley. His post-operative course was unremarkable, with normal healing and return to activities. However, he now notes increasing difficulty with walking and has had to limit activities again due to pain. He has tried "all" of the over-the-counter pain medications without relief. His past medical history is unremarkable, aside from the surgical fracture repair. He has no chronic medical conditions, no other surgeries, and takes no chronic medications. He has no medication allergies. On physical exam, the patient is asked to walk across the room. He limps and appears to be in pain. The entire hip region is evaluated and no erythema, ecchymosis, or edema is noted. The incision site is healing well, with no tenderness and no fluctuant tissue. With range of motion tests, the patient reports pain, especially with internal rotation of the left hip. He is sent for several tests. Blood culture is negative. X-ray shows a crescent sign over the left femoral head. An MRI shows a low-signal intensity on the left femoral head, and increased uptake is seen also in this region with nuclear bone scan. The fracture site shows normal post-operative changes. Question What best describes the underlying mechanism responsible for this patient's complication from his hip fracture? Answer Choices 1 Autoimmune destruction of synovial membrane 2 Damage to bone marrow, resulting in decreased white blood cell immunity 3 Damaged vascular supply to femoral head 4 Development of traumatic arthritis 5 Introduction of bacteria during surgery

3 Damaged vascular supply to femoral head This patient's condition is suspicious for avascular necrosis (or the more preferred term, osteonecrosis) of the hip. Avascular necrosis (AVN) is a relatively common complication following a traumatic hip dislocation and/or fracture and results in damaged vascular supply to the femoral head, with subsequent bone death. Some potential conditions on the differential may include arthritis, malignancy, and osteomyelitis. The history and physical exam may be similar in all these conditions, but the history of hip fracture and the diagnostic studies support a diagnosis of AVN. Definitive treatment is surgical, with a variety of approaches, from core decompression to bone grafting. Autoimmune destruction of synovial membrane is one of the mechanisms associated with rheumatoid arthritis. Damage to bone marrow, resulting in decreased white blood cell immunity, is not a process specific to one condition. Because only a minimal portion of one bone is affected, this patient's marrow production of red and white blood cells would be expected to be normal. Development of traumatic arthritis is another potential complication following hip fracture and/or dislocation. However, this patient's current findings show bone death, not joint inflammation. He could still be at risk for developing post-traumatic arthritis, but this is not the current explanation of his symptoms. Introduction of bacteria during surgery, leading to osteomyelitis or a joint infection, is a possible complication of this patient's hip fracture (but less common than AVN). If a bacterial infection were responsible for this patient's symptoms, his blood culture would be expected to be positive for a causative organism, and imaging findings would differ.

A 24-year-old man presents with radial-sided wrist pain following a fall on his outstretched hand 3 days ago. He did not seek immediate medical attention due to a lack of swelling. Despite self-treatment with ice and with analgesics, his wrist still hurts, especially with ulnar deviation and while trying to open a jar or grip the steering wheel of his car. On examination, his tenderness seems to be in the anatomic snuffbox of the wrist. Radiographs are normal in all views. What is the proper treatment for this man at this time? Answer Choices 1 Elastic bandage for support, continue ice and analgesics, follow up PRN 2 Wrist volar splint, continue ice and analgesics, follow up in a month 3 Thumb spica splint and refer to an orthopedic surgeon 4 Immediate referral to an orthopedic surgeon 5 Routine referral to an orthopedic surgeon

3 Thumb spica splint and refer to an orthopedic surgeon If a patient with a suspected scaphoid fracture has radiographs that are read as 'normal', but clinically there is a fracture, place the patient in a long arm thumb spica splint and refer the patient to an orthopedic surgeon. Elastic bandage and volar splints are not adequate immobilization for a possible scaphoid fracture. Immediate referral is not warranted because you have not identified a positive diagnosis, and routine referral is not warranted for the same reason. However, if after a positive diagnosis there is also evidence of non-union or worsening fracture gap, a referral to an orthopedic surgeon is definitely warranted.

An 18-year-old man presents 30 minutes after falling on his outstretched arm while skateboarding. He is guarding his left forearm near his wrist with his right hand, and he has his left arm against his body for support. There is a 'dinner fork' deformity with edema and ecchymosis at the wrist. He is neurovascularly intact and the skin is closed. With this type of deformity, what fracture is most likely? Answer Choices 1 Smith fracture 2 Torus fracture 3 Salter-Harris IV fracture 4 Colles' fracture 5 Dye-punch fracture

4 Colles' fracture Explanation This patient presents with a Colles' fracture. This fracture causes the 'dinner fork' appearance of the wrist, as the hand and distal fracture fragment of the distal radius (or distal radius and ulna) are dorsally angulated. A torus fracture is the buckling of the cortex of the bones of the wrist (radius or ulna), and it usually lacks an angulation deformity. Salter-Harris fractures are seen in growing children. This patient is likely skeletally mature. A dye-punch fracture may be angulated as well, but it is a compression of the radial articular surface. In Smith fractures, there is palmar displacement of the distal fragment of radius, in contrast to the dorsal displacement seen in Colles' fractures.

A 17-year-old boy presents with a 5-day history of intermittent fever, joint pain, and redness and swelling of the joints. The patient gives a history of pain in the right knee and ankle. 3 days ago, he had pain and swelling in his left knee, but now it has improved. On examination, temperature is 102° F, pulse is 108/min, RR is 20/min, and BP is 110/80 mm Hg. The patient's right knee is swollen, tender, and warm. There is a limitation of range of motion due to pain. The right ankle appears swollen and warm. Other system exams are normal. Lab tests are ordered; during the follow-up exam you note elevated erythrocyte sedimentation rate (ESR) and C-reactive protein (CRP). Question How long should the patient should receive secondary prophylactic antibiotics? Answer Choices 1 10 days 2 1 month 3 6 months 4 1 Year 5 5 years

5 years The correct response is 5 years. Rheumatic fever is an inflammatory condition that is precipitated by group A streptococcal infection. It is more common in children, but it also occurs in adults. The symptoms include joint pain, carditis, fever, subcutaneous nodules, abdominal pain, nosebleeds, and chorea. It is more commonly diagnosed in close living quarters, such as a dormitory. The diagnosis is established by a positive group A streptococcal culture, plus fulfillment of the modified Jones criteria. Modified Jones criteria Must have 2 major findings, or 1 major and 2 minor findings: MAJOR FINDINGS MINOR FINDINGS Polyarthritis Fever Chorea Elevated erythrocyte sedimentation rate/CRP Carditis Prolonged PR interval on EKG Erythema marginatum Arthralgia (if arthritis not counted as a major finding) Subcutaneous nodules Supporting evidence of preceding group A streptococcal infection: a positive throat culture, a rapid streptococcal antigen test, or an elevated or rising streptococcal antibody titer This patient already meets the criteria to establish the diagnosis of rheumatic fever. Long-term antibiotic prophylaxis is indicated because recurrence of rheumatic fever is common, especially during the 1st 5 years. Adults with rheumatic fever usually receive at least 5 years of prophylactic antibiotics, usually administered as 1.2 million units of benzathine penicillin G IM each month, or penicillin VK 250 po bid. The prophylactic regimen is life-long for certain patients, such as those who already have pre-existing valvular heart disease. The duration of secondary prophylaxis for RF without carditis is 5 years or until the patient reaches the age of 21 years (whichever is longer). The duration of secondary prophylaxis for RF with carditis and no residual heart disease is 10 years or well into adulthood (whichever is longer). The duration of secondary prophylaxis for RF with carditis and residual heart disease is for a minimum of 10 years since the last episode; sometimes, prophylaxis is required for life. Because the mechanism is thought to be an inflammatory autoimmune reaction to the streptococcal infection, aspirin is the mainstay of symptomatic treatment. It is sometimes necessary to take relatively large doses of aspirin until the symptoms are controlled. Untreated rheumatic fever can lead to mitral stenosis, congestive heart failure, and an increased risk of recurrent episodes of rheumatic fever.

A 45-year-old Caucasian woman presents with a 3-month history of hot flashes. She complains of occasional night sweats, extreme heat sensitivity, and mood swings. Her last menstrual period was 12 months prior to presentation; it followed an 8-month period of oligomenorrhea. She denies weight loss and fevers. She has no significant medical or surgical history. She denies any history of bone fractures. She does not take any medications. The patient is married and has 2 children. She works as a dental assistant. She denies nicotine, alcohol, or any illicit drugs. Her family history is significant for a sister who had pre-menopausal breast cancer at age 43 and a mother with severe osteoporosis, with a resultant Colles' fracture, femoral neck fracture, and multiple vertebral compression fractures. Physical examination shows a slightly overweight woman. Vital signs are unremarkable. Breast exam is normal. Gynecologic exam shows slightly dry and atrophic vaginal mucosa, with otherwise normal findings. Stool sample is negative for occult blood. The remainder of her physical examination is unremarkable. Remarkable laboratory studies include normal liver function tests and normal cholesterol profile. She underwent a dual-energy X-ray absorptiometry (DEXA scan) with the following results: Bone Mass Density (BMD) T Score Z Score Lumbar Spine (L2-L4) 1.036 -1.4 -0.2 Hip (Femoral Neck) 0.898 -.70 +0.4 The T score is an index of the patient's BMD compared to the average young normal female BMD. According to the World Health Organization, low bone mass (osteopenia) is a T score between -1 and -2.5 SD. Osteoporosis is a T score at or below -2.5. The Z score is a measure of how a patient's BMD compares with age-matched females. Question Based on these results and the patient's history, what treatment program would be most appropriate? Answer Choices 1 Advise a weight-bearing aerobic exercise program, calcium carbonate (1500 mg/day) and vitamin D supplementation 2 Advise medroxyprogesterone to relieve hot flashes 3 Combination therapy (estrogen and progestin) 4 Estradiol therapy 5 Alendronate (Fosamax) therapy

Alendronate Alendronate (Fosamax) is a bisphosphonate used for the treatment and prevention of osteoporosis. This medicine is taken on an empty stomach and requires the person to be sitting or standing (without bending) at least 30 minutes to 1 hour after its intake in order to prevent esophagitis. Because bisphosphonates may form insoluble complexes with food or drink, this patient should limit her ingestion to water while taking alendronate. This is the best treatment option for the patient in this question due to her medical and family history and her ability to comply with the manner of taking this medication. This patient is experiencing the effects of reduced estrogen levels because of menopause. The patient's DEXA scan results show a low bone density (osteopenia) in the lumbar spine, but she does not have osteoporosis. She has several risk factors for osteoporosis, including being a Caucasian woman, having a family history of osteoporosis, and undergoing menopause at age 45. Other risk factors for osteoporosis include: Caucasian or Asian race Cigarette smoking High caffeine or alcohol intake High protein diet Lean Body Frame Sedentary Lifestyle Because her sister had pre-menopausal breast cancer, she has a relative contraindication to taking estrogen therapy. Postmenopausal hormone replacement therapy (PHT) has been shown to have benefits in preventing osteoporotic fractures and colorectal cancer1. According to the American Heart Association, women should not take postmenopausal hormone therapy (PHT), formerly called HRT, or selective estrogen receptor modulators (SERM) for the purpose of reducing the risk of coronary heart disease or stroke4. According to the Heart and Estrogen/Progestin Replacement Study (HERS), a large study conducted by Dr. Deborah Grady at the University of California San Francisco, there was no reduction in the risk of heart attack or stroke among women who took the hormones for 4 years. Estrogen raises the high-density lipoprotein (HDL) cholesterol and lowers the LDL (low-density lipoprotein) cholesterol. Progesterone has the opposite effect on cholesterol levels; however, the overall protective effect of estrogen has been linked to its inhibition of vascular constriction and direct action on the arterial walls. Women who use unopposed estrogen therapy were at increased risk of breast cancer after longer-term use2. Another study showed that women using estrogen and testosterone therapies have a significantly higher risk of invasive breast cancer3. Endometrial cancer incidence is significantly elevated when unopposed estrogen is used, as compared with nonusers. There is increased incidence of thromboembolic events, including deep vein thrombosis and pulmonary embolism among estrogen users, with the highest incidence in the 1st year of use. Among the benefits of PHT is the relief of genital atrophy, vaginitis, relief of hot flashes, and dyspareunia. The HRT administration regimen may involve continuous estrogen and progestin (most commonly oral medroxyprogesterone), thus avoiding the withdrawal bleeding, or continuous estrogen with progestin 14 days a month. However, studies have shown that women receiving PMH (postmenopausal hormone) therapy containing estrogen and testosterone therapies have a significantly increased risk of invasive breast cancer. Weight-bearing exercise, calcium supplementation, and vitamin D are all preventive measures for osteoporosis.

Approximately what percentage of patients with a solid primary tumor elsewhere will end up with metastatic disease of the vertebrae during the clinical course of their cancer? A 10% B 20% C 30% D 40% E 50%

50% Fifty percent of cancer patients will develop metastatic disease of the vertebrae at some point during the course of their illness. The highest percentages of cancers that lead to such spinal lesions are carcinomas of the breast, lung, prostate, colon, thyroid and kidney. This likely occurs through hematogenous spread. Pain is a common presenting symptom, but it may be found while still asymptomatic if routine screenings are done with bone scans, MRI or CT in patients with a known primary tumor elsewhere in the body. The pain is usually worse with weight bearing activities and better when lying down, but pain that persists through the night and prevents sleep needs to be evaluated for possible neoplasm. Some metastatic disease of the spine may present with neurological symptoms such as sensory or motor deficits following a spinal nerve root distribution or more generalized neurological deficits due to spinal cord compression or cauda equina syndrome.

A 49-year-old man, who is well known to your practice, presents with pain in his left lower extremity. Upon further questioning, the patient describes doing construction work 3 months prior during which he jumped onto his feet from an elevated height of about 5 ft. Since this episode, he has noted increasing issues of left-sided hip and knee pain. He describes the pain as radiating into the left groin and front middle thigh area. The pain is relieved with sitting, and it is aggravated by walking and climbing up stairs. The patient denies paresthesias, numbness, bowel/bladder dysfunction, fever, night sweats, and chills. Radiographs performed reveal marked irregularity of the left femoral head with sclerosis, subchondral lucency, and the presence of mild collapse. Question Given these findings, what would be highest on your differential diagnosis? Answer Choices 1 Multiple myeloma 2 Osteoporosis 3 Avascular necrosis 4 Osteoarthritis 5 Rheumatoid arthritis

AVN Avascular necrosis, which is sometimes also referred to as osteonecrosis, is osseous cell death resulting from vascular compromise. Common sites that are affected include the proximal or distal femoral head or even the ankle, shoulder, or elbow. Common causes include, but are not limited to, corticosteroid use, alcoholism, trauma, systemic lupus erythematosus, pancreatitis, gout, or even sickle cell disease. Radiographic findings will include mild density changes early on; sclerosis and a more progressed disease state will lead to the pathognomonic crescent sign, which is subchondral collapse. Later disease states will show joint-space narrowing and degenerative changes in the acetabulum. Multiple myeloma typically initially presents as spine, rib, or proximal long bone pain, and it is more commonly in older adults, with the median age of presentation being 65 years. Many of these patients will present with signs and symptoms relating to pathology, including anemia, infections, kidney failure, spinal cord compression, or even hyper-viscosity syndrome. Radiologic evidence of multiple myeloma is expressed as punched-out lytic lesions, diffuse osteopenia, or areas of fracture. Osteoporosis is not common in this patient population group; therefore, it is much lower on the differential diagnosis. Its occurrence is higher in Caucasian women with other risk factors, such as tobacco or alcohol abuse, excess/deficiency in hormones, malignancy, and some genetic disorders. Diagnosis of osteoporosis is initiated from assessment of the microarchitecture,which is accomplished through a bone biopsy. Although the patient may have some signs of osteoarthritis on future imaging studies, it is most likely not the main cause of what has brought him in to see you now. Major radiographic features of osteoarthritis include joint space narrowing, subchondral sclerosis, osteophytes, and subchondral cysts. Rheumatoid arthritis has an insidious onset, with characteristic morning stiffness; it symmetrically affects small joints of the hands and feet. Key radiologic findings in rheumatoid arthritis range from normal early in disease to soft tissue swelling or periarticular swelling. Eventually severe forms of the disease will be evident by progressive erosion through the cortex of the bone around the margins of the joint, especially the metacarpophalangeal and proximal interphalangeal joints.

A 59-year-old woman presents with right knee pain. The pain has progressed over the last 2 years and worsens with physical activity. She notes a decreased range of motion. She denies any known injury to the area. Past medical history includes Crohn's disease and hypertension. She is currently on mesalamine and metoprolol. X-ray of the right knee reveals joint space narrowing and osteophytes. Question What is the recommended initial medication in this patient? Answer Choices 1 Naproxen 2 Acetaminophen 3 Oxycodone 4 Methotrexate 5 Prednisone

Acetaminophen is correct. The patient has symptoms of knee pain and x-ray findings of joint narrowing with osteophyte formation. These findings are consistent with osteoarthritis. Acetaminophen is the recommended first-line treatment for osteoarthritis. Studies have shown that their efficacy is equal to NSAIDs. Naproxen is incorrect. Naproxen is an NSAID. While NSAIDs are used in the treatment of osteoarthritis, this patient has a history of Crohn's disease. Patients with Crohn's disease should avoid NSAIDs, as they may exacerbate the disease. Oxycodone is incorrect. Narcotic medications should be avoided in chronic conditions such as osteoarthritis. Methotrexate is incorrect. Methotrexate may be used in the treatment of rheumatoid arthritis, but not osteoarthritis. Prednisone is incorrect. While prednisone could alleviate the symptoms of osteoarthritis, it has a side effect profile and should be avoided.

A 17-year-old high school football player (punter) was tackled and forcefully slammed onto his right dominant shoulder 2 hours ago during a game. He had immediate pain, but was able to continue punting. He has full active and passive range of motion, but some pain (4/10) with abduction. There is no obvious deformity, and the skin over the shoulder is intact and not tented. There is no crepitus on palpation while moving the arm. Most of his tenderness seems to be on the superolateral right shoulder, with pain over the AC joint. Question What is the most likely diagnosis? Answer Choices 1 Clavicle fracture 2 Sternoclavicular dislocation 3 Acromioclavicular separation, Grade III 4 Glenohumeral dislocation 5 Acromioclavicular separation, Grade I

Acromioclavicular separation, Grade I Explanation This is most probably a Grade I acromioclavicular separation. A fall directly onto the shoulder can result in either an acromioclavicular (AC) separation or a clavicle fracture. Since this patient had no deformity, crepitus, and was able to continue punting, the likelihood of fracture is very low. Also with a Grade III AC separation the joint is completely displaced and deformity is obvious. Movement or activity (such as punting) would cause considerable pain. Sternoclavicular dislocation would have pain more centrally located near the sternum. Glenohumeral dislocation would have deformity and activity would be very limited.

A 17-year-old high school football player (punter) was tackled and forcefully slammed onto his right dominant shoulder 2 hours ago during a game. He had immediate pain, but was able to continue punting. He has full active and passive range of motion, but some pain (4/10) with abduction. There is no obvious deformity, and the skin over the shoulder is intact and not tented. There is no crepitus on palpation while moving the arm. Most of his tenderness seems to be on the superolateral right shoulder, with pain over the AC joint. Question What is the most likely diagnosis? Answer Choices 1 Clavicle fracture 2 Sternoclavicular dislocation 3 Acromioclavicular separation, Grade III 4 Glenohumeral dislocation 5 Acromioclavicular separation, Grade I

Acromioclavicular separation, Grade I This is most probably a Grade I acromioclavicular separation. A fall directly onto the shoulder can result in either an acromioclavicular (AC) separation or a clavicle fracture. Since this patient had no deformity, crepitus, and was able to continue punting, the likelihood of fracture is very low. Also with a Grade III AC separation the joint is completely displaced and deformity is obvious. Movement or activity (such as punting) would cause considerable pain. Sternoclavicular dislocation would have pain more centrally located near the sternum. Glenohumeral dislocation would have deformity and activity would be very limited.

A 25-year-old sexually active woman presents with a 2-day history of pain and swelling of her dorsal right wrist and fingers. For the past week, this pain and swelling has been in different joints of her body, including her left knee, left elbow, and right ankle. On exam, you note edema, effusion, and erythema over the dorsal right wrist with the wrist held in 15 degrees of extension. It is very tender on palpation and has virtually no range of motion secondary to the pain. You aspirate synovial fluid for lab studies. The lab studies reveal the following information: Synovial fluid: WBCs - 57,000 cells/mcL; Gram stain: too numerous to count WBCs with no bacteria seen; Culture: pending. Question Based on this history, physical, and lab findings thus far, what should be the initial treatment for this woman? Answer Choices 1 Prescribe oral doxycycline for 10 days 2 Admit to the hospital for IV ceftriaxone (Rocephin) 3 Wait for the results of the synovial fluid culture 4 Admit for immediate surgical arthroscopic irrigation 5 Admit for IV penicillin

Admit to the hospital for IV ceftriaxone (Rocephin) Explanation Gonococcal septic arthritis, which this woman most likely has based on age, history, and physical and lab findings, should be admitted for IV ceftriaxone (Rocephin) until culture results return and have sensitivities determined. Oral treatment, especially with doxycycline, is ineffective in treating gonococcal septic arthritis. One should not wait for culture results to start treatment. Surgical arthroscopic irrigation may be needed if after 24-48 hours there is no improvement on the IV antibiotic treatment. 4 to 5% of gonococcal isolates produce a β-lactamase that confers penicillin resistance; therefore, it is not first-line treatment.

A 45-year-old man presents for evaluation of ongoing left hip pain since falling from a roof 8 weeks ago. The patient was evaluated at the ER right after the fall and had a left hip X-ray done that was unremarkable. He states: "I have had a minor limp since the fall but now I can barely stand on the leg." He rates the pain 10/10 when weight bearing or moving the leg. He states the pain radiates to his groin and anteriomedial thigh. He denies fever, urinary complaints, numbness, or weakness. Physical exam reveals tenderness on palpation of the left hip and increased pain with active and passive range of motion in all planes. Distal pulses are normal; sensation is fully intact; and calf compartments are soft and compressible without a palpable mass. When asked to stand on his left leg, he is unable to do so secondary to pain and has resorted to using a wheel chair to maneuver. He has no chronic medical problems and takes no medications. You obtain an MRI of the hip, which reveals evidence of osteonecrosis. Question What is most appropriate next step? Answer Choices 1 Discharge home with return precautions and instructions to follow-up with an orthopedist within the next 2 weeks. 2 Initiate antibiotics and discharge home with a mandatory recheck with his PCP within 48 hours. 3 Admit to the hospital for an inpatient orthopedic evaluation. 4 Admit to the hospital and initiate anticoagulation therapy. 5 Initiate NSAID therapy and discharge home with instructions to follow-up with rheumatology.

Admit to the hospital for an inpatient orthopedic evaluation. The correct answer is to admit this patient to the hospital for orthopedic consultation. This patient presents with avascular necrosis of the left hip, and there are several concerning factors in this patient's presentation. First, the diagnosis of avascular necrosis (AVN) in a man at the age of 45 is likely to require surgical repair;1 second, his pain level of 10/10 will likely be difficult to control on an outpatient basis; and lastly, the patient is unable to bear weight, making him a significant fall risk. While it is unclear if the patient will require emergent surgery at this time, it is appropriate to admit this patient for an immediate orthopedic consultation. There is no evidence of secondary infection at this time that would prompt the initiation of antibiotics. There is no evidence that is concerning for a DVT in this patient that would require initiation of anticoagulation therapy. Additionally, there is the possibility of the patient undergoing operative care, and therefore, you would not want to anticoagulate him at this time. Osteonecrosis is not a rheumatologic disease, and involvement of rheumatology is not necessary.

A 66-year-old man presents with a 2-day history of a red, hot, swollen ankle. He denies fever, chills, headache, and any other symptoms. He is unable to put his weight on the foot, and he denies any trauma. Laboratory analysis shows negatively birefringent crystals in the synovial fluid; there is no evidence of bacterial growth. The patient receives treatment, and his condition improves rapidly. However, 6 months later, he has a similar attack. Question What treatment is used in the prophylaxis of this condition? Answer Choices 1 Allopurinol 2 Sulfinpyrazone 3 Corticosteroids 4 NSAIDs 5 Antibiotics

Allopurinol This patient has gout, which is a form of inflammatory arthritis. It is the body's response to the deposition of uric acid crystals in the joints. Gout presents as acute monoarticular arthritis in 90% of patients. In early gout, usually only 1 or 2 joints are involved. Usually, they are the smaller, lower-extremity joints. Podagra, or inflammation of the first metatarsophalangeal joint, is the initial joint manifestation involved in about half of all cases. However, podagra may also be observed in patients with pseudogout, reactive arthritis, gonococcal arthritis, psoriatic arthritis, and sarcoidosis. The attacks usually begin abruptly, and they can reach maximum intensity in 6 - 12 hours. The joints are red, hot, and extremely tender. Untreated, the characteristics of gout change over time. The attacks become more polyarticular. Although more joints may become involved, inflammation in a given joint may become less intense. Attacks occur more frequently and last longer. Eventually, patients may develop a chronic polyarticular arthritis, which can be symmetrical and resemble rheumatoid arthritis. Tophi, which are collections of uric acid crystals in the soft tissues, occur frequently in untreated patients. They can be found in multiple locations, including the fingers, toes, and olecranon bursae; they can also be found along the olecranon, where they may appear to be rheumatoid nodules. Tophi tend to develop after 10 years in untreated patients who develop chronic gouty arthritis. Acute flares of gout can occur in situations that lead to increased levels of serum uric acid, such as the use of alcohol, overindulgence of certain foods rich in protein, trauma, hemorrhage, or the use of medications that elevate levels of uric acid. Allopurinol is used in the prevention of future attacks. It is not used to treat acute flare-ups. Patients with frequent gout attacks may begin prophylactic treatment to prevent further episodes. Allopurinol is used in such cases. Allopurinol blocks xanthine oxidase, thereby reducing the production of uric acid. Therefore, it should be used in patients who overproduce uric acid. It is the most effective agent to lower serum uric acid levels. The dosage is 200 - 300 mg daily. The side effects include rash, gastrointestinal symptoms, headache, urticaria, interstitial nephritis, and (rarely) hypersensitivity syndrome. It was originally developed as a chemotherapeutic agent. However, alcohol can interfere with its effectiveness. NSAIDs are the drugs of choice in most patients without underlying health problems. They have an anti-inflammatory effect that works by inhibiting cyclooxygenase, which acts to produce leukotrienes from arachidonic acid. Indomethacin is the drug of choice and is usually given at a dose of 25 - 50 mg 4 times a day, unless the patient is elderly. However, other NSAIDs, such as ibuprofen, naproxen, sulindac, and ketoprofen, may be used; aspirin cannot be used because it can elevate uric acid levels. Some of the side effects of NSAID therapy include gastropathy, nephropathy, and liver dysfunction. It can also cause fluid overload in patients with congestive heart failure. Therefore, their use should be limited in these patients. Colchicine is an antimitotic drug that is often used in patients who cannot take NSAIDs or corticosteroids. However, due to its side effects (such as nausea, vomiting, and diarrhea and following intravenous administration--bone marrow suppression, renal failure, and death), it is not used as a first-line treatment. The normal dosage is 0.5 - 0.6mg orally every hour, until relief or side effects occur, or until a maximum dosage of 6 mg. Sulfinpyrazone, a uricosuric agent, is an alternative agent to aid in the prevention of attacks. However, it is not used as frequently due to the fact that it can cause bone marrow suppression. Antibiotics are used when symptoms are caused by an infectious agent. This is not the case in a patient who has gout.

male presents to the emergency department complaining of pain to the right shoulder region while playing basketball. He states that his arm was pulled back and rotated while he was moving forward, and then felt a popping sensation in the shoulder. Since then he has not been able to move the shoulder at all due to pain and immobility. It is suspected that he has a dislocation. Given this scenario, what would be the most likely type? What type of shoulder dislocation did he most likely experience? A Anterior B Inferior C Multidirectional D Posterior E Superior

Anterior A All of the various types of dislocations mentioned above are possible, but anterior dislocations are by far the most common (>95%) and they are the most common of all joint dislocations. Most occur as a result of a fall or other traumatic event and they may become recurrent. The shoulder is most susceptible to an anterior dislocation when it is abducted and externally rotated. The shoulder joint is considered a very mobile joint, but this also renders it very susceptible to injury

A 52-year-old male has a 200-pound file cabinet fall on his right leg. He comes to the emergency department complaining of pain and swelling to the right leg. He is also complaining of parasthesias to the leg also. Based on these findings on history, what part of the leg would be the most likely site of compartment syndrome? A Anterior compartment of the leg B Deep posterior compartment of the leg C Dorsal compartment of the forearm D Lateral compartment of the leg E Superficial posterior compartment of the leg

Anterior Compartment of the leg Anterior compartment syndrome is most commonly found in the anterior compartment of the leg, with the volar compartment of the forearm also a common location. The mechanism is generally an acute crushing trauma to the affected area that causes an increase in pressure within the compartment that inhibits venous outflow and a decrease in arterial blood flow. This adversely affects tissue perfusion and ischemia of the involved tissues can occur. The symptoms can also occur with chronic exertion or when there is a dramatic increase in the amount of exercise being performed. Anterior compartment syndrome of the leg is likely most common due to its vulnerable location and susceptibility to injury in athletics and motor vehicle accidents. The rate of occurrence of acute anterior compartment syndrome is also likely due to the fact that the anterior compartment lies adjacent to the tibia and the tibia is the most frequently fractured long bone. Compartment syndrome could occur in any compartment of the upper or lower extremities if the necessary circumstances were in place (either acute trauma to the area or excessive use of the muscles that are associated with each of the compartments).

A 22-year-old male hurts his right knee while playing football on artificial turf. He states that he planted his foot and went to turn, but his leg didn't turn with his body. He instantly felt a popping sensation in the knee. A few hours later he develops an effusion. Based on the history of the injury, which knee structure was likely injured? A Anterior cruciate ligament (ACL) B Lateral collateral ligament (LCL) C Medial collateral ligament (MCL) D Posterior cruciate ligament (PCL) E Quadriceps tendon

Anterior Cruciate Ligament (ACL) Non-impact rotational or hyperextension forces are the most common mechanisms for sustaining a tear of the ACL. One third of patients report hearing an audible popping sound as their ACL tear occurred. Because the ACL is a vascular structure, when it tears a rapid bloody effusion (hemarthrosis) usually develops which effects mobility of the joint. Lateral collateral ligaments are the least likely to be injured as the type of force necessary to cause injury would be a varus stress which is unlikely to occur in typical circumstances. Medial collateral ligament injuries are fairly common and produced by a valgus force that stresses the ligament. This can occur in many sporting events including those in which another competitor might fall on or dive into the lateral aspect of the knee. Trauma to a knee can result in tears of both the ACL and MCL in certain situations. A tear of the quadriceps tendon usually occurs when a person falls on a knee that is partially flexed. As the quadriceps muscle contract to prevent excessive flexion, the force and momentum of the fall may overwhelm the knee extension mechanism and cause the rupture. No such mechanism occurred in our scenario. Posterior cruciate injuries occur when the tibia is driven posterior in relation to the femur as may happen when a car dashboard is driven into the tibias during a major front impact collision. A powerful hyperextension force can result in both ACL and PCL tears (usually in that order). PCL tears are much more uncommon than ACL tears and don't generally occur with basic rotational forces as described in our patient scenario.

A 39-year-old woman presents after getting hurt at her karate class the night before. She had been doing kick-jumps and landed on the outside of her left plantar flexed and inverted foot with all of her weight. There was a popping sound, and sharp pain set in immediately. The instructor had ice put on the ankle and elevated it right away. Before leaving the facility, a compression wrap was put on. Treatment was sought about 14 hours later because of pain and swelling. On examination, swelling in the area in front of the lateral ankle is visible. The anterior drawer test shows a difference of 5mm between injured and non-injured ankle. The talar tilt test reveals no instability, and weight-bearing ability is only slightly restricted. What ligament is most likely affected? Answer Choices 1 Syndesmosis 2 Delta ligament 3 Anterior talofibular ligament 4 Posterior talofibular ligament 5 Calcaneofibular ligament

Anterior talofibular ligament Considering the mechanism of injury, the most likely affected ligament is the anterior talofibular ligament. It is usually the result of landing on a plantar flexed and inverted foot when jumping, stepping in a hole, or walking on rough ground. The anterior talofibular ligament is the weakest one within the lateral ankle complex. It tears first, followed by the calcaneofibular and posterior talofibular ligament. The anterior drawer test (image 2) measures the anterior translation of the talus and determines the integrity of the anterior talofibular ligament. Since translation can be up to 9 mm, comparison to the non-injured ankle is very important. If the foot is dorsiflexed and inverted when injured, the calcaneofibular ligament is hurt. In this case, the talar tilt test (image 3) shows instability. External rotation force on a dorsiflexed foot results in injury of the syndesmosis. The delta ligament tears in eversion and external rotation injuries. Anterior drawer test to measure the integrity of the anterior talofibular ligament is as follows: steadying the lower leg with one hand while the other hand grasps the heel and pulls the foot forward. Physiologically, the movement is between 2 and 9 mm. Therefore, it is important to compare the injured with the non-injured ankle. The test is considered positive when the difference is more than 4 mm. Talar tilt test to measure the integrity of the calcaneofibular ligament is as follows: steadying the lower leg with one hand the other hand grasps the heel and inverts the foot. Physiologically, the movement ranges from 5° to 23°, and it is also essential to compare injured and non-injured ankle. The test is considered positive when the difference is more than 6°.

A 19-year-old man presents with pain and deformity of his right dominant shoulder after a sudden jerking movement to the same from a wrestling competitor approximately 1 hour ago. He states he felt a clunking sensation when it happened. He was unable to continue wrestling and has pain with movement of the right shoulder. What diagnostic studies should be performed? Answer Choices 1 MRI stat 2 CT of the shoulder 3 Anterior/Posterior (AP) and internally rotated humeral view radiographs 4 Anterior/Posterior (AP) and axillary or transscapular lateral radiographs 5 Internal and external rotated views of the humeral head

Anterior/Posterior (AP) and axillary or transscapular lateral radiographs Explanation AP and axillary lateral or transscapular lateral (Y-scapula) views should be obtained. It is imperative that a lateral view of the shoulder joint be obtained to assess the position of the humeral head. Posterior dislocations can be easily missed if the lateral view is not obtained.

A 24-year-old pregnant woman presents with fever, joint pain, loss of hair, and a rash over the bridge of her nose and cheeks. She also suffers bouts of depression. What is the most valuable laboratory test for the diagnosis of her condition? Answer Choices 1 The fluorescent antinuclear antibody test 2 Detection of the LE cells 3 Measurement of C3 complement component 4 Antibodies to double-stranded DNA 5 The lupus anticoagulant

Antibodies to double-stranded DNA Explanation The correct response is antibodies to double-stranded DNA. The patient's presentation is characteristic of systemic lupus erythematosus. It is a non-organ specific, autoimmune disease. It is 9 times more common in women, especially during pregnancy, and almost 10 times more common in West Indian blacks. It is a disease of relapses and remissions. Musculoskeletal system involvement occurs in the vast majority of cases, commonly as arthralgia, myalgia, and myositis. Photosensitive butterfly rashes, scarring alopecia, livedo reticularis, oral ulceration, and conjunctivitis are the common cutaneous manifestations. Other common features include: fever, splenomegaly, psychosis, fits, cranial nerve involvement, and meningitis. Renal symptoms include proteinuria, edema, hypertension, and renal failure. Pleurisy and pleural effusion as well as pericardial effusions may occur together with endocarditis and cutaneous vasculitis. 80% have positive antinuclear antibody, but high titer of anti-double stranded DNA is almost exclusive to SLE. 40% have a rheumatoid factor. 11% have a false positive for syphilis serology. Complement is low. LE cells test is less sensitive, very time consuming, and hardly more specific. They are no longer looked for. Lupus anticoagulant is an anticardiolipin antibody, which may also give false positive biological tests for syphilis. High titers are associated with thrombocytopenia, thrombolytic manifestations, and in some cases, recurrent abortion. Complement levels are non-specific. The system is activated either as part of an antibody-mediated immune response (classical pathway) or by bacterial or other chemical stimuli (alternative pathway). Both lead to lysis of the cells.

A 40-year-old African-American woman presents with diffuse headache and joint pain. The headache started few days ago; it is dull and becoming progressively worse. Joint pain is localized in fingers, starts in the morning, and improves during the day; it returns when she tires. 2 months ago, she was treated in the ED because of several weeks of lasting fatigue, low-grade fevers, joint pain, hair loss, and oral ulcers. Her laboratory tests were normal, except for positive VDRL and antinuclear antibody tests results. Urine and blood cultures were negative for evidence of infection, and her chest X-ray was normal. Because she felt better after 10-day tapering course of prednisone, she did not appear to the scheduled control. Today, your examination reveals an ill-appearing woman in distress. Her temperature is 39 C. Her fingers are swollen and red, and she has malar rash and oral ulcers. Question What additional test should you order as the most specific for her condition? Answer Choices 1 Synovial fluid analysis 2 Urine protein/creatinine ratio 3 Antibodies to the Sm antigen 4 Cerebrospinal fluid analysis 5 Complement system test

Antibodies to the Sm antigen Explanation The diagnosis of SLE is based on clinical features and the presence of autoantibodies. Current criteria for the diagnosis are 4 or more of the following: malar rash, discoid rash, photosensitivity, oral ulcers, arthritis, serositis, renal disorder, neurologic disorder, immunologic disorder, and antinuclear antibodies ("1997 Update of the 1982 American College of Rheumatology Revised Criteria for Classification of Systemic Lupus Erythematosus"). This patient meets the criteria. She has a history of positive ANA and alopecia; there is the presence of a neuropsychiatric disorder, oral ulcers, arthritis, and fever, with infection ruled out. Antibodies to the Sm antigen in the presence of characteristic clinical picture are the most specific for SLE. Noninflammatory synovial fluid will contribute to the diagnosis of SLE only in the presence of compatible clinical manifestations. Synovial fluid analysis is not necessary in this patient. Urine protein/creatinine ratio demonstrating kidney disease favors the diagnosis of SLE in the presence of compatible clinical manifestations. It is not necessary in this patient. Cerebrospinal fluid analysis is not indicated in a case of a headache without focal neurological signs and a clear clinical picture of SLE. A complement test may be used to monitor the course and treatment efficacy in patients with autoimmune disorders. Patients with active SLE may have lower-than-normal levels of the complement proteins C3 and C4, particularly in the presence of lupus nephritis. Complement system test is not specific for SLE.

A 32-year-old woman presents with a 2-day history of sharp pleuritic-type chest pain. She indicates that the pain worsens when she lies down and feels better when she sits upright. She also admits to having polyarticular arthritis involving the small joints of her hands. Her obstetric history is positive for 2 1st trimester spontaneous abortions. You order a CXR, which reveals pericarditis. Question What laboratory test could you order to help assist in your diagnosis? Answer Choices 1 Complete blood count (CBC) 2 Rheumatoid factor (RF) 3 Antinuclear antibodies (ANA) 4 Parathyroid hormone (PTH) 5 Antineutrophil cytoplasmic antibody (ANCA)

Antinuclear antibodies (ANA) Explanation The clinical picture is suggestive of systemic lupus erythematosus (SLE). Antinuclear antibodies are positive in 95 - 100% of patients with SLE. Patients with SLE can present with an abnormal CBC (anemia, leucopenia, and thrombocytopenia), but it is not diagnostic in SLE. Rheumatoid factor can be present in 20% of patient with SLE, but it is not diagnostic for SLE. The parathyroid hormone would be normal in a patient with SLE. Antineutrophil cytoplasmic antibody is only 0 - 1% positive in patients with SLE.

A 42-year-old African-American woman presents with acute deterioration of her mental state; she is hallucinating and talking about people hiding under her bed. She cannot sleep, and she refuses to eat. Several months ago, her family practitioner told her that she might have lupus erythematosus; however, she refused further investigations. On examination, she appears agitated and is disorientated to place and time; she has impaired memory, and she confirms the presence of auditory hallucinations. Question What test would best confirm the diagnosis of cerebral systemic lupus erythematosus? Answer Choices 1 Antinuclear antibodies in blood 2 Oligoclonal bands in cerebrospinal fluid 3 Antineuronal antibodies in blood 4 Protein levels in cerebrospinal fluid 5 Complement activity in serum

Antinuclear antibodies in blood Antinuclear antibody (ANA) testing is used for serologic testing for SLE. There are 2 major types of ANA: autoantibodies to DNA and histones and autoantibodies to extractable nuclear antigens. The first group concerns antibodies against single and double-stranded DNA (dsDNA); significant levels of anti-dsDNA antibodies are considered confirmatory in the diagnosis of SLE (anti-histone antibodies are considered confirmatory in the diagnosis of drug-induced SLE). Most instances of the second group (autoantibodies to extractable nuclear antigens) are specific to a disease, e.g., autoantibody to Smith antigen is considered to be specific for SLE. Other examples include: autoantibodies to ribonucleoproteins, SSA/Ro, or SSB/La, Scl-70, Jo-1, PM1, etc. Oligoclonal bands are immunoglobulins seen in serum, plasma, or cerebrospinal fluid (CSF). The presence of oligoclonal bands in CSF with their absence in serum shows that immunoglobulins are produced within the central nervous system. Since 80-90% patients with multiple sclerosis have oligoclonal bands, they are considered an indicator in the diagnosis of multiple sclerosis. They are not confirmatory for SLE. Antineuronal antibodies are commonly associated with paraneoplastic neurological diseases (Anti-Hu, Yo, Ri amphiphysin, Tr, CV2, and Ta antibodies). Some of these antibodies are specific for certain types of cancer or neurological syndromes. They are not confirmatory for SLE. Protein levels in CSF are not specific; they may be elevated in several conditions (e.g., diabetes mellitus, brain tumor, brain abscess, meningitis, multiples sclerosis, hemorrhage, Guillain-Barre ayndrome); they may be normal in viral infections, or they may be low (recent lumbar puncture, chronic CSF leakage, water intoxication). Complement system plays a role in many diseases with an immune component (asthma, glomerulonephritis, multiple sclerosis, inflammatory bowel disease, etc); therefore, its levels are not specific for SLE.

A 17-year-old boy presents following an injury to his right dominant upper arm while playing football a half hour ago. While attempting to throw a pass, an opposing player grabbed his arm and tackled him. The opposing player landed directly on the thrower's flexed elbow, while driving the thrower's shoulder into the ground.The injured player's mother heard a "cracking sound" in the stands. He had immediate pain and exhibited a deformity in the distal 1/3rd of his right arm. Besides the inspection findings of edema, ecchymosis, and deformity, what specific physical examination technique should be performed? Answer Choices 1 Sensory testing of the axillary nerve 2 Assessment of wrist extension and dorsal hand sensation 3 Compartment pressure testing 4 Palpation for lymphadenopathy 5 Full passive range of motion testing of both the arm and elbow

Assessment of wrist extension and dorsal hand sensation Since radial nerve injuries can be associated with this fracture, checking the neurological status of the radial nerve by asking the patient to extend his wrist (which should not be painful at the fracture site) and checking the sensory distribution on the dorsal hand is imperative. The axillary nerve is proximal to the fracture and is unlikely to be affected. Compartment syndrome, although possible, is a later finding and not one of the first concerns during the physical exam. Lymphadenopathy is not an acute finding due to a fracture. Even gentle minimal passive range of motion of the elbow and arm is painful, so full passive range of motion should NOT be attempted.

A 49-year-old man presents with pain in his left lower extremity. He is well known to your practice. Upon further questioning, the patient describes doing construction work 3 months prior and jumping onto his feet from an elevated height of about 5 ft. Since he did so, he has noted increasing issues of left-sided hip and knee pain. He describes the pain as radiating into the left groin and front middle thigh area. The pain is relieved with sitting, and it is aggravated by walking and climbing up stairs. The patient denies any paresthesias, numbness, bowel/bladder dysfunction, fever, night sweats or chills. Pertinent medical history includes alcoholism for the past 15 years and 3 episodes of acute pancreatitis requiring hospitalization in the past 4 years. Question What is the most likely diagnosis? Answer Choices 1 Multiple myeloma 2 Osteoporosis 3 Avascular necrosis 4 Osteoarthritis 5 Rheumatoid arthritis

Avascular necrosis, which is sometimes referred to as osteonecrosis, is osseous cell death resulting from vascular compromise. Common sites that are affected include the proximal or distal femoral head, ankle, shoulder, and/or elbow. Common causes include (but are not limited to) corticosteroid use, alcoholism, trauma, systemic lupus erythematosus, pancreatitis, gout, and sickle cell disease. The patient shows several risk factors for development of avascular necrosis, keeping the condition very high on the differential diagnosis. Multiple myeloma typically initially presents as spine, rib, or proximal long bone pain; it is seen more commonly in older adults, with the median age of presentation being 65 years. Many of these patients will present with signs and symptoms relating to the pathology, including anemia, infections, kidney failure, spinal cord compression, and hyper-viscosity syndrome. Osteoporosis is not common in this patient's population group; therefore, it is much lower on the differential diagnosis. Its occurrence is higher in Caucasian women who have other risk factors, such as tobacco or alcohol abuse, excess/deficiency in hormones, malignancy, and some genetic disorders. Although the patient may have some signs of osteoarthritis on future imaging studies, it is unlikely that it is the main cause of his current symptoms. Rheumatoid arthritis has an insidious onset; it is characterized by morning stiffness that symmetrically affects the small joints of the hands and feet.

Shoulder dislocation commonly involves injury to which nerve? A Axillary B Median C Peroneal D Radial E Ulnar

Axillary The axillary nerve is in close proximity to the glenohumeral joint, thus making it vulnerable to injury during a shoulder dislocation. The median, radial, ulnar nerves are more distal in the upper extremity and and the peroneal nerve is in the leg.

A 32-year-old man is working on his farm when a wild horse suddenly jerks the bridle, forcing the man's right arm over his head and backwards. There is a 'clunking' sound, and the man is in immediate pain and unable to move his arm. There is a deformity of the shoulder with a depressed area (dimple) noted in the anterior shoulder. Question Which nerve MUST be carefully assessed in this patient before AND after it is fixed because of the possible injury to it? Answer Choices 1 Musculocutaneous 2 Median 3 Axillary 4 Radial 5 Ulnar

Axillary nerve Explanation With a glenohumeral dislocation, as in this man, a risk of neurapraxia of the axillary nerve is possible. Neurovascular function, particularly that of the axillary nerve, must be carefully assessed before and after reduction of the dislocation. Anterior-inferior displacement of the humeral head into the quadrangular space has the potential of injury to the axillary nerve. The musculocutaneous nerve supplies the coracobrachialis muscle, leaves the axilla by piercing that muscle, and is not exposed to the force of dislocation. The median nerve arises from the lateral and medial cords of the brachial plexus, passes downward on the lateral side of the axillary artery, and also is not exposed to the force of dislocation. The radial nerve, the longest branch of the brachial plexus, lies behind the axillary artery and also is out of harm's way. The ulnar nerve arises from the medial cord of the brachial plexus, descends in the interval between the axillary artery and vein, and is also not subject to the force of dislocation.

A 28-year-old man presents first thing Monday morning due to a left ankle sprain. The patient describes walking in the woods the day before while wearing flip flops and inadvertently stepping into a hidden hole. The patient immediately noticed extreme pain with weight bearing on the left ankle, making it extremely difficult for him to walk out of the woods. Ambulation is still exceptionally difficult at this time. He describes it as an 8-9/10 on a 1-10 pain scale. The patient admits to having ankle sprains before, but this one is much more severe and debilitating. He has also noted significant swelling and exquisite tenderness to the touch. He has treated his ankle with elevation, ice for 20 minutes at a time, and ibuprofen 200 mg every 8 hours. Physical examination reveals a moderate degree of ecchymosis of the left ankle with substantial mechanical instability and moderate restriction of range of motion. Question Based on the history and physical examination findings, what would be the most appropriate clinical intervention at this time? Answer Choices 1 Bracing of the ankle 2 Casting of the ankle 3 No weight bearing activities 4 No modified activities 5 Observation only

Bracing of the ankle Explanation The scenario above is describing the Grade II ankle sprain. This injury typically involves an incomplete tear of a ligament. Patients will experience moderate pain, swelling, tenderness, and ecchymosis. There will be mild to moderate joint instability during exam and some restriction of the range of motion, as well as loss of function. Ambulation and weight bearing are painful. Grade I ankle sprain results from only mild stretching of a ligament with potentially microscopic tears. Patients will only present with mild swelling and tenderness; there will not be evidence of joint instability, and the patient will be able to bear weight or ambulate fairly easily. Grade III ankle sprain involves a complete tear of a ligament. There will be severe pain, swelling, tenderness, and ecchymosis. Significant instability will be seen on exam, as well as loss of function, inability to bear weight, or ambulate whatsoever. Immediate intervention of an ankle sprain should follow the MICE mnemonic: modified activities, ice, compression, and elevation, which our patient has been somewhat following. Subsequent treatment commonly involves protected weight bearing with crutches and the use of an ankle stabilizer brace. This is considered especially in those patient suspected of having either a Grade II or III ankle injuries. Early motion is essential, so the response of no weight bearing is incorrect. Patients are often encouraged to perform appropriate home and physical therapy exercises. A cast for this injury would also not be an appropriate choice. It may be considered for a high ankle sprain, which is not the case in the above scenario. Observation only is also not the correct answer.

A 12-year-old boy presents with left shoulder pain and deformity after a fall from his skateboard while attempting a half-pipe maneuver an hour ago. He heard a "cracking" sound and was unable to continue skateboarding. He has pain with movement of the arm in any direction. On exam, you note tenderness over the superior central shoulder with a palpable deformity and "tenting" of the skin. What is his most likely diagnosis? Answer Choices 1 Acromioclavicular (AC) separation 2 Glenohumeral dislocation 3 Rotator cuff tear 4 Clavicle fracture 5 Frozen shoulder

Clavicle fracture Explanation With a superior central deformity and tenderness, along with tenting of the skin, the most likely diagnosis is clavicle fracture. AC separation would have deformity laterally on the shoulder. Glenohumeral dislocation would have shoulder contour deformity. Rotator cuff tear does not have acute deformity. Frozen shoulder is a slow, progressive chronic problem and not an acute injury.

Out of all cervical vertebrae, which two are responsible for the greatest amount of rotation? A C1 & C2 B C2 & C3 C C3 & C4 D C4 & C5 E C5 & C6

C1 and C2 Approximately 50% of cervical rotation takes place between the C1 (atlas) and C2 (axis) vertebrae. These first two cervical vertebrae have a different shape from the other cervical vertebrae that allow for this greater range of motion. The remaining 50 % of cervical rotation is split fairly evenly between the remaining vertebrae. Approximately 50 % of flexion and extension occurs between the occiput at the base of the skull and C1 with the remaining 50% distributed fairly evenly between the remaining vertebrae with a slightly higher percentage occurring at the C5 & C6 level

A patient presents with sensory loss in the area of the thumb and forefinger. Which nerve root or roots of the spinal cord is suspected? Answer Choices 1 C4 2 C5 3 C6 4 C7 5 C6 thru T1

C6 thru T1 The median nerve arises from the medial and lateral cords of the brachial plexus and includes the spinal nerve roots of C6, thru and including C7, C8 and T1.

Radiculopathy due to nerve root compression occurs most commonly at which nerve root within the brachial plexus? A C5 B C6 C C7 D C8 E T1

C7 The C7 nerve root is affected the most often (approximately 45-60%). This radiculopathy can result from foraminal encroachment of the spinal nerve, cervical disk herniation, tumor, and multiple sclerosis. C7 radiculopathy can present with weakness in the triceps, which cause elbow extension, and finger flexion and extension. C6 is another common site of radiculopathy. C6 radiculopathy can present with weakness in the biceps, brachioradialis, and wrist extensor muscles. Cervical radiculopathy at the C5, C8, and T1 are less common, but still possible. C5 radiculopathy can present with deltoid and biceps muscle weakness. C8 radiculopathy can present with finger flexor weakness and T1 radiculopathy with finger abduction weakness.

An 84-year-old female is taking prednisone for the past year, secondary to a diagnosis of biopsy-confirmed giant cell arteritis. Her dose was tapered to 20 mg/day, but her symptoms returned several months ago and her dose was increased. In addition she is taking hydrochlorothiazide (HCTZ) for hypertension, but denies any other medications or medical problems. What treatment should you recommend to this patient at this time? A glucophage B simvastatin C calcium and vitamin D D fosamax E naprosyn

Calcium and Vit d The correct answer is (C). This patient requires chronic prednisone treatment. Common complications of chronic glucocorticoid treatment include osteoporosis and diabetes. At this point with this patient there is no information stating that she has diabetes or osteoporosis. Before initiation of treatment with glucophage, the patient should have a diagnosis of diabetes mellitus. Before initiation of fosamax the patient should have a bone density scan to look for osteopenia/osteoporosis. In the meantime, calcium supplementation with vitamin D is warranted for prevention of osteoporosis. Simvastatin is not indicated, and naprosyn may increase risk of gastrointestinal bleeding with chronic prednisone.

A 64-year-old female presents for an annual examination. She is 5'0" and weighs 92 pounds; compared to her examination 3 years ago she has lost an inch in height. After performing a dual-energy x-ray absorptiometry (DXA) scan the diagnosis of osteoporosis is confirmed. Which of the following supplements should be recommended? A Calcium carbonate 1,000 mg and vitamin D 800 international units daily B Calcium carbonate 500 mg and vitamin D 2,500 international units daily C Calcium carbonate 2,000 mg and vitamin D 500 international units daily D Calcium carbonate 800 mg and vitamin D 100 international units daily

Calcium carbonate 1,000 mg and vitamin D 800 international units daily Calcium supplementation provides beneficial effects on bone mass throughout postmenopausal life and may reduce fracture rates up to 50%. Postmenopausal women receiving supplemental calcium over a 3-year period in a placebo-controlled, randomized clinical trial had stable total body calcium and BMD in the lumbar spine, femoral neck, and trochanter compared with the placebo group. Recommendations for calcium carbonate are 1,000-1,500 mg once daily orally. Vitamin D increases calcium absorption in the gastrointestinal tract, making calcium more available for reabsorption and circulation. Recommendations for Vitamin D are 800-2,000 international units once daily orally. Calcium Carbonate less than 1,000 mg per day is not sufficient (B and D). Vitamin D levels less than 800 IU per day is not sufficient (C and D).

A 21-year-old man presents with repetitive contractions of distal muscles of the hands and feet. On examination, he is found to have carpal spasms and pedal spasms. Physiologically, this condition is caused by the increased intracellular concentration of what? Answer Choices 1 Calcium ions 2 Sodium ions 3 Potassium ions 4 ATP 5 Chloride ions

Calcium ions The basis of muscular contraction is the release of calcium ions from the sarcoplasmic reticulum, which binds to Troponin C of the light chain, leading to increased cross bridges between the light chain and the heavy chain in the sarcomeres, resulting in the contraction of the muscle fibers. In conditions of increased excitability of peripheral nerves, more calcium ions are released from the sarcoplasmic reticulum than can be re-accumulated in normal circumstances. This leads to increased intracellular accumulation of calcium ions, leading to more cross bridging and resulting in increased and repetitive muscular contraction. This condition is called tetany. Intracellular sodium and potassium ions do not change appreciably during action potential phase. Therefore, their intracellular concentration remains more or less normal. ATP is the energy-releasing substrate known as adenosine triphosphate. It releases energy, which is used for muscular contraction. In sustained muscular contraction, a lot of energy is used and the intracellular concentration of ATP decreases. Chloride ions do not have a major role to play in the formation of action potential; therefore, their concentration is not appreciably changed from the normal.

A 32-year-old man presents due to low back pain. He reports a 3-year history of chronic low back pain that progressively worsened. He notes a lot of stiffness with any movements that involve his low back. He is especially stiff upon morning awakening and now has to ask his wife to help him get out of bed. He denies a history of trauma. He works in retail, with some bending and light lifting duties. He has taken acetaminophen repeatedly, even exceeding recommended doses, with no relief. He knows his paternal grandfather had some type of serious arthritis in his back, but he does not know the diagnosis. Other than his chronic back pain, the patient reports that he is in good health, with no chronic conditions. He takes no regular medications, except for the analgesics mentioned above. He has no allergies and has never had surgery. On physical exam, some limited range of motion is observed in his back and hips, and the patient reports discomfort with the motions. Otherwise, his exam is normal, with no visible edema, muscle wasting, or bony abnormality. You obtain a plain radiograph, and the report indicates a classic "bamboo spine" appearance and some evidence of sacroiliitis. His sedimentation rate and c-reactive protein levels are elevated. Question What medication would be an appropriate first-line treatment for this patient's condition? Answer Choices 1 Acetylcysteine (Mucomyst) 2 Allopurinol (Aloprim) 3 Celecoxib (Celebrex) 4 Cyclobenzaprine (Flexeril) 5 Milnacipran (Savella)

Celecoxib (Celebrex) This patient's presentation is consistent with a diagnosis of ankylosing spondylitis (AS), a progressive inflammatory arthritis that primarily affects the axial skeleton. NSAIDs are the primary first-line treatment. An NSAID, such as celecoxib (Celebrex), would be reasonable. The cyclooxygenase-2 (COX-2) inhibitors, such as celecoxib, have not been necessarily shown to have better efficacy, but are suggested to have fewer gastrointestinal side effects than traditional NSAIDs. Regular use of NSAIDs is recommended. Other potential treatments include medications such as methotrexate, sulfasalazine, and newer "biologics" like infliximab, etanercept, and adalimumab. Acetylcysteine (Mucomyst) is a medication used for acetaminophen overdose. The patient's current symptoms do not suggest acute acetaminophen toxicity. Allopurinol (Aloprim) is a medication used primarily for gout. While gout can cause joint pain, this patient's presentation is not consistent with gout. Cyclobenzaprine (Flexeril) is an older muscle relaxant medication. Although the patient reports "stiffness," his symptoms are due to joint damage and not muscle spasm. This medication would not help his AS. Milnacipran (Savella) is a medication approved for fibromyalgia. While chronic pain are found in both AS and fibromyalgia, none of this patient's diagnostic studies are consistent with fibromyalgia.

A 55-year-old woman presents with a 3-week history of dull pain in the left femur. She does not remember any trauma to her leg. After seeing an abnormality on the X-ray, you obtain a bone biopsy and see poorly-to-well-formed cartilage-like cells with a bizarre appearance. What is the most likely diagnosis? Answer Choices 1 Osteomalacia 2 Old fracture 3 Paget's disease 4 Bone cyst 5 Chondrosarcoma

Chondrosarcoma Explanation Since there are cartilage-like cells in the biopsy, it is very likely a chondromatous tumor, the widely spread differentiation points to a sarcoma. It predominantly occurs in the pelvis, femur, and shoulder girdle in middle-aged to older adults. It may be primary, arising from cartilage cells, or secondary to a preexisting benign lesion. In osteomalacia (failure to calcify the bone matrix), there would be a decreased amount of well-calcified bone. An old fracture would show unorganized meshwork of woven bone (callus). In Paget's disease, also known as osteitis deformans, there would be dense, well-calcified, erratically shaped bony spicules with abnormal grouped cement lines, which form a mosaic pattern. A bone cyst would show a big, solitary, cystic cavity.

Which bone is the most susceptible and most often fractured at birth? A Calcaneus B Clavicle C Femur D Humerus E Patella

Clavicle B The clavicle is the most common bone broken during childbirth. It often is associated with shoulder dystocia, but clavicular fractures can occur in uncomplicated pregnancies. They are usually of the greenstick variety and heal without complications. Calcaneal and patellar fractures are highly unlikely to occur since they are not long bones which are much more vulnerable to fracture. Fractures of the humerus and femur are possible during childbirth, but generally only in traumatic births. Humerus and femur fractures are much less common than clavicular fractures

A 29-year-old woman is in her taekwondo class and misjudges what her opponent is going to do. She falls while her arm is abducted, and she injures it. On physical examination, there is tenderness, swelling, and visible deformity in her upper arm. X-rays of her arm are done and show a fractured humerus. After reviewing the X-ray, the doctor comes back into the examining room and tells her that her bone has broken into several pieces. What type of fracture is this? Answer Choices 1 Comminuted 2 Depressed 3 Greenstick 4 Oblique fracture 5 Simple

Comminuted Explanation A comminuted fracture is when the bone has broken into many (more than 2) pieces. A simple fracture is a complete break in the bone that does not break the skin. This is also called a closed fracture. A depressed fracture is when there is inward displacement of the bone. An oblique fracture is a fracture that runs diagonally against the long axis of the bone. A greenstick fracture is an incomplete break in the bone with bending of the bone. It is named as if you were to try to break a live green twig, but only partially break it.

A young mother brings her 5-week-old female infant for a routine check up. Pregnancy and delivery had been uneventful. The infant is doing fine, but the mother tells you that she has problems spreading the girl's left leg when she is changing her diaper. You suspect a dislocated hip. What should be looked for in order to determine whether or not the infant has a dislocated left hip? Answer Choices 1 Dislocation on X-ray 2 Shortening of the affected femur when supine with knees bent 3 An increase in skin folds on the unaffected side 4 Crying/discomfort with Ortolani's test applied on the left 5 Presence of Trendelenburg's sign on the left

Congenital dislocation of the hip is more common in female infants, infants with a positive family history, and breech in utero positioning. Gravity will cause the femur to drop toward the bed, which makes the affected femur appear shorter when the baby is held in a supine position. Hip X-rays are very difficult to interpret, especially at an early age. Therefore, they are not the method of choice for determining whether or not an infant has a dislocated hip. Hip ultrasounds are very accurate in early diagnosis. There would also be an increase in skin fold on the affected side. Since the major sign of hip dislocation or subluxation is the Ortolani's test, the inability to completely abduct the thigh to the surface of the examining table when the hip and knee are flexed will be present on the left side. This is not typically a painful test and should not produce discomfort in the patient. The Trendelenburg's sign is positive in persons with congenital hip dislocation and pelvis abnormality. When the person stands on the affected leg, the opposite (normal) gluteal fold falls. This test is almost impossible to do on an infant.

A healthy 37-year-old woman sustained a hairline fracture of the left tibia. Her leg is put in a cast and immobilized. 6 weeks later, the cast is removed from her leg. A repeat X-ray shows healing of the fracture line. What best characterizes her gastrocnemius muscle at this time? Answer Choices 1 Muscle undergoes fibrosis 2 Increased fat deposition 3 Decrease in number of myofibrils 4 Increase in calcium levels 5 Increase in number of satellite cells

Decrease in number of myofibrils Explanation Due to long term restriction of activity, the gastrocnemius muscle is expected to undergo disuse atrophy. The muscle fibers decrease in size and there is a decrease in number of myofibrils. The muscle fiber type (fast or slow fiber) is determined by the type of innervation. Whether training and detraining can interchange the muscle fiber types is controversial, although there is more support to the idea of fast fibers converting to slow fibers with detraining. Satellite cells are normally quiescent cells in skeletal muscle that become active in regeneration and hypertrophy.

Foot drop and a high-stepping gate are the result of damage to what nerve? Answer Choices 1 Deep peroneal nerve 2 Superficial peroneal nerve 3 Tibial nerve 4 Sciatic nerve 5 Obturator nerve

Deep peroneal nerve Explanation The anterior muscles of the lower leg are involved in dorsiflexion of the ankle. The nerve that controls these muscles (deep peroneal) will dorsiflex the foot when walking. If the deep peroneal nerve is damaged, the foot will not dorsiflex and the patient must lift his/her foot up high so that the toes clear the ground when walking. The superficial peroneal nerve innervates the lateral peroneals (evert the foot). The tibial nerve innervates posterior muscles of the leg. The sciatic nerve, while giving rise to the superficial and deep peroneal nerves, also innervates the hip (extension), resulting in excessive hip flexion for the foot to clear the ground. The obturator nerve innervates the adductor muscles. If damaged, unopposed abduction occurs and the patient swings the leg wide.

An 11-year-old girl presents with an 8-month history of right hip and thigh pain. The patient's health has been good overall with the exception of her being diagnosed as obese. You decide to order X-rays because you suspect the patient might have slipped capital femoral epiphysis (SCFE). Question If this patient is diagnosed with SCFE, what should be the initial management and what definitive treatment should follow? Answer Choices 1 No crutches, then surgical pinning in situ 2 No crutches, then traction 3 Crutches, then surgical pinning in situ 4 Crutches, then traction 5 SCFE is self-limiting and will resolve in 3-6 months

Crutches, then surgical pinning in situ The correct answer is crutches, then surgical pinning in situ. SCFE is a disorder that occurs when there is separation of the proximal femoral epiphysis through the growth plate. It usually occurs during early adolescence and is more common in males. Occasionally, there can be a metabolic cause, such as thyroid or pituitary diseases; obesity is also a known risk factor. Patients usually present with pain and/or a limp that has been going on for some prolonged period of time. Pain can be referred to the thigh or the medial side of the knee in some cases. Diagnosis is made based on X-ray findings; both anteroposterior and lateral X-rays should be taken. Treatment is important due to the risk of avascular necrosis in about 30% of these patients. When a patient presents with non-acute SCFE, such as in this case, the definitive treatment is to pin the slip. Until this can take place, patients should not bear any weight on the affected side, hence the crutches. 'No crutches, then surgical pinning in situ' is not the correct answer. The surgical correction is the most definitive treatment of a patient with chronic SCFE, but they need to be non-weight bearing until surgery can take place. This limits the long-term effects that the slip can have on the patient. 'No crutches, then traction' is not the correct answer. Crutches are needed in order to keep the patient non-weight bearing until surgery can take place. This limits long-term sequelae. Traction can be an appropriate treatment in patients who have an acute case of SCFE. The patient's SCFE is not likely to be acute given the presence of symptoms for 8 months. 'Crutches, then traction' is not the correct answer. Crutches in the immediate time period are pertinent to limit weight bearing. However, traction is an appropriate treatment in patients who have an acute case of SCFE. The patient's SCFE is not likely to be acute given the presence of symptoms for 8 months. 'SCFE is self-limiting and will resolve within 3-6 months' is not the correct answer. SCFE is not self-limiting, and NOT treating the patient will likely result in avascular necrosis of the affected side. Treatment is imperative in these patients.

A 5-year-old girl presents after falling off a shopping cart, tripping, and then falling onto her right arm. On examination, temp is 98.7, pulse 97, respirations 18, blood pressure 127/80. She is alert, oriented, and in no acute distress. Significant findings related to the right arm, which was mildly swollen, deformed, and diffusely tender. There was decreased range of motion of the right elbow due to pain. Sensation was intact. A radiographic examination was performed. Question What is the most common long-term complication associated with the radiology finding? Answer Choices 1 Cubitus varus 2 Myositis ossificans 3 Median nerve injury 4 Nonunion 5 Volkmann's contracture

Cubitus Varus Cubitus varus is an elbow deformity with a decreased carrying angle (so that, with the arm extended at the side and the palm facing forward, the forearm and hand are held at less than 5 degrees). There is deviation of the forearm toward the midline of the body. It is the most common complication of a supracondylar fracture of the elbow and is usually caused by poor reduction of the fracture with medial displacement, internal rotation, and extension of the distal fragment. The distal fragment then tilts into a varus position. The use of AP, lateral, and oblique view X-ray and CT scan to determine the position of the distal fragment and ensure that its rotation is less than 10 degrees is helpful in preventing this. Cubitus varus occurs in various studies ranging from 3 to 57%. It produces a cosmetic deformity, but is usually of little functional significance. Proximal migration of the fracture fragment as well as nonunion with displacement can lead to a valgus deformity, cubitus valgus. Most supracondylar fractures (95%) are extension fractures with posterior displacement of the distal fragment. Flexion fractures (5%), which are much rarer, cause the distal fragment to move forward. In the x-ray, the fracture line runs transversely just above the condyles. 5% of supracondylar fractures are associated with vascular injury. This is more likely with posterolateral displacement of the distal fracture fragment which then displaces the neurovascular bundle over a spike on the metaphyses. This presents as a pulseless pink hand. Less than 1% of children with vascular compromise develop a compartment syndrome due to injury of the brachial artery. This can be masked by a median nerve injury. Contracture occurs as a result of inadequate arterial perfusion and venous stasis. Within 4-6 hours, irreversible muscle necrosis begins. The resulting edema further impairs the circulation. The area with the most degeneration of the muscle in the forearm is the middle third of the muscle belly of the pronator and flexor muscles. Fibrosis and contracture follow. If blood supply does not improve by closed reduction, then open reduction and surgical exploration of the artery is indicated. Nerve injuries occur in 5-19% of these fractures and most often are neurapraxias. In these injuries, nerve continuity is maintained, but there is focal demyelination and a conduction block. In type III fractures up to 50% can show median nerve deficits. The most commonly involved nerve is the anterior interosseous branch of the median nerve, but the radial and ulnar nerves can be involved. Motor function generally recovers occurs in 3-4 months and sensory function in 6 months, both as remyelination occurs. Myositis ossificans is an extremely rare complication of supracondylar fracture in children. Calcification in a hematoma in the brachialis muscle over the anterior aspect of the elbow joint, around the periosteum and the capsular ligaments may all contribute to this new bone formation. Early findings consist of slight warmth with limitation of movement due to muscle spasm. This progresses to a consolidating stage in which a firm lump may be palpated in front of the elbow. In advanced stages, a firm bony lump surrounds the elbow with total loss of movement. Serial radiographs are obtained to distinguish it from osteosarcoma where the calcification extends from center to periphery. Surgery is indicated only if it obstructs movements or impinges on nerves.

A 3-year-old boy has velvety lax skin, hyperextensible joints, and mitral valve prolapse. What is the most likely diagnosis? Answer Choices 1 Marfan syndrome 2 achondroplasia 3 epidermolysis bullosa 4 Ehlers-Danlos syndrome 5 osteogenesis imperfecta

Ehlers-danlos syndrome There are several different types of Ehlers-Danlos syndrome (EDS). The clinical vignette above describes the classical, type I variant. The velvety, distensible skin, hyperextensibility of many joints, mitral valve prolapse, and hernia are all associated with the type I variant. Although the exact nature of the biochemical defect is just now being sorted out, all types of EDS are due to mutations in collagen genes. In recent experimental studies, a mutation in the gene encoding the α1(V)-chain of type-V collagen, was identified. Type- V collagen is associated with type-I collagen and is probably involved in assembly of type-I collagen fibers. Type-I collagen is abundant in ligaments of joints and in the dermis, thus the characteristic and unusual symptoms in the skin and joints.

A 17-year-old female distance runner with no significant PMH complains that she has diffuse, aching anterior knee pain that is worsened when she walks up or down stairs or when she squats down. There has been no acute trauma, but she has been increasing her running mileage. No effusion is present. What is the probable diagnosis? A Anterior cruciate ligament tear B Medial meniscal tear C Osteoarthritis of the knee joint D Patellofemoral syndrome E Posterior cruciate ligament tear

D Patellofemoral syndrome is a common condition in active adolescents due to repetitive stresses on the patellofemoral joint. This can be exacerbated by altered patellofemoral tracking due to growth and development in adolescents. Other biomechanical issues that may contribute to this problem include poor flexibility, weakness of the vastus medialis muscle, which results in a more lateral tracking of the patella, and excessive foot pronation. This condition does not produce an effusion. Anterior and posterior ligament tears would be highly unlikely given that there was no history of an acutely traumatic event and because ligaments are fairly vascular structures, an effusion would be expected with any cruciate ligament tear. Meniscal tears in a young patient are generally associated with a traumatic twisting event and while not as vascular as a ligament, meniscal tears generally lead to the gradual development of an effusion. Older patients may experience a degenerative tear with minimal trauma, but that is unlikely in our 17-year-old patient. Osteoarthritis is generally associated with gradual wear and tear after many years or the process can be accelerated after a significant acute trauma to a joint. Neither scenario is applicable to this patient.

You are evaluating a 32-year-old male who has possibly injured his Achilles tendon while planting his foot into the soft ground while playing touch football. When evaluating a patient with this suspected injury, you have the patient plantar flex his ankle as part of the exam. Based on your knowledge of anatomy, what two plantar flexing muscles attach into the Achilles tendon? A Anterior tibialis and soleus B Flexor hallucis longus and flexor digitorum C Flexor hallucis longus and gastrocnemius D Gastrocnemius and soleus E Peroneus longus and peroneus brevis

D The gastrocnemius and soleus muscles form a common tendon at their distal insertions known as the Achilles tendon. This the strongest and thickest tendon in the body and it attaches at its distal end onto the calcaneus bone of the foot. Contraction of the gastrocnemius and soleus muscles are responsible for plantar flexion of the foot and ankle. The anterior tibialis muscle is one of the important dorsiflexors of the foot and ankle. Flexor hallucis longus and flexor digitorum are both involved in plantar flexion, but are not as strong as the gastrocnemius and soleus muscles and do not attach to the Achilles tendon. The flexor hallucis longus is primarily responsible for plantar flexion of the great toe and flexor digitorum produces plantar flexion of the remaining toes. The peroneus longus and peroneus brevis muscles do contribute to plantar flexion, but are not connected to the Achilles tendon. They are also the primary evertor muscles of the foot and ankle.

What is the most common joint dislocation in children? A Ankle B Elbow C Finger D Knee E Shoulder

Elbow In children the elbow is the most commonly dislocated joint and it is the third most common joint dislocation for adults. The shoulder and finger are dislocated more frequently than the elbow in adults. Knee dislocations at the femur-tibia joint are rare (patellofemoral dislocations are more common) and ankle dislocations are also relatively rare.

A 72-year-old female presents with a several month history of symmetric pain to the hands, wrists, and feet. She also is noticing a change to the appearance of the joints of the hands and feet, with a more "nobby" appearance than before. When the patient wakes up in the morning, the joints are particularly stiff and red at times. Out of all the areas that are painful, it is the hands that are the most painful. Based on this history, what physical exam findings would you expect to find on the hands that are classic findings of this disease? A Radial deviation and subluxation of the DIP joints B Radial deviation and subluxation of the MCP joints C Radial deviation and subluxation at the PIP joints D Ulnar deviation and subluxation at the MCP joints E Ulnar deviation and subluxation at the PIP joints

D Ulnar deviation and subluxation at the MCP joints Rheumatoid arthritis is a systemic autoimmune disease that is incurable and affects woman more than men by a 3 to one ratio. It can damage any joint, but it most often affects joints of the hands, wrists, knees, feet and ankles. The classic appearance of hands that have been afflicted with progressive rheumatoid arthritis is that they demonstrate ulnar deviation and subluxation at the MCP joints. Involvement of the PIP joints is not uncommon, but DIP joints are generally not as significantly affected. Radial deviation is not a typical characteristic of rheumatoid arthritis of the hands.

Which of the following joints has the lowest occurrence rate of osteoarthritis? A Elbows B Hands C Hips D Knees E Spine

Elbows A Because the elbow is not a weight bearing bone, the rate of osteoarthritis in the elbow is considerably less that what is found in locations like the hips, knees and spine. The hands have one of the highest rates of occurrence of osteoarthritis, likely due to their near constant use and propensity for minor (or major) injury. When elbow arthritis does develop it is often post-traumatic osteoarthritis related to a significant injury in the past that disrupted joint surface integrity or as a result of rheumatoid arthritis, a systemic illness. Osteoarthritis of the elbow will generally present with pain, stiffness, and decreased range of motion. Osteophytes that form on the medial elbow might be implicated should neurological symptoms develop that correlate with ulnar nerve distribution as this nerve does pass in close proximity to the elbow on the medial side.

You are evaluating a 53-year-old post-menopausal patient during her routine annual examination in a primary care office. You notice that the patient has not been properly assessed for risk for osteoporosis, and does have some complaints that are of concern for osteoporosis. Based on this history, and the standard of care, what test is used to confirm osteoporosis in this patient? A Bone scan B CT scan C DEXA scan (dual energy x-ray absorptionmetry) D MRI scan E X-ray

DEXA C A DEXA scan is the best test to determine bone density and is commonly used as a screening tool for those at risk of developing osteoporosis. This test is fast, reproducible and exposes patients to a relatively low dose of radiation. The results are reported as Z and T scores. Z scores indicate how the patients bone density compares to peers and T scores compare the patient to young, health individuals. Z and T scores are presented in the form of standard deviations below the comparison group. If the T score is 0 to greater than -1 the test is interpreted as normal. T scores of -1 to -2.5 are indicative of someone with osteopenia and a T score lower than -2.5 is consistent with osteoporosis. None of the other imaging options presented are ideal for determining bone density, but they each play important roles in other aspects of patient care. Bone scans are best suited for imaging studies designed to look for occult fractures, tumors, inflammatory or infectious processes within the bones, or metabolic bone diseases. CT is useful for a detailed examination of bone when looking for fractures or lesions and intra-articular pathology. MRI is helpful in the evaluation of spinal column pathology and various soft tissue injuries involving the muscles, tendons, ligaments, and cartilage. MRI is also useful in the evaluation of stress and occult fractures, osteomyelitis and early osteo necrosis. Radiographs (x-ray) are useful for most initial evaluation of musculoskeletal pathology, but are somewhat limited in their usefulness in visualizing some conditions. Low bone density can be perceived on x-ray, but only after the bone loss has been rather extensive. It is not a great early screening tool for osteoporosis

You are evaluating a 67-year-old female who has history and physical exam findings consistent with osteoarthritis. Based on your knowledge of the disease, which joints are most commonly affected? A Carpometacarpal (CMC) B Distal interphalangeal (DIP) C Metacarpophalangeal (MCP) D Proximal interphalangeal (PIP) E Radiocarpal

DIP Osteoarthritis can affect all of the joints mentioned, but generally has the highest prevalence in the DIP joints, especially the second DIP joint. Osteophyte formation at the DIP joints produce enlargements referred to has Heberden's nodes. The first carpometacarpal joint is likely the second most commonly affected joint. This painful and potentially debilitating condition at the base of the thumb can make grasping activities difficult and be exacerbated by prolonged or strenuous use of the thumb. While metacarpophalangeal and proximal interphalangeal joints can definitely be damaged in osteoarthritis, they are the more classic locations for rheumatoid arthritis changes to be manifested. Radiocarpal joints at the wrist are susceptible to osteoarthritis, but not at the same high frequency as the DIP joints.

What joint is not commonly affected in RA?

DIP joint

A 45-year-old man presents to his orthopedic office for a 1-month history of worsening pain in his left groin and buttock. He had a surgical hip fracture repair approximately 4 months ago, 2 days after a fall in a bowling alley. His post-operative course was unremarkable, with normal healing and return to activities. However, he now notes increasing difficulty with walking and has had to limit activities again due to pain. He has tried "all" of the over-the-counter pain medications without relief. His past medical history is unremarkable, aside from the surgical fracture repair. He has no chronic medical conditions, no other surgeries, and takes no chronic medications. He has no medication allergies. On physical exam, the patient is asked to walk across the room. He limps and appears to be in pain. The entire hip region is evaluated and no erythema, ecchymosis, or edema is noted. The incision site is healing well, with no tenderness and no fluctuant tissue. With range of motion tests, the patient reports pain, especially with internal rotation of the left hip. He is sent for several tests. Blood culture is negative. X-ray shows a crescent sign over the left femoral head. An MRI shows a low-signal intensity on the left femoral head, and increased uptake is seen also in this region with nuclear bone scan. The fracture site shows normal post-operative changes. Question What best describes the underlying mechanism responsible for this patient's complication from his hip fracture? Answer Choices 1 Autoimmune destruction of synovial membrane 2 Damage to bone marrow, resulting in decreased white blood cell immunity 3 Damaged vascular supply to femoral head 4 Development of traumatic arthritis 5 Introduction of bacteria during surgery

Damaged vascular supply to femoral head Explanation This patient's condition is suspicious for avascular necrosis (or the more preferred term, osteonecrosis) of the hip. Avascular necrosis (AVN) is a relatively common complication following a traumatic hip dislocation and/or fracture and results in damaged vascular supply to the femoral head, with subsequent bone death. Some potential conditions on the differential may include arthritis, malignancy, and osteomyelitis. The history and physical exam may be similar in all these conditions, but the history of hip fracture and the diagnostic studies support a diagnosis of AVN. Definitive treatment is surgical, with a variety of approaches, from core decompression to bone grafting. Autoimmune destruction of synovial membrane is one of the mechanisms associated with rheumatoid arthritis. Damage to bone marrow, resulting in decreased white blood cell immunity, is not a process specific to one condition. Because only a minimal portion of one bone is affected, this patient's marrow production of red and white blood cells would be expected to be normal. Development of traumatic arthritis is another potential complication following hip fracture and/or dislocation. However, this patient's current findings show bone death, not joint inflammation. He could still be at risk for developing post-traumatic arthritis, but this is not the current explanation of his symptoms. Introduction of bacteria during surgery, leading to osteomyelitis or a joint infection, is a possible complication of this patient's hip fracture (but less common than AVN). If a bacterial infection were responsible for this patient's symptoms, his blood culture would be expected to be positive for a causative organism, and imaging findings would differ.

A 47-year-old grossly obese woman presents with left non-dominant shoulder pain and limited motion. The pain began about a month ago, and her shoulder has progressively lost motion during that time until now she cannot reach overhead with that arm. There is no history of trauma; it does not wake her at night; and she can sleep on the left side. Her past medical history is significant for Diabetes Type I and hypothyroidism. She is on regular insulin and NPH insulin, as well as levothyroxine (Synthroid). What would you expect to find on exam? Answer Choices 1 Decrease in active range of motion, but full passive range of motion 2 Decrease in active and passive range of motion (by at least 50%) 3 Minimal pain with motion especially at the extremes of motion 4 Weakness with isometric rotator cuff muscle testing 5 Tenderness of the sternoclavicular (SC) joint

Decrease in active and passive range of motion (by at least 50%) Explanation This is adhesive capsulitis (frozen shoulder). Diabetes and hypothyroidism are common risk factors. Pain usually precedes the progressive stiffness. Age is commonly between 40 and 60 years old. Loss of both active and passive range of motion by at least 50% is the hallmark of this disorder. Motion is painful, especially at the extremes. Tenderness is common at the deltoid insertion and not over the SC joint. The rotator cuff muscles are not as weak with isometric testing, as if the rotator cuff muscle-tendon were inflamed. This is a disorder of the glenohumeral joint capsule and not the rotator cuff muscles. There may be some weakness due to atrophy from non-use because of the pain and stiffness.

A 3-year-old boy is brought to the pediatrician after his mother noticed that he has been "walking abnormally." She also says that the boy gets tired easily and rarely goes out to play. He has always had a tendency to fall often. On examination, the calf muscles are enlarged. The boy walks with an awkward gait on his toes. Gower's sign is positive. Lab studies reveal elevated creatine phosphokinase levels. You order genetic testing and muscle biopsy; they show a mutation in the dystrophin gene and absence of dystrophin, respectively. Question What is the most probable diagnosis? Answer Choices 1 Duchenne muscular dystrophy 2 Becker muscular dystrophy 3 Limb girdle dystrophy 4 Congenital dystrophy 5 Myotonic dystrophy

Duchenne muscular dystrophy DMD is an X-linked recessive progressive muscular dystrophy characterized by proximal muscle weakness and wasting of voluntary muscles. Pseudodystrophy results in enlargement of calf and deltoid muscles. The symptoms are evident by 3 to 7 years of age. Being X-linked, boys are more often affected, and females are usually carriers. One-third of DMDs are due to a new mutation, and 10% of these are due to gonadal mosaicism. DMD is caused by a mutation in the dystrophin gene at locus Xp21. Diagnosis is by elevated creatine phosphokinase levels, absent dystrophin on muscle biopsy, and mutation detected by genetic testing. Becker muscular dystrophy is similar to DMD, except that the diagnosis is usually made in adulthood and progresses at a slower rate. Dystrophin is usually present, but in a lesser percentage than normal. Limb girdle dystrophy affects the shoulder and hip muscles and is characterized by slowly progressive muscular weakness. Congenital dystrophy is present at birth and is characterized by slowly progressive muscular weakness and joint deformities. Myotonic dystrophy can occur at any age and is characterized by muscular dystrophy, along with other findings such as cataracts, myotonia, etc. The mutation in the dystrophin gene and absence of dystrophin in the muscle favor a diagnosis of DMD and not the other dystrophies.

In a boxer's fracture, the presence of how much angulation of the fifth metacarpal neck would require referral for reduction as opposed to treatment with a simple ulnar gutter splint? A Less than 10 degrees B 10-19 degrees C 20-29 degrees D 30-39 degrees E Greater than 40 degrees

E When there is greater than 40 degrees of angulation or an extensor lag (the patient cannot fully extend the affected finger) then a referral is required. Lesser degrees of angulation without an extensor lag can generally be handled conservatively with an ulnar gutter.

A 14-year-old boy is seen for a sports physical for the freshman basketball team. Past history is significant for a high degree of myopia bilaterally, first diagnosed at age 4 years, and a dislocated shoulder at age 10 years that was easily reduced. Family history is significant for several unidentified ancestors having died in their forties of an unidentified cardiovascular disorder. Physical examination revealed normal vital signs. Height is 6' 1" and weight 145 lbs. The upper to lower segment ratio is 0.65 (decreased). Arm span was 76". The palate is highly arched and mild pectus excavatum is present. A 2/6 early diastolic murmur is present and best heard at the second intercostal space at the right sternal border. Arachnodactyly of the fingers and toes and generalized loose jointedness and pes planus are also present. Question As to whether he should be permitted to play basketball, what should be advised? Answer Choices 1 Echocardiography needs to be performed, with follow-up thereafter 2 Family history needs to be further elucidated 3 He cannot play basketball due to the prior shoulder dislocation 4 Heart murmurs are a relative contraindication to participation in competitive athletics 5 There is no contraindication to his playing basketball

Echocardiography needs to be performed, with follow-up thereafter The presentation is highly suggestive of Marfan syndrome, with skeletal findings (decreased upper/lower segment ratio, dolichostenomelia, arachnodactyly, pectus deformity, pes planus, and hyperextensible joints), ocular manifestations (here, high myopia, but the most characteristic finding is ectopia lentis), and a suggestive family history of early cardiovascular death and cardiac findings in the patient. The most important factor in determining this young man's fitness for competitive athletics is identification of the nature and extent of any cardiac involvement (e.g., mitral valve prolapsed, aortic regurgitation, dilated aortic root). While elucidating family history would be helpful regarding the diagnosis of Marfan syndrome, it will not identify the cardiac abnormality (if there is one) in this patient. A prior, nonrecurrent shoulder dislocation is not a contraindication to athletic participation. Heart murmurs can be innocent or indicative of mild, moderate, or severe cardiac (or noncardiac) conditions. In and of themselves, they are not a contraindication to athletic participation. The cause must first be determined. One cannot decide upon the appropriateness or not of athletic participation until a cardiac evaluation is performed.

A 3-year-old boy has velvety lax skin, hyperextensible joints, and mitral valve prolapse. What is the most likely diagnosis? Answer Choices 1 Marfan syndrome 2 achondroplasia 3 epidermolysis bullosa 4 Ehlers-Danlos syndrome 5 osteogenesis imperfecta

Ehlers-Danlos syndrome Explanation There are several different types of Ehlers-Danlos syndrome (EDS). The clinical vignette above describes the classical, type I variant. The velvety, distensible skin, hyperextensibility of many joints, mitral valve prolapse, and hernia are all associated with the type I variant. Although the exact nature of the biochemical defect is just now being sorted out, all types of EDS are due to mutations in collagen genes. In recent experimental studies, a mutation in the gene encoding the α1(V)-chain of type-V collagen, was identified. Type- V collagen is associated with type-I collagen and is probably involved in assembly of type-I collagen fibers. Type-I collagen is abundant in ligaments of joints and in the dermis, thus the characteristic and unusual symptoms in the skin and joints.

A 67-year-old presents with a 2-year history of numbness in the lower extremities. The most relevant complaint is gait impairment and instability due to loss of deep tendon sensation. Neurologic examination shows a disturbed sense of position with preserved tactile and temperature sense, normal muscular force, and normal reflexes. Electrophysiologic evaluation excludes a sensitive polyneuropathy Question What is the most likely cause of the patient's symptoms? Answer Choices 1 Lipoma 2 Astrocytoma 3 Hemangioblastoma 4 Oligodendroglioma 5 Ependymoma

Ependymoma Explanation This patient has an ependymoma. Ependymomas make up approximately 60% of spinal cord tumors. They are relatively slow-growing and may develop anywhere along the entire length of the spinal cord. However, a large percentage of these tumors are found at the lower sections of the cord. They develop from ependymal cells and can be seen in all age groups; however, the most common age group is between 20 and 40 years. Men and women are affected equally. Tumors may metastasize here from other parts of the body or they may be made of the same types of cells. Spinal tumors may be intradural, extramedullary, intramedullary, or extradural. Astrocytomas are the 2nd most common type of spinal cord tumor. They are found more often in children than in adults. Other rare types include developmental tumors, oligodendogliomas, lipomas, and hemangioblastomas. The usual presentation is pain, and classically it is aggravated in supine or flat position. As the tumor advances, abnormal sensation and eventually weakness follow, until finally the normal bladder and bowel function is lost. Surgery is the treatment of choice. Radiation therapy is sometimes given if the total surgical removal of the tumor is not possible or if the tumor appears to grow rapidly.

A 51-year-old-woman presents due to menstrual irregularity, hot flashes, and mood changes. Physical examination reveals an atrophic vagina and breasts that have decreased in size. You explain that those symptoms and signs are physiologic and related to her age. The patient decides not to start hormonal replacement therapy (HRT). Question 8 years later a dual-energy X-ray absorptiometry (DEXA) reveals a T score of -3, reflecting a very low bone density. Change in the serum level of what hormone or electrolyte is most likely responsible for the alterations detected by the DEXA scan? Answer Choices 1 Calcium (Ca++) 2 Cholecalciferol (vitamin D) 3 Parathyroid hormone (PTH) 4 Estrogen 5 Progesterone

Estrogen Estrogen level fall around and after menopause due to age-linked decline in number of ovarian follicles. This permanent loss of estrogen may result in the increase of bone re-absorption named osteoporosis. Normally, estrogen inhibits apoptosis in osteoblasts (bone-forming cells) and induces apoptosis in osteoclasts (bone-resorbing cells). Therefore, the absence of this hormone will trigger bone reabsorption, leading to loss of bone mass and interconnections in trabecular bone. Levels of calcium (Ca++), cholecalciferol (vitamin D), and parathyroid hormone (PTH) are normal in menopause-related osteoporosis. A complex of Ca++ and phosphate (PO43−) compose the inorganic matrix of bone. Vitamin D is one of the fat soluble vitamins, along with vitamins A, E, and K. The active form of vitamin D, 1,25 (OH)2D3, facilitates Ca++ absorption from the intestine, bone growth, and mineralization. Vitamin D is ingested in the diet or manufactured in the skin through ultraviolet light activation. The conversion of the inactive metabolite 25(OH)D3 to the active metabolite 1,25(OH)2D3 occurs is the kidney. PTH increases the release of Ca++ and PO43− from bone to blood. In the kidney, it increases reabsorption of Ca++ while decreases reabsorption of PO43−, and stimulate the conversion of 25(OH)D3 (inactive) to 1,25(OH)2D3 (active). Like the estrogen level, progesterone levels also fall after the menopause. However, the decrease in progesterone is not responsible for the signs and symptoms of menopause; it is also not responsible for the signs and symptoms of osteoporosis.

An 89-year-old Caucasian man is brought in by his daughter due to pain in his left shin. He bumped his leg on a coffee table about 3 weeks ago; he developed some mild discomfort, bruising, and a small gash in the skin. It had seemed as if his condition was healing well, but his condition has worsened over the past few days. Now he is moaning due to pain, and he says it hurts to walk on the leg. He describes the pain as "horrible" and an 8 on a 1 - 10 pain scale. The daughter reports the pain keeps him up at night and is unresponsive to ibuprofen and narcotic pain pills. He denies fever and chills. On physical exam, the patient is in obvious pain and is assisted to the exam table with limited weight bearing on his left leg. The lower extremities are examined; significant findings include healing and a scabbed lesion of approximately 3 cm in length across mid-tibia, with surrounding erythema and edema. Tenderness is elicited along the shin, extending well past the area of erythema. Homan's sign is negative. Distal pulses, temperature, coloration, knee range of motion, and lower extremity reflexes are symmetric and normal. Right lower extremity is normal. Question What test finding would be most diagnostic for this patient's likely condition? Answer Choices 1 Elevated C-reactive protein 2 Elevated erythrocyte sedimentation rate 3 Elevated white blood cell count (WBC) 4 Evidence of tibial edema and necrosis on magnetic resonance imaging (MRI) 5 Soft tissue swelling on plain radiography

Evidence of tibial edema and necrosis on magnetic resonance imaging (MRI) Explanation Evidence of tibial edema and necrosis on magnetic resonance imaging (MRI) is the most diagnostic for this patient's condition. Along with this patient's history and physical, this MRI finding indicates bone involvement and osteomyelitis, a condition that can develop in the elderly after trauma. Staphylococcus aureus is a common cause, but biopsy reveals the causal organism(s). Osteomyelitis can be acute or chronic, and it can affect all ages of patients. Certain groups, such as diabetics with foot ulcers and peripheral vascular disease or intravenous drug users, may be especially at risk. Diagnosis is based upon history, physical, and a combination of diagnostic tests. Elevated C-reactive protein and erythrocyte sedimentation rate both indicate inflammation in the body. However, these tests do not point to the specific etiology of the inflammation. An elevated white blood cell count (WBC) likewise indicates infection and/or inflammation, but it does not help differentiate cellulitis versus osteomyelitis. Plain radiography (X-rays) is often recommended early in the workup of a suspected case of osteomyelitis, both because of low cost and widespread availability. If positive for bone involvement, a diagnosis can be made. However, most of the time, especially in early osteomyelitis, the plain films are non-specific and may simply show soft tissue swelling. Further testing would be recommended.

A 13-year-old boy presents with pain in his right leg. The pain has been present for about 2 months, but it has worsened with time; he has also developed a low-grade fever. He denies any known injury to the area. On examination, there is tenderness and mild swelling near the right fibula. X-ray reveals a lesion involving the right fibula, with a periosteal onion-skin reaction. Question What is the most likely diagnosis? Answer Choices 1 Osteoid osteoma 2 Ewing's sarcoma 3 Multiple myeloma 4 Osteosarcoma 5 Osteochondroma

Ewing's sarcoma is correct. The patient's X-ray demonstrates a periosteal onion-skin reaction, which is classic for Ewing's sarcoma. Osteoid osteoma, multiple myeloma, osteosarcoma,and osteochondroma are incorrect responses. While these are all causes of bone lesions, they are not associated with the periosteal onion skin reaction seen in Ewing's sarcoma.

A 13-year-old boy presents with pain in his right leg. The pain has been present for about 2 months, but it has worsened with time; he has also developed a low-grade fever. He denies any known injury to the area. On examination, there is tenderness and mild swelling near the right fibula. X-ray reveals a lesion involving the right fibula, with a periosteal onion-skin reaction. Question What is the most likely diagnosis? Answer Choices 1 Osteoid osteoma 2 Ewing's sarcoma 3 Multiple myeloma 4 Osteosarcoma 5 Osteochondroma

Ewings sarcoma Explanation Ewing's sarcoma is correct. The patient's X-ray demonstrates a periosteal onion-skin reaction, which is classic for Ewing's sarcoma. Osteoid osteoma, multiple myeloma, osteosarcoma,and osteochondroma are incorrect responses. While these are all causes of bone lesions, they are not associated with the periosteal onion skin reaction seen in Ewing's sarcoma.

A 40-year-old recreational tennis player presents with pain in his playing-side elbow since a tournament last weekend. He played 5 matches of singles and 2 matches of doubles games. He cannot even hold a pen without pain. He says he needs to get better fast because he has another tournament coming in a week. On exam, he has pain with extension against resistance of his affected side wrist. Question What treatment option should be recommended on this first visit? Answer Choices 1 Recommend he do strengthening exercises now to get ready for the stress on his elbow at the next tournament. 2 Prescribe him muscle relaxants, since spasms cause the pain. 3 Tell him he should switch arms to give his normal playing arm a rest. 4 Inject him today with a long acting anesthetic agent to relieve his pain. 5 Prescribe an NSAID of choice and recommend skipping the next tournament.

Explanation Activity modification and NSAIDs are first line treatment for lateral epicondylitis. This may have to include changing to a lighter racket, changing the tension in the strings, or overwrapping the grip to make it slightly larger. Use of commercially available "tennis elbow" straps may be beneficial as well. Once the acute pain phase subsides, gentle stretching and strengthening exercise will be the key to resolution and prevention. Muscle relaxant medications have no role in treatment of this disorder. If symptoms persist despite conservative treatment, injection of a corticosteroid to the point of maximal tenderness at the lateral epicondyle may be helpful as well.

A 42-year-old African-American woman presents with acute deterioration of her mental state; she is hallucinating and talking about people hiding under her bed. She cannot sleep, and she refuses to eat. Several months ago, her family practitioner told her that she might have lupus erythematosus; however, she refused further investigations. On examination, she appears agitated and is disorientated to place and time; she has impaired memory, and she confirms the presence of auditory hallucinations. Question What test would best confirm the diagnosis of cerebral systemic lupus erythematosus? Answer Choices 1 Antinuclear antibodies in blood 2 Oligoclonal bands in cerebrospinal fluid 3 Antineuronal antibodies in blood 4 Protein levels in cerebrospinal fluid 5 Complement activity in serum

Explanation Antinuclear antibody (ANA) testing is used for serologic testing for SLE. There are 2 major types of ANA: autoantibodies to DNA and histones and autoantibodies to extractable nuclear antigens. The first group concerns antibodies against single and double-stranded DNA (dsDNA); significant levels of anti-dsDNA antibodies are considered confirmatory in the diagnosis of SLE (anti-histone antibodies are considered confirmatory in the diagnosis of drug-induced SLE). Most instances of the second group (autoantibodies to extractable nuclear antigens) are specific to a disease, e.g., autoantibody to Smith antigen is considered to be specific for SLE. Other examples include: autoantibodies to ribonucleoproteins, SSA/Ro, or SSB/La, Scl-70, Jo-1, PM1, etc. Oligoclonal bands are immunoglobulins seen in serum, plasma, or cerebrospinal fluid (CSF). The presence of oligoclonal bands in CSF with their absence in serum shows that immunoglobulins are produced within the central nervous system. Since 80-90% patients with multiple sclerosis have oligoclonal bands, they are considered an indicator in the diagnosis of multiple sclerosis. They are not confirmatory for SLE. Antineuronal antibodies are commonly associated with paraneoplastic neurological diseases (Anti-Hu, Yo, Ri amphiphysin, Tr, CV2, and Ta antibodies). Some of these antibodies are specific for certain types of cancer or neurological syndromes. They are not confirmatory for SLE. Protein levels in CSF are not specific; they may be elevated in several conditions (e.g., diabetes mellitus, brain tumor, brain abscess, meningitis, multiples sclerosis, hemorrhage, Guillain-Barre ayndrome); they may be normal in viral infections, or they may be low (recent lumbar puncture, chronic CSF leakage, water intoxication). Complement system plays a role in many diseases with an immune component (asthma, glomerulonephritis, multiple sclerosis, inflammatory bowel disease, etc); therefore, its levels are not specific for SLE.

The mother of a 3-year-old boy states that her child does not walk normally. The mother feels that the child has been falling more frequently; she also tells you that his symptoms are worsening quickly. On exam, the child uses his hands to push himself into an upright position from sitting on the ground. He walks with a waddling gait. There is hypertrophy of the bilateral calves, lumbar lordosis, and hyporeflexia. There is also cognitive delay. His labs reveal an increased creatine kinase. Question What is the most likely diagnosis? Answer Choices 1 Facioscapulohumeral muscular dystrophy 2 Myotonic muscular dystrophy 3 Duchenne muscular dystrophy 4 Becker muscular dystrophy 5 Limb-girdle muscular dystrophy

Explanation Duchenne muscular dystrophy is the correct response; symptoms are usually noted between the ages of 3 to 5 years in boys. The disease is present at birth; however, symptoms often are not noted until the child starts walking. A waddling gait, hypertrophy of the calves, lumbar lordosis, and hyporeflexia are all common exam findings with this disease. The way in which the child pushed himself into an upright position is known as Gower's sign. There is also an increase in creatine kinase. This is the most common form of muscular dystrophy in children. Facioscapulohumeral muscular dystrophy is incorrect; this disease usually starts later in the teenage years. Weakness is usually noted in the face, arms, and around the shoulders. Facial weakness is usually the initial sign of this disease. Myotonic muscular dystrophy is incorrect; it typically involves many other systems in addition to the musculoskeletal system. It is the most common adult form of muscular dystrophy. People with this disorder have long faces and drooping eyelids. Neck muscles and distal limb muscles are the first muscles to be involved. Becker muscular dystrophy is incorrect. The typical onset of symptoms is later than Duchenne, normally occurring after the age of 15 years old. It also has a milder course than Duchenne muscular dystrophy. It resembles Duchenne, but it is far less common. Hypertrophy of the calves would also be a finding with this disease. Limb-girdle muscular dystrophy is incorrect. Both male and female patients are affected by the disorder; its symptoms occur later than Duchenne, appearing anywhere from 10 to 40 years of age. Muscle weakness is present in the shoulder and pelvic muscles.

A 23-year-old man presents with a 2-week history of fever and back pain. The pain radiates from the midline to 1 side. The patient also has a 1-week history of right lower limb weakness and urinary incontinence. The patient gives history of trauma to the back following motor accident 1 month prior to presentation. Examination by touch and percussion demonstrates localized pain in the spine; you also note neurological losses on the right side of the body that include decreased sensation and weakness. The patient's temperature is 101 F, and his skin is moist. What investigative procedure has the greatest diagnostic accuracy in diagnosing the patient's condition? Answer Choices 1 Magnetic resonance imaging (MRI) 2 Myelography 3 Lumbar puncture (LP) 4 Computed tomography (CT) scanning 5 Plain radiographs

Explanation Magnetic resonance imaging (MRI) of the spine confirms the diagnosis of epidural abscess. Gadolinium-enhanced MRI is the imaging modality of choice for the diagnosis of epidural abscess. It is useful in distinguishing epidural abscess from the adjacent compressed thecal sac and other potential compressive lesions, such as a herniated disc. Epidural abscess is a disorder that is characterized by inflammation and a collection of infected material (pus) between the dura (the outer membrane covering the brain and spinal cord), and the bones of the skull or spine. The infection is usually bacterial (staphylococcus is common) but may be fungal or viral. The condition is very rare. Pus forms as a collection of fluid, destroyed tissue cells, white blood cells, and live and dead microorganisms. The infection spreads through the bones and membranes of the spine. The risks include boils (furuncles) on the skin, particularly on the back or scalp. Associated predisposing conditions include a compromised immune system, such as in patients with diabetes mellitus, AIDS, chronic renal failure, alcoholism, cancer, or following epidural anesthesia, spinal surgery, or trauma. Abscess may also result from the spread of any infection through the bloodstream from another body location. Symptoms include fever, chills, lower backache (mild, progressive, and typically radiating to an extremity), sudden or rapid progression of neurologic symptoms, loss of movement of an area of the body, weakness or paralysis, loss of sensation of an area of the body, numbness in a specific, limited area (localized), loss of bladder or bowel control, and male impotence. An examination often shows localized tenderness over the spine. A neurologic examination may indicate spinal cord compression, with involvement of the lower body (paraplegia), or the entire trunk, arms, and legs (quadriplegia). The extent of neurologic losses corresponds with the location of the lesion. A CSF examination may be normal or may contain a few white blood cells and slightly elevated protein levels. Myelography, which was commonly used previously to diagnose spinal epidural abscess, is no longer recommended because of the risk of introducing infection. Lumbar puncture also entails the risk of spreading bacteria into the subarachnoid space with consequent meningitis; therefore, it should not be performed. CT scanning is performed only when MRI cannot be performed. Plain radiographs occasionally demonstrate osteomyelitis, but they are of almost no utility. The goals of treatment are the relief of spinal cord compression and the eradication (cure) of the infection. The therapeutic method of choice is laminectomy for surgical decompression, combined with antibiotics. Conservative treatment alone is justifiable only for specific indications. Broad-spectrum antimicrobials or a combination of antibiotics are the most commonly used antimicrobials. Corticosteroids may occasionally be prescribed to reduce swelling and compression of the spinal cord.

A 12-year-old girl presents to the emergency room with worsening foot pain. 2 weeks ago, she stepped on a nail while wearing tennis shoes. The area was thoroughly cleaned, but she has developed worsening pain, redness, and drainage from the area. X-ray shows periosteal changes at the site of the wound. A wound culture is obtained. Question Which organism is most likely to grow on culture? Answer Choices 1 Staphylococcus aureus 2 Escherichia coli 3 Pseudomonas aeruginosa 4 Serratia marcescens 5 Salmonella enterica

Explanation Pseudomonas aeruginosa is correct. Pseudomonas aeruginosa is the most common cause of osteomyelitis when a penetrating object passes through the sole of the shoe. Staphylococcus aureus is incorrect. While Staphylococcus aureus is the most common cause of osteomyelitis overall, Pseudomonas aeruginosa is the most common cause when the infection has been caused by a puncture wound through the sole of a shoe. Escherichia coli, Serratia marcescens, and Salmonella enterica are incorrect. All of these organisms may cause osteomyelitis, but Pseudomonas aeruginosa is the most likely cause in this patient.

A 35-year-old man with no significant past medical history has been experiencing progressive, moderately severe right knee pain for the past 3 months. Pain was initially felt only at night; however, it is now constant throughout the course of the day for the last several weeks. It is especially severe upon ambulation and during knee ranges of motion, causing him to limp. He denies fever, chills, weight changes, history of gout, sexually transmitted diseases, hip or back pain, recent instrumentation, trauma, or injuries. His physical exam reveals an antalgic gait with limp, limited ranges of motion of the right knee, and a 3 cm diameter firm, tender mass at the distal femur. There is no erythema, crepitus, alignment deformity, or effusion noted. Question What is the next appropriate step in the management of this patient? Answer Choices 1 Order a serum uric acid level and prescribe indomethacin 2 Wrap the right knee in an elastic bandage and prescribe physical therapy 3 Perform a joint aspiration with microscopic cellular assessment 4 Obtain a blood culture and begin empiric dicloxacillin 5 Order an LDH, ALP, and a plain radiograph of the knee

Explanation This patient's manifestations are most suggestive of osteosarcoma. It is typically seen in patients in their second or third decade, occurs more frequently in men than in women, and is found in the metaphyseal areas of long bones, with 50% of lesions seen about the knee joint. The distal femur is the most common site, followed by the proximal tibia, and then the proximal humerus. The only blood tests with prognostic significance are lactic dehydrogenase (LDH) and alkaline phosphatase (ALP). Patients with an elevated ALP at diagnosis are more likely to have pulmonary metastases. In patients without metastases, those with an elevated LDH are less likely to do well than are those with a normal LDH. Other laboratory tests include a complete blood cell (CBC), including a differential and platelet level, electrolytes, and liver and renal function tests. The practitioner should also obtain plain films of the suspected lesions in 2 views. Elevation of the periosteum may appear as the characteristic Codman triangle; in approximately 60% of cases, extension of the tumor through the periosteum may result in a so-called sunburst appearance.

A 24-year-old man was playing basketball when he jumped up to shoot a basketball and landed on another player's leg. The patient grabbed his left ankle immediately and screamed in pain. His whole ankle started to swell, and he had difficulty bearing weight. His friends took him to the emergency room. On exam, the patient had tenderness to the tip of the lateral malleolus. Pain is elicited with forced ankle inversion. Talar tilt test and anterior drawer test cannot be assessed due to swelling and pain. X-rays are negative for a fracture. Question What ligament did the patient most likely sprain? Answer Choices 1 Anterior tibiotalar ligament 2 Tibiocalcaneal ligament 3 Posterior tibiotalar ligament 4 Tibionavicular ligament 5 Anterior talofibular ligament

Explanation The anterior talofibular ligament is the only correct choice, as this is the only ligament listed above that is in the lateral aspect of the ankle. The mechanism of injury and physical exam points to the lateral part of the ankle. The other 4 choices compose what is commonly known as the deltoid ligament complex and are all found medially. The patient jumped up to shoot the basketball and most likely had his foot in a plantar flexion; upon landing on another player, the ankle was inverted. The anterior talofibular ligament is the first to be injured and can be the only ligament injured in lateral ankle sprains. However, the stronger force can injure the other 2 ligaments in the lateral aspect of the ankle, the calcaneofibular ligament and posterior talofibular ligament. The special physical exam tests have very limited usefulness in this case, since this is an acute injury, and they should be used after the pain and swelling have gone down.

A 9-year-old girl presented to her pediatrician several months ago with arthralgias, myalgias, weight loss, and morning stiffness of her joints. She has since been diagnosed with Juvenile Rheumatoid Arthritis (JRA) and has been taking nonsteroidal anti-inflammatory (NSAIDs) medications to treat the symptoms. Her parents have recently taken her to see a rheumatologist in the hopes of making sure they are considering every form of therapy for the child despite positive results from the NSAIDs thus far. Question What first-line adjunctive treatment should be recommended? Answer Choices 1 Addition of a salicylate 2 Continued NSAIDs use is all that she requires 3 Addition of a disease-modifying antirheumatic agent 4 Addition of physical and occupational therapy 5 Addition of folic acid

Explanation The correct answer is addition of physical and occupational therapy. The prognosis of JRA is typically good, but it needs to be treated early and rather aggressively in order to prevent osteopenia, asymmetrical skeletal development, systemic manifestations, and chronic eye disease. Aggressive therapy refers to pharmacologic therapy as well as lifestyle and dietary changes. The goal of physical therapy (PT) is to strengthen muscles, decrease the impact loading on joints, and increase range of motion. Patients are given instructions for using moist heat on the joints and are also taught both passive and active assisted exercises to perform 2-3 times daily. Occupational therapy (OT) is used to improve the patient's body mechanics, posture, and other modalities in order to protect their joints. Patients are given instructions by an occupational therapist about proper posture, improvement in activities of daily living, body mechanics, and how to protect their joints in general. It is important for patients with JRA to use exercise during all stages of their disease. During the early stages, patients should use passive range of motion in order to maintain range of motion during the most painful period. As pain is improved and inflammation decreases, patients should slowly increase to active forms of exercise. Addition of a salicylate is not the correct answer. Salicylates can be used as first-line treatment for patients with JRA, but would not be used in conjunction with an NSAID. NSAIDs have become preferred over salicylates in recent years due to the decreased frequency of dosing, their liquid form, and diminished side effects. Continued NSAIDs use is all that she requires is not the correct answer. While NSAIDs are commonly used as a first-line agent and the patient has shown improvement, JRA is a disease that has historically shown better results with early and aggressive treatment. Aggressive treatment refers to the use of a combination of various forms of treatment, such as pharmacologic treatments, in conjunction with lifestyle modifications. Addition of a disease-modifying antirheumatic agent is not the correct answer. Disease-modifying antirheumatic agents (DMARDs) are considered a second-line form of treatment for patients with JRA unless the patient has contraindications to NSAIDs and salicylates. DMARDs are not given in conjunction with an NSAID, but may be used as combination therapy; most commonly, 2 DMARDs are used together. Combination therapy is first-line treatment in patients with erosions on radiograph or other signs of more aggressive disease at the time of diagnosis. Addition of folic acid is not the correct answer. Folic acid is given to children with JRA when they are started on MTX, a purine inhibitor, during combination therapy with another DMARD for patients with severe disease. MTX is an affective DMARD, and toxicity is decreased when used in conjunction with folic acid.

A 14-month-old girl presents in the middle of the night. Her mother tells you that she has been fussy and has not been feeding well for the past 48 hours. In the last 4 hours, she has developed a high fever; it was 102 degrees Fahrenheit when measured at home. Her mother also notices that the child appears reluctant to move her right leg; the girl appears to keep it rotated outward. Examination reveals a distressed, febrile child. Her cardiovascular, abdominal, and respiratory exams appear normal. Her right leg is held fixed in external rotation and partial flexion, with minimal hip joint effusion. A needle aspirate of the joint fluid reveals the following; WBC count - 60,000/cc, neutrophils 95% No crystals Gram stain is positive for Gram-positive cocci in clusters Question What is the most important step in preventing destruction of the joint? Answer Choices 1 Early administration of broad spectrum antibiotics 2 High dose acetaminophen administration 3 Arthrotomy and irrigation of joint space 4 Repeat needle aspiration 5 Administration of indomethacin/allopurinol

Explanation The correct answer is to perform an arthrotomy and irrigate the joint space. The child has septic arthritis, which is proven by the history and joint fluid analysis showing a white count of >50,000 with predominant neutrophils. In addition Gram stain shows organisms likely to be staphylococcus. Septic arthritis in a patient so young is an emergency. Without drainage, accumulation of pus in the joint results in rapid destruction of the articular cartilage and permanent disability. Infants often present with non-specific symptoms, and the classical symptoms of septic arthritis may appear late in the course of the disease. Early administration of antibiotics is incorrect. While antibiotics are important, they are not the primary means of preventing disability in a child with septic arthritis. High dose acetaminophen administration is incorrect. High doses of acetaminophen are hepatotoxic, and will help only to reduce pain. Repeat needle aspiration is incorrect. The initial aspirate leaves no doubt as to the diagnosis of septic arthritis; therefore, a repeat needle aspiration is unnecessary. Administration of indomethacin/allopurinol is incorrect. Both drugs are used in gout; they are not used in septic arthritis.

A 21-year-old man presents with a 3-month history of pain in his left shoulder. The pain began when he started spring training and has gradually worsened. In addition, the patient is now having difficulty lifting the arm above his head. He denies injury to the shoulder itself and states that ibuprofen provides some relief. On physical exam, the patient has tenderness to palpation of the lateral left shoulder just under the acromion, limited abduction of the left shoulder, and a negative 'drop arm' sign and 'crossover test'. Question What is the next appropriate step in the management of this patient? Answer Choices 1 Trial prescription of non-steroidal anti-inflammatory medications (NSAIDs) 2 Pain management with opiate analgesics 3 Prescribe a muscle relaxant 4 Magnetic resonance imaging (MRI) of the shoulder 5 Orthopedic referral for surgical therapy

Explanation The correct response is a trial prescription of non-steroidal anti-inflammatory medications (NSAIDs). The patient above is most likely suffering from rotator cuff tendinitis. Repetitive movements of the shoulder, such as throwing or swimming, can lead to inflammation of the tendons of the rotator cuff muscles. A patient with a rotator cuff tendinitis, in addition to an inability to lift the arm above shoulder level (abduction and external rotation), will have tenderness with palpation of the area just under the acromion on the affected side. Though the "drop arm" sign is used to assess the status of the rotator cuff, a positive drop arm sign indicates an actual complete tear in the rotator cuff, not just tendinitis. The patient in the presentation above had a "negative drop arm" test. For most rotator cuff disorders, rest from activities that increase pain is helpful. In addition, NSAIDs and moist heat can afford some relief, and they are usually the first-line treatment for these conditions. Corticosteroid injections and physical therapy can also be used in patients who do not get full relief from NSAIDs, or in those who have recurring issues. Opiate or narcotic analgesics are usually never indicated in the management of these inflammatory conditions. Generally, muscle relaxers are not helpful because the pathology of these conditions involve tendons, not muscles. Magnetic resonance imaging (MRI) is indicated in patients with persistent symptoms or in those who do not improve after months of conservative therapy, and it is used to rule-out rotator cuff tendon tears. Surgical repair by an orthopedic surgeon is usually reserved for patients who do not improve after months of conservative therapy or who have complete rotator cuff tears. However, depending on the patient's symptoms and functional status, even complete tears may not always require surgical repair.

A 34-year-old man presents for evaluation of 10/10 left calf pain after being run over by a car. Physical exam reveals tense calf swelling, increased pain with passive muscle stretching, and tenderness on calf palpation. Question What other findings are expected in this patient? Answer Choices 1 Pain out of proportion to exam 2 Normal calf compartment pressure 3 Erythematous streaks 4 A palpable calf cord 5 Groin lymphadenopathy

Explanation The correct response is pain out of proportion to exam. This patient presents with acute compartment syndrome after a crush injury. Compartment syndrome occurs when excessive muscle swelling increases the pressure within the fascial sheath compartment, compressing vessels and nerves. Compression of the artery leads to ischemia.1,2 Compartment syndrome is a medical emergency and can result in the loss of the limb if the pressure is not relieved immediately.2 Compartment syndrome is a clinical diagnosis based on "the 6 P's" (pain out of proportion, pressure on palpation, paresthesia, paralysis, pallor, and pulselessness).3 In addition to the 6 P's on PE, diagnosis is confirmed by obtaining the compartment pressure, in which anything above 10mmHg is abnormal. At 20mmHg, ischemia can occur, and anything above over 30mmHg requires an emergent fasciotomy.3 Streaking erythema is a sign of lymphangitis, which is inflammation or infection that has spread to the lymphatic system. A palpable calf cord can be appreciated in patients who present with a DVT or varicose veins. Lymphadenopathy is swelling of a lymph node secondary to infection, autoimmune disease, or malignancy.

A 15-year-old boy presents for a routine physical. He reveals a 1-month history of mildly painful swelling of the anterior superior left shin. It has been unaccompanied by fever, erythema, or joint complaints. He is his high school football team's tailback. He has been assisting his father in caulking the hull of their new boat, which entailed considerable kneeling that worsened the pain in the affected area. Examination is unremarkable except for mild, slightly tender swelling of the left anterior shin approximately 5 cm below the knee. Radiographs of the left knee showed mild irregularity of the tibial tubercle. Question What treatment recommendation is appropriate in this case? Answer Choices 1 Aspiration of the left knee joint 2 Bone biopsy 3 Rest, ice, wrapping, elevation, NSAIDs, physical therapy 4 Rheumatoid factor 5 Serial knee X-rays every 4 weeks

Explanation The diagnosis is Osgood-Schlatter disease (condition), which is an inflammation of the tibial tuberosity. Once the significant conditions are eliminated, treatment is supportive and symptomatic. This condition is not intra-articular; arthrocentesis is of no value. The condition is diagnosed clinically and radiographically. There is no need for a biopsy in the typical case. There is nothing in the presentation to suggest an actual arthritis or arthropathy. Repeat X-rays are of no value in the typical case and would be warranted only in the case of worsening physical findings and symptoms.

A 17-year-old male college freshman presents with chronic fatigue, back pain, and stiffness. The pain has been present for the past several months, but it appears to be worsening. The back symptoms are worse at night and first thing in the morning; they improve somewhat during the day. However, because he is always tired, he is having difficulty staying productive at school. Question Given the patient's presentation, what laboratory finding is most probable? Answer Choices 1 Decreased c-reactive protein 2 Increased b-natriuretic protein 3 Increased erythrocyte sedimentation rate 4 Increased white blood cell count 5 Increased uric acid level

Explanation The patient is presenting with signs and symptoms indicative of inflammatory arthritis; it is most likely ankylosing spondylitis (AS). Characteristic symptoms include back pain that is worst in the morning and improves with exercise. Back stiffness is also reported, as is extreme fatigue. AS is more common in male patients. As with other inflammatory arthritides, patients usually present at a younger age (18 - 35 years) relative to patients with mechanical causes of back pain or arthritis. AS is a clinical diagnosis, but many patients will have elevated erythrocyte sedimentation rates and c-reactive protein rates; both are nonspecific markers of inflammation. Elevated b-natriuretic protein is associated with congestive heart failure. An increased white blood cell count is usually a sign of acute infection; it may be seen in infective arthritides, but it is not a characteristic finding in cases of inflammatory spondyloarthropathy. Elevated uric acid levels suggest the diagnosis of gouty arthritis; however, some patients with this diagnosis will have normal uric acid levels.

A 27-year-old man is admitted to the hospital following a motor vehicle accident. He sustained lacerations to his arms bilaterally and has fractures of the right tibia and fibula. A cast is placed and the patient is scheduled for surgery the following day. A few hours after the cast is placed, he develops severe pain; the pain is unresponsive to several doses of intravenous morphine. His pain increases when he extends his right leg. Peripheral pulses are weak, but present. Question Based on the history and physical, what should be done next? Answer Choices 1 X-ray of the tibia and fibula 2 MRI of the right leg 3 Remove cast and check compartment pressure 4 Ultrasound of the lower extremity 5 Monitor and continue giving narcotics

Explanation The patient should have the cast removed and compartment pressure measured. This patient is showing signs of compartment syndrome. Compartment syndrome develops when there is accumulation of pressure within a muscle compartment. It typically occurs following trauma to a limb. Patients typically present with pain unrelieved by analgesia as well as pain with extension of the involved limb. As the swelling within the compartment worsens, patients develop absence of pulse, paresthesias, pallor, and poikilothermia. The compartment pressure should be checked, and if it is elevated, the patient should be taken for fasciotomy. An X-ray of the tibia and fibula would not demonstrate compartment syndrome. An MRI of the right leg would not be useful in this patient. An ultrasound of the lower extremity would be useful in diagnosing a lower extremity DVT. However, the patient's presentation is highly suggestive of compartment syndrome, and it should be ruled out before pursuing any other diagnoses. Monitor and continue giving narcotics is incorrect. If compartment syndrome is left untreated, it can lead to tissue death.

A 55-year-old retired professional football player presents to your office with bilateral knee pain. The patient reports a history of hypertension and peptic ulcer disease. The patient stands 6' 5" and weighs 325 lbs. On examination, you note bilateral knee joint effusions and crepitus on passive movement. There is also bony enlargement at the distal interphalangeal joints of the hands bilaterally. Lachman's test is negative on examination, and the X-ray report reveals joint space narrowing, osteophyte formation, and femoral subchondral cyst formation in both knees. Upon further questioning, the patient states that his knee pain has not limited his mobility yet. Question At this moment, what therapeutic recommendation would best prevent progression of this patient's disease? Answer Choices 1 Weight reduction 2 Anterior cruciate ligament repair 3 Oral prednisone 4 Total knee replacement 5 Aspirin 6 Calcium and vitamin D supplements

Explanation The patient's signs and symptoms are most consistent with the diagnosis of osteoarthritis. Weight reduction is the best therapeutic option because it will most likely have the greatest impact on slowing his disease progression. Crepitation over the area of osteoarthritis is a key characteristic of the disease. Classically, there is also bony enlargement with the osteophyte formation, and there is also narrowing at the DIP joints. Osteoarthritis typically affects weight-bearing joints, and occupational osteoarthritis is common in professional athletes, such as football players. Because our patient's mobility is still good, at this point there is no need to consider more invasive options. Anterior cruciate ligament repair is indicated in patients with ACL tears, which are often seen in patients who are athletes, especially those who play football. However, the negative Lachman's test suggests that the presence of an associated ACL injury is low. However, if an ACL injury is suspected clinically, this could be confirmed with an MRI examination of the knee. Oral corticosteroids have no place in the management of osteoarthritis. However, occasional intra-articular injections of corticosteroids may provide a temporary benefit in flare-ups and in the relief of symptoms in affected patients. Both methotrexate and tumor necrosis factor inhibitors are incorrect answers; they are treatment options for patients suffering from rheumatoid arthritis, not osteoarthritis.

The mother of a 3-year-old boy brings her child to the family doctor because the child was slow to begin walking, has difficulty in walking and standing, and fatigues easily. She recalls that the youngest brother of her mother had similar problems and died in his late teens. Tests of the child reveal creatinuria and elevated serum levels of creatine phosphokinase (CPK). A histological specimen taken from a muscle biopsy from this patient is shown in the image. Question What is the most likely diagnosis? Answer Choices 1 Nemaline myopathy 2 Becker muscular dystrophy 3 Duchenne muscular dystrophy 4 Myasthenia gravis 5 Mitochondrial myopathy

Explanation This child is suffering from Duchenne muscular dystrophy (DMD). This X-linked condition is maternally transmitted to male children only. It usually manifests in the first decade of life as difficulty in standing and walking. Muscle weakness initially appears in the muscles of the hip girdle and upper legs. Later, the condition involves muscles of the arms and respiratory system, with death usually occurring before age 20. Patients fail to produce the protein dystrophin. This is a member of the spectrin superfamily, with a MW = 427,000. Dystrophin is found underneath the sarcolemma in normal muscle, especially where membrane folds are formed, e.g., at myotendinous and neuromuscular junctions. It also interacts with actin filaments, stabilizing their side-side association as well as binding to the amino-terminus. Muscle from DMD patients shows myocytes of variable diameter (compared to the diameters of b and c) and unusual infiltration of leukocytes (at a). Physical symptoms of DMS occur most often before age 6 and include fatigue, learning difficulties and muscle weakness. It begins most commonly in the legs and pelvis area with progressive difficulty walking. Muscles of DMD patients are more fragile and more easily ruptured. In addition, DMD patients show reduced levels of enzymes normally found in the sarcoplasm with concomitant increases in serum levels of creatine phosphokinase (CPK), lactate dehydrogenase, and glucose phosphate isomerase. Urinary levels of creatine are usually elevated. One of the normal functions of dystrophin is stabilization of the sarcolemma against the physical stresses caused by frequent folding during cycles of contraction and relaxation.Myosin and myoglobin levels would be reduced due to loss on sarcomeres. Lysosomal hydrolases would be elevated because of increased activity from phagocytic cells. Becker muscular dystrophy is similar to DMD in its genetics and clinical course, although it is less common and less severe. In such instances, dystrophin is formed in lower quantities and has a slightly lower molecular weight than normal dystrophin. Nemaline myopathy is usually caused by autosomal recessive mutations that lead to accumulations of rod-shaped bodies just beneath the membrane of muscle cells. It probably represents mutations in the protein α-actinin, leading to bundling of Z-line proteins. Myasthenia gravis is an autoimmune disease wherein patients develop antibodies to the acetylcholine receptor of the neuromuscular junction. Routine histological biopsies of muscle tissue are apparently normal. It does not usually present in young children, but when seen in patients under 40, it is more common in women than in men. Mitochondrial myopathy usually presents in young adults initially as proximal muscle weakness, especially in the oculomotor muscles. The disease is commonly characterized by peripheral accumulations of mitochondria (demonstrable only by special stains) in muscle cells.

A 19-year-old boy presents with pain and deformity of his right dominant shoulder after a sudden jerking movement to the same from a wrestling competitor approximately 1 hour ago. He was unable to continue wrestling and has pain with any movement of the right shoulder. On exam you see a loss of normal shoulder contour anteriorly. There is no Acromioclavicular joint tenderness. What is the most likely diagnosis? Answer Choices 1 Rotator cuff tear 2 Shoulder separation 3 Glenohumeral dislocation 4 Fractured clavicle 5 Sternoclavicular subluxation

Explanation This is a glenohumeral dislocation (shoulder dislocation). With an acute dislocation there is considerable pain with any movement. There is loss of normal shoulder contour (more so with anterior dislocation rather than posterior dislocation). With a cuff tear there is gross weakness to resistance and gravity. Shoulder (AC) separation would have tenderness over the injury site as would a clavicle fracture or sternoclavicular subluxation.

Police bring in a 28-year-old male prisoner who was in a fight with another man 2 hours prior. During the fight, your patient struck the other in the mouth; he sustained a laceration over his 1st and 2nd knuckle of his right hand. A prompt initial inspection reveals a laceration that is 2.5 cm in length; there is no foreign body and the depths of the wound do not appear to involve bone or tendon. There is noted tenderness of the head of the first metacarpal bone, but no crepitus. There is no limitation to passive and active motion. The patient's last tetanus booster was 1 year ago. His physical examination, aside from his hand, is within normal limits. Question What should be the immediate course of action for this patient? Answer Choices 1 Refer a hand surgeon for a consultation 2 Copious irrigation of the wound 3 Discharge him into police custody on oral antibiotics 4 Initiate intravenous antibiotics as soon as possible 5 Do nothing else and discharge the patient

Explanation This patient's injury is sometimes called a clenched fist injury due to the position of the fist when the injury occurs. This injury may also be called a closed fist injury or fight bite. Although many occurrences of this type of injury do not initially appear severe, the overall consequences of this injury are very deceiving. The possibility of significant infection is an issue due to there being over 42 different organisms that one may be exposed to from a human bite. Another factor that makes this injury serious is the fact that many times the injury itself does not bleed heavily and the underlying injury is hidden by soft tissue. Just range of motion of the injured joint could seal the bacteria that may be introduced during the impact; severe complications can range from cellulitis, osteomyelitis, septic arthritis, or even tenosynovitis. Delay in seeking treatment will only worsen these complications. Treatment should begin promptly and consists of an extreme irrigation of copious amounts of sterile water or even an antiseptic solution. Typically this is done via a high pressure stream of water or solution into the wound. A prompt referral to a hand surgeon would then be appropriate. Incision and drainage of the wound may also be considered and splinting may occur for stabilization after proper evaluation and cleaning; bite wounds however should NEVER be sutured. Intravenous antibiotics may be administered for the first 24 hours. Once the patient has been discharged he is usually switched to oral antibiotics for 5 - 7 days. Antibiotic choice is dependent on those used by the hand surgeons in the community. Not treating the patient as described above would very highly likely lead to devastating consequences with significant morbidity.

You are evaluating a 34-year-old African American man for a 5-week history of increasing right groin pain. He denies any injury or history of similar pain. The pain is worse with movement and has progressed to the point that the patient has severe pain with weight bearing. He denies fever, chills, urinary symptoms, or any other complaint at this time. He has a past medical history of sickle cell disease and hypothyroidism. Physical exam reveals tenderness upon palpation of the groin with increased pain on both active and passive range of motion of the hip. Homan's sign is negative. Distal pulses and sensation are intact and normal. Question What diagnostics study should be ordered at this time? Answer Choices 1 Ultrasound of the right lower leg 2 CBC 3 MRI of the right hip with and without contrast 4 CXR 5 Urinalysis

Explanation This patient's presentation is concerning for avascular necrosis of the right hip. The most appropriate diagnostic test to order at this time is an MRI of the hip with and without contrast. Avascular necrosis, also known as osteonecrosis, is a focal infarct of the bone due to an interruption of blood supply.1,2 Avascular necrosis (AVN) most commonly occurs in the hips, jaw, and scaphoid bone of the hand.2 Risk factors for AVN include trauma, coagulopathy, alcoholism, chronic corticosteroid use, sickle cell disease, and auto-immune diseases such as RA and SLE.1,2 The differential of DVT does need to be considered in this patient; however, with the current clinical picture, it is unlikely a DVT is the primary cause of the patient's symptoms. While it is plausible this patient may have a DVT as well, ultrasound does not provide any information on the bone integrity and cannot diagnosis AVN. A CBC, CXR, and UA may be ordered as components of this patient's work up, but individually, these tests will not provide enough information to make a diagnosis.

A 32-year-old man presents due to low back pain. He reports a 3-year history of chronic low back pain that progressively worsened. He notes a lot of stiffness with any movements that involve his low back. He is especially stiff upon morning awakening and now has to ask his wife to help him get out of bed. He denies a history of trauma. He works in retail, with some bending and light lifting duties. He has taken acetaminophen repeatedly, even exceeding recommended doses, with no relief. He knows his paternal grandfather had some type of serious arthritis in his back, but he does not know the diagnosis. Other than his chronic back pain, the patient reports that he is in good health, with no chronic conditions. He takes no regular medications, except for the analgesics mentioned above. He has no allergies and has never had surgery. On physical exam, some limited range of motion is observed in his back and hips, and the patient reports discomfort with the motions. Otherwise, his exam is normal, with no visible edema, muscle wasting, or bony abnormality. You obtain a plain radiograph, and the report indicates a classic 'bamboo spine' appearance and some evidence of sacroiliitis. His sedimentation rate and c-reactive protein levels are elevated. Question What medication would be an appropriate first-line treatment for this patient's condition? Answer Choices 1 Acetylcysteine (Mucomyst) 2 Allopurinol (Aloprim) 3 Celecoxib (Celebrex) 4 Cyclobenzaprine (Flexeril) 5 Milnacipran (Savella)

Explanation This patient's presentation is consistent with a diagnosis of ankylosing spondylitis (AS), a progressive inflammatory arthritis that primarily affects the axial skeleton. NSAIDs are the primary first-line treatment. An NSAID, such as celecoxib (Celebrex), would be reasonable. The cyclooxygenase-2 (COX-2) inhibitors, such as celecoxib, have not been necessarily shown to have better efficacy, but are suggested to have fewer gastrointestinal side effects than traditional NSAIDs. Regular use of NSAIDs is recommended. Other potential treatments include medications such as methotrexate, sulfasalazine, and newer "biologics" like infliximab, etanercept, and adalimumab. Acetylcysteine (Mucomyst) is a medication used for acetaminophen overdose. The patient's current symptoms do not suggest acute acetaminophen toxicity. Allopurinol (Aloprim) is a medication used primarily for gout. While gout can cause joint pain, this patient's presentation is not consistent with gout. Cyclobenzaprine (Flexeril) is an older muscle relaxant medication. Although the patient reports "stiffness", his symptoms are due to joint damage, not muscle spasm. This medication would not help his AS. Milnacipran (Savella) is a medication approved for fibromyalgia. While chronic pain are found in both AS and fibromyalgia, none of this patient's diagnostic studies are consistent with fibromyalgia.

A 36-year-old G1P1001 woman presents with a 4- to 5-week history of pain along her right lateral wrist; the pain worsens when she tries to grasp something. There was no trauma. She gave birth 6 weeks ago, and carrying her baby is difficult secondary to pain. She is right handed, and she has never had any issues like this before. On exam, there is tenderness and edema over the radial styloid. You then have the patient fully flex her thumb, adduct, and grasp it with that hand. You then place her hand in ulnar deviation, which re-produces the pain described above. Question What is the most likely diagnosis? Answer Choices 1 Rheumatoid arthritis 2 Ganglion cyst 3 Scaphoid fracture 4 de Quervain tenosynovitis 5 Radial nerve entrapment

Explanation de Quervain Tenosynovitis, which is the correct response, is a tenosynovitis of the abductor pollicis longus and extensor pollicis brevis tendons at the radial styloid process. It is most commonly seen in women ages 35 to 55. It can happen when women carry children with their thumb overextended. On exam, there can be tenderness and swelling over the radial styloid. There is also a positive Finkelstein test, which is what is described in the scenario. Rheumatoid arthritis is an incorrect response; it usually affects the small bones in the hands and is bilateral. Patients have morning stiffness. Ganglion cyst is an incorrect response; frequently, patients present with a lump in the wrist, which is inconsistent with the scenario. Scaphoid fracture is an incorrect response; our patient did not have tenderness over the scaphoid bone. She also did not have any trauma. A scaphoid fracture most commonly occurs by falling on an outstretched hand. Radial nerve entrapment is an incorrect response; it usually causes burning pain and paresthesia. A positive Tinel's sign would be seen.

A 32-year-old Caucasian woman presents with pain, stiffness, fatigue, and swelling of the fingers. These symptoms have been present over the last 6 months. Over period of several visits you rule out any neuromuscular abnormalities. Laboratory work includes complete blood count to rule out infectious diseases. Erythrocyte sedimentation rate is mildly elevated. Thyroid function studies are normal. On physical exam, you note bilateral tenderness to palpation of more than 12 points, including the upper edge of the trapezii, neck muscle insertion at the occiput, deltoids, infraspinatus, and second costochondral junction, inferior to the lateral (elbow) epicondyles, trochanters, and lower lumbar area, medial fat pads of the knees, and the medial and lateral insertions of the Achilles tendons reveal 12 or more tender points. What is the most likely diagnosis? Answer Choices 1 Cellulitis 2 Fibromyalgia 3 Gout 4 Polymyalgia Rheumatica 5 Reiter's syndrome

Explanation Fibromyalgia is a soft tissue, non-articular pain disorder characterized by chronic, generalized musculoskeletal aches, pains, and stiffness that occur primarily in muscles and their attachments. It is associated with specific sites of exaggerated tenderness. Palpation of the upper edge of the trapezii, neck muscle insertion at the occiput, deltoids, infraspinatus, and second costochondral junction, inferior to the lateral (elbow) epicondyles, trochanters, and lower lumbar area, medial fat pads of the knees, and the medial and lateral insertions of the Achilles tendons reveal 12 or more tender points. First treatment is patient education. The client will feel better just knowing they have a diagnosable syndrome that can be managed. The use of antidepressant drugs can help. Drugs like amitriptyline, fluoxetine, chlorpromazine, or cyclobenzaprine are used. Exercise is another important aspect of treatment. A healthy diet can also help the client feel in control of their condition. Corticosteroids and opioids should not be used and are not helpful. Reiter's syndrome refers to a spectrum of reactive arthritis (ReA) that is not consistent with other spondyloarthropathies. It is a triad of arthritis, nongonococcal urethritis, and conjunctivitis. The syndrome characteristically is an acute, asymmetric, arthritis of the lower extremities that may involve the sub talus or present a dactylitis (sausage digit). It may be isolated to joints or include systemic illness, characterized by fatigue, fever, and weight loss. Classic symptoms accompanying, or intercurrent manifestations, involve the following three systems: Mucutaneous disease ranging from painless oral or genital ulceration to a keratoderma blenorrhagicum, a psoriatic skin lesion typically on the palms and soles. Urethritis or cervicitis (including prostatitis and salpingitis) Ocular manifestations ranging from sporadic conjunctivitis to debilitating uveitis Myalgia, joint stiffness, and low back ache. Most cases are positive for HLA B27, except HIV patients. Gout occurs most commonly in males (95 percent). Presentation is usually acute with a swollen, red, big toe. Females may develop gouty arthritis in the perimenopausal phase as estrogen levels drop. In many cases, if the disorder is left untreated, urate crystals are deposited, usually in and around the joints of the extremities deposited, usually in and around the joints of the extremities. The needle-shaped monosodium urate (MSU) crystals, can be deposited in the parenchyma of organs such as the kidney. In some clients, deposits of uric acid form within the urinary collecting tubules. Cellulitis is invasion of the skin, its appendages, and/or subcutaneous layer by way of thermal, mechanical, chemical, or physiologic injury and the colonization, or infection, with pathogenic bacteria of both aerobe and anaerobic nature. Lesions are usually due to immune system response, and purulence is present.

If the musculocutaneous nerve were to become damaged, what function would be lost? Answer Choices 1 Forearm flexion 2 Forearm extension 3 Wrist flexion 4 Wrist extension 5 Pronation

Forearm flexion Explanation The major muscles innervated by the musculocutaneous nerve are the brachialis and biceps brachii. Therefore, forearm flexion would be lost. Forearm extension is controlled by the triceps brachii which is innervated by the radial nerve. The wrist flexors are innervated mostly by the median and ulnar nerves. Wrist extensors are innervated by the radial nerve. The pronators (pronator quadratus and pronator teres) are innervated by the anterior interosseous and median nerves.

A 54-year-old man presents to your clinic with a 2-month history of left non-dominant shoulder pain. There is no history of trauma, but the pain began about a week after shoveling wet heavy snow from his 100-foot driveway. At first the pain seemed to come and go from day to day, but has gotten progressively worse and more constant. Pain is worse with overhead use and he can't sleep on his left side and will wake up if he rolls over onto his left shoulder. You diagnose the patient with Impingement Syndrome. Which of the following physical exam findings led you to your diagnosis? Answer Choices 1 Tenderness with palpation of the AC (acromioclavicular) joint 2 Positive (+) drop arm test 3 Positive Neer and Hawkins signs 4 Positive sulcus sign 5 Edema of the shoulder muscles

Explanation Neer and Hawkins signs are positive in impingement syndrome. Drop arm test is positive in rotator cuff tear. Tenderness is over the greater tuberosity of the proximal humerus and not the AC joint (unless associated with arthritis of the AC joint). Sulcus sign is seen with glenohumeral joint instability. Atrophy, not edema, is common with this disorder.

A 45-year-old female patient is seen in your office and complains of chronic dry mouth and dry eyes. She tells you that she has difficulty shedding tears and is very tired. Upon examination of her mouth, you note apparent decay in many of her teeth. This patient is likely to have which of the following diseases? Answer Choices 1 Osteoporosis 2 Diabetes mellitus 3 Sjögren's syndrome 4 Lupus erythematosis 5 Scleroderma

Explanation Sjögren's syndrome is an autoimmune disorder that is characterized by chronic dry mouth, dry eyes, vaginal dryness, skin dryness, and fatigue. The lack of saliva often results in rampant tooth decay. Sjögren's syndrome often goes undiagnosed, being attributed to "natural" changes related to aging and menopause. 90% of the 200,000 Americans diagnosed with Sjögren's syndrome are women. However, the disease also attacks men. There is no cure for this disease, but several treatments can alleviate some symptoms. These include the use of artificial tears and saliva to treat dryness and nonsteroidal anti-inflammatory drugs and corticosteroids to treat inflammation. Maintaining an exercise program can also help.

Question A 54-year-old man presents with a 2-month history of left non-dominant shoulder pain. There is no history of trauma, but the pain began about a week after shoveling wet heavy snow from his 100-foot driveway. At first the pain seemed to come and go from day to day, but it has gotten progressively worse and more constant. Pain is worse with overhead use and he can't sleep on his left side and will wake up if he rolls over onto his left shoulder. What diagnostic tool can be utilized to help with the diagnosis confirmation? Answer Choices 1 AP and Transscapular lateral plain radiograph 2 A subacromial injection of 10 ml of 1% Xylocaine 3 MRI 4 CT 5 EMG

Explanation This is rotator cuff tendonitis/subacromial bursitis/impingement syndrome. A simple subacromial injection of 10 ml of 1% Xylocaine will numb the inflamed tendon/bursa and strength will be normal again as the injected anesthesia takes effect and decreases the associated pain of inflammation (if no cuff tear is associated with it). Plain radiographs are usually normal in any of these disorders. MRI may help rule out a cuff tear but is less sensitive for tendonitis. CT is good for bony pathology mostly and less with soft-tissues. EMG is used for nerve study.

An elderly woman sustained a fracture of the proximal humerus 4 days ago while she was out of town visiting her daughter. She was seen in an Urgent Care clinic there and was told to follow up with her doctor when she got home. They placed her arm in a shoulder immobilizer and told her to leave it on. She was and is neurovascularly intact distal to her fracture. She comes to your office now for further care. What physical exam finding should be expected after 3 or 4 days? Answer Choices 1 Further deformity of the fracture 2 Increased pain from the fracture 3 Gross edema of the hand 4 Large ecchymotic area 5 Compartment syndrome of the forearm

Explanation With a proximal humerus fracture in an elderly woman (which is common), gross ecchymosis will be present between 2 and 4 days post-injury. It may even travel into her upper chest and breast tissue. Since she was placed in a shoulder immobilizer, further deformity is unlikely unless she had further injury, such as a fall onto the arm. Although edema is associated with this fracture, it will stay in the arm near the fracture site, especially since she was splinted. Also since she has been splinted, the pain should be less since movement is restricted. Compartment syndrome is possible in the arm compartment but not likely in the forearm compartment.

A 22-year-old Asian man with no significant past medical history presents with a dull pain. It is insidious in onset and has been felt deep in the lower lumbar gluteal region for the past 5 months. It is accompanied by low-back morning stiffness of up to a few hours' duration. The stiffness improves with activity and returns following inactivity. For the past 2 months, the pain has ascended; it has become more persistent and bilateral. He notes that nocturnal exacerbation of pain forces him to rise and move around. He also admits to bilateral hip and ankle pain, anorexia, malaise, and an intermittent low-grade fever. His physical exam reveals a stooped forward-flexed position, limitation of anterior and lateral flexion and extension of the lumbar spine, a positive Schober test, restricted ranges of motion of his hips, and reduced chest expansion. A lower back and hip X-ray is performed. It yields the attached image. Question What additional clinical manifestation would be most likely in this patient? Answer Choices 1 Eye pain and conjunctival injection 2 Hematuria and dysuria 3 Positive straight leg raise test 4 Subcutaneous nodule formations 5 Warm and swollen hip joints

Eye pain and conjunctival injection The correct response is eye pain and conjunctival injection. This patient's most likely diagnosis is ankylosing spondylitis (AS). The most common extra-articular manifestation is acute anterior uveitis, which occurs in 40% of patients and can antedate the spondylitis. Attacks are typically unilateral, causing pain, photophobia, and increased lacrimation. These tend to recur, often in the opposite eye. Cataracts and secondary glaucoma are not uncommon sequelae. Up to 60% of patients have inflammation in the colon or ileum. This is usually asymptomatic, but frank IBD occurs in 5 - 10% of patients with AS. Hematuria and dysuria suggest a genitourinary pathology. A positive straight leg raise test indicates a likelihood of her herniated lumbar disc. Subcutaneous nodule formation in the setting of joint pain and stiffness suggest rheumatoid arthritis. This patient however is diagnosed with a spondyloarthropathy. Peripheral enthesitis occurs in approximately 33% of patients. These lesions are painful and tender upon examination and may be associated with swelling of the tendon or ligament insertion. Warmth of joints is not typical. Additionally, the most common and characteristic peripheral sites of enthesitis are the insertion of the Achilles tendon on the calcaneus and the insertion of the plantar fascia on the calcaneus. Warm and swollen joints are more typical of rheumatic, crystal-induced arthropathy or joint infection.

A 4-year-old girl presents after falling on an outstretched right dominant arm with her elbow fully extended from a slide (about 4 feet fall) 30 minutes ago. There was no loss of consciousness, but there was extreme pain, and she immediately grabbed her right forearm and her right elbow. There was no obvious deformity at the elbow. What findings on radiographs suggest a fracture? Answer Choices 1 A tear-drop sign 2 A fat-pad sign 3 An open physis 4 A radial head aligned with the capitellum 5 An olecranon process in the olecranon fossa

Fat pad sign A positive (+) fat-pad sign, usually associated with an occult fracture in recent injuries, is indicative of bleeding into the capsule of the elbow. Anterior fat-pad signs may be normal. Posterior fat-pad signs are never normal and indicate bleeding with a history of trauma to the arm/elbow. An open physis just indicates a growing child. A tear-drop sign, seen in displaced fractures, is what should be seen in the normal lateral view of the distal humerus. A radial head should align with the capitellum, and the olecranon process fits into the olecranon fossa.

A 61-year-old Caucasian man presents with upper extremity pain. He describes the pain as coming on gradually, limiting the movement of his arm. He states there was no trauma, or past history of trauma. The pain just started and is gradually getting worse. What might he have? Answer Choices 1 Overuse 2 Frozen shoulder 3 Subluxation 4 Fibrositis Syndrome 5 Subacromial bursitis

Frozen shoulder Explanation Suspect a frozen shoulder, also known as adhesive capsulitis, if the pain comes on gradually, limiting the motion at both the glenohumeral and scapulothoracic joint. Often there will be limitation of abduction, external rotation, and forward flexion. The hallmark of frozen shoulder is a spontaneous onset of pain that worsens gradually and insidiously, followed by loss of range of motion. The pain may radiate, but pain below the elbow is not common. Acute subacromial bursitis, which is painful enough to cause marked limitation of movement, happens more often in younger clients. The point of tenderness is often over the subacromial bursa, along with the swelling and warmth. This is noticed most when the client is lifting objects and combing their hair. Tendinitis or tenosynovitis in the rotator cuff, biceps, or biceps tendon sheath often precedes subacromial bursitis. The pain following a severe trauma to the shoulder may come from a subluxation or dislocation. Often the client will state that the shoulder just doesn't feel right, something popped, or something is out of place. Fibrositis syndrome is a form of rheumatism, which can cause pain in multiple locations, including the upper extremity. Some doctors do not believe these syndromes exist. Others have demonstrated that there is exaggerated, reproducible tenderness at specific anatomic locations in clients with this condition. Overuse syndromes may involve the elbow. Muscle strains may be vocational or recreational due to weight lifting, throwing, or racquet sports. There is pain and acute tenderness at the origin of the extensor muscles of the forearm, which attach just distal to the lateral epicondyle of the humerus. Overuse syndromes can include a joint that is used in a repetitive motion over a period of time.

A 46-year-old female presents with pain to her left wrist. She complains that it is painful and swollen as she points to the volar aspect of the wrist on the radial side. On examination, there is a small, soft bump on the dorsum of her wrist with a jelly-like consistency. What is the most likely diagnosis? A Cancerous tumor B Fracture C Ganglion cyst D Hematoma E Lipoma

Ganglion Cyst C Ganglion cysts commonly occur on the dorsal or volar aspect of the wrist. They result when a joint capsule or tendon sheath is damaged, allowing synovial fluid to escape producing a one-way valve, which allows fluid into the cyst, but not back out. The accumulating fluid forms the ganglion cyst. These cysts may or may not be tender and can fluctuate in size depending on activity level of the affected extremity. Cancerous tumors would tend to be much more firm, but also may be relatively pain free. Fractures would generally be exquisitely tender and if the bump is due to a displaced bone, it would be much more firm than a ganglion cyst. Hematomas are generally associated with acute trauma and would be tender and ecchymotic in many situations. Lipomas are benign fatty tumors that are more commonly seen on the thenar eminence than the dorsum of the wrist and their size does not change based on activity level.

A 33-year-old woman presents with joint pain and morning stiffness that lasts more than an hour. Her past medical history is significant for postpartum depression 2 years ago. On examination, you find her body mass index (BMI) is 18 and she displays signs of symmetrical small joint inflammation. You are waiting for X-rays and labs to confirm the diagnosis of rheumatoid arthritis. In the meantime, she is asking about risk factors, particularly the possibility of worsening her joint condition if she becomes pregnant again. Question What is the primary risk factor in this patient's case? Answer Choices 1 Age 2 Gender 3 Depression 4 Body mass index 5 Pregnancy

Gender The correct response is gender. Rheumatoid arthritis affects women 3 times more often than men. Although it may present at any age, rheumatoid arthritis is most frequently diagnosed between the ages of 40 and 50. This patient's age is not a risk factor for the disease. Depression is usually a consequence of the arthritis, not its cause. Arthritis is more common in obese patients; this patient is underweight (low BMI). Pregnancy is not a risk factor for rheumatoid arthritis; it actually has ameliorating effects on the disease activity.

A 43-year-old woman presents after being diagnosed with breast cancer 2 years ago. Tests have indicated that the cancer has spread. You discover, upon examination, that she has a visibly protruding vertebra. What term best describes this deformity? Answer Choices 1 Scoliosis 2 Scoliokyphosis 3 Kyphosis 4 Lordosis 5 Gibbus

Gibbus Explanation Gibbus is an angular deformity of a collapsed vertebra. The deformity may be protruding outward and may be caused by metastatic cancer or tuberculosis of the spine. Scoliosis is the lateral curvature of the spinal column. This can be functional or structural when it compensates for other abnormalities. Scoliokyphosis is the combined lateral and posterior curvature of the spine. Kyphosis is the posterior (backward) curvature of the spine as viewed from the side. It is a rounded thoracic convexity. This is common in aging and in women. Lordosis is the curvature of the lumbar spine. It is the accentuation of the normal lumbar curve. It may exist as a compensation for other deformities, i.e., kyphosis.

Upon testing a patient for function of the hip extensors, which muscle is considered the primary muscle responsible for most extension? A Gluteus maximus B Pectineus C Semimembranosus D Semitendinosus E Vastus lateralis

Gluteus Maximus The gluteus maximus is a large muscle that is partially responsible for giving shape to the buttocks. It is the dominant muscle responsible for hip extension. It is easily palpable with a patient in the prone position with buttocks squeezed together or with the hip extended and the knee flexed. The pectineus muscle is considered a secondary hip adductor. The Semimembranosus and Semitendinosus are two of the three hamstring muscles. They are primary movers in knee flexion, but only secondary contributors to hip extension. Vastus lateralis is one of the four quadriceps muscles and plays a role in knee extension, but not hip extension.

A 48-year-old man is awakened from his afternoon nap by an agonizing pain at the base of his big toe on the right side. On examination the first metatarsal phalangeal joint is swollen, red, tender, and warm to touch. The patient also appeared to have an inflammed knee and as a result, a decision was made to tap the right knee rather than the first metatarsal joint. Analysis of the fluid from the right knee shows: Leukocytes 68,000/mm3 Neutrophils 75% Crystal analysis negatively birefingent Gram stain negative What is the most likely diagnosis? Answer Choices 1 Gout 2 Osteoarthritis 3 Reactive arthritis 4 Rheumatoid arthritis 5 Septic arthritis

Gout Gout is an inflammatory arthritis associated with hyperuricemia. Uric acid crystals get deposited in the joint. These crystals appear as needle-shaped, negatively birefringent crystals. It is commonly seen in middle-aged men. Onset is sudden with agonizing pain, swelling, and redness of the joint. There is often a family history of gout. In osteoarthritis, as well as rheumatoid arthritis, there would not be crystals in neutrophils in the joint fluid. In septic arthritis there would be bacteria, but no crystals in the fluid. Reactive arthritis is a sterile synovitis, which occurs 1 to 2 weeks after an infection. In this case, too, there are no crystals in the fluid.

A 28-year-old man comes to see you at an urgent care clinic first thing in the morning. He notes a left ankle sprain. The patient describes walking in the woods the day before while wearing flip-flops; he inadvertently stepped into a hidden hole. The patient immediately noticed extreme pain with weight bearing on of the left ankle, making it extremely difficult for him to walk out of the woods. Ambulation is still extremely difficult at this time. He describes it as an 8 - 9/10 on a 1 - 10 pain scale. The patient admits to having ankle sprains before during his life, but this one is much more severe and debilitating. He has noted significant swelling and exquisite tenderness to touch. He has treated this with elevation, ice for 20 minutes at a time, and ibuprofen 200 mg every 8 hours. Physical examination reveals a moderate degree of ecchymosis of the left ankle, with substantial mechanical instability, and moderate restriction of range of motion. Question Based on the history and physical examination findings, how would you classify this ankle sprain based on the traditional ankle sprain grading system? Answer Choices 1 Grade 0 2 Grade I 3 Grade II 4 Grade III 5 Grade IV

Grade 2 The scenario above is describing the Grade II ankle sprain. This injury typically involves an incomplete tear of a ligament. Patients will experience moderate pain, swelling, tenderness and ecchymosis. There will be mild to moderate joint instability during exam and some restriction of the range of motion as well as loss of function. Ambulation and weight bearing are painful. Grade I ankle sprain results from only mild stretching of a ligament with potentially microscopic tears. Patients will present with mild swelling and tenderness; there will not be evidence of joint instability, and the patient will be able to bear weight and ambulate fairly easily. Grade III ankle sprain involves a complete tear of a ligament. There will be severe pain, swelling, tenderness, and ecchymosis. Significant instability will be seen on exam, as well as loss of function, with the inability to bear weight or ambulate whatsoever. Grades 0 or IV do not exist in the common grading system of ankle sprains.

A 6-year-old boy falls off a ladder and lands on his arm. He is in a lot of pain, so his parents bring him for an examination where some abrasions and swelling in his arm are noted. An X-ray reveals an incomplete break in the boy's forearm. What type of fracture is most likely? Answer Choices 1 Comminuted 2 Depressed 3 Greenstick 4 Oblique fracture 5 Simple

Greenstick Explanation A greenstick fracture is an incomplete break in the bone with bending of the bone. It is named as if you were to try and break a live green twig, but are able to break it only partially. Greenstick fractures occur more commonly in children. A simple fracture is a complete break in the bone that does not break the skin. This is also called a closed fracture. A depressed fracture is when there is inward displacement of the bone. An oblique fracture is a fracture that runs diagonally against the long axis of the bone. A comminuted fracture is when the bone has broken into many (more than two) pieces.

A 23-year-old Caucasian man presents with lower back pain. The pain has been occurring for several months. The pain is worst in the morning and improves with activity. He has associated stiffness. There is no known injury to the area, and the pain has not responded to rest and heat. He has also noticed blurred vision and photophobia. On examination, there is decreased range of motion in the back. Uveitis is noted. Question Based on the patient's findings, which laboratory study would likely be positive? Answer Choices 1 Anti-CCP antibodies 2 HLA-B27 3 Rheumatoid factor 4 Anti-nuclear antibodies 5 Anti-phospholipid antibodies

HLA-B27 is correct. The patient has lower back pain with uveitis, which is suggestive of ankylosing spondylitis. HLA-B27 is positive in 90% of Caucasian patients and 50% of African-American patients with ankylosing spondylitis. Anti-CCP antibodies and rheumatoid factor are incorrect. Anti-CCP antibodies and rheumatoid factor are found in rheumatoid arthritis and will be negative in ankylosing spondylitits. Anti-nuclear antibodies is incorrect. Anti-nuclear antibodies are negative in ankylosing spondylitits and are found in conditions such as systemic lupus and scleroderma. Anti-phospholipid antibodies is incorrect. Anti-phospholipid antibodies are found in anti-phospholipid syndrome and would not be present in ankylosing spondylitits.

A 48-year-old African American woman presents with a concern about osteoporosis. Her elderly father-in-law recently suffered a fractured hip, which was due to severe osteoporosis. The patient has been having intermittent bilateral hip pains and is worried that she also has osteoporosis. She gets the hips pains more with prolonged standing, walking and heavy lifting. At today's exam, she reports the pain as mild (1 out of 10, on 1 - 10 scale). A summary of her past medical history is shown in the table: Medications: Fluticasone/salmeterol (Advair HFA) inhaler daily; prednisone, for current asthma exacerbation, (uses 2 - 3 times per year) Medical history:Asthma, mild osteoarthritis OB/GYN history:G2P2. Has regular menses. Social history: Patient denies use of tobacco, alcohol, and illicit drugs. She is divorced, with 2 children at home. She walks for exercise and works in a potato processing facility, which involves frequent lifting of moderate weight. Family history: Positive for breast cancer in patient's mother; hypertension in her father; asthma in 1 brother. Question What aspect of this patient's history puts her most at risk for development of osteoporosis? Answer Choices 1 Her age 2 Her current symptoms 3 Her ethnicity/race 4 Her family history 5 Her medication history 6 Her social history

Her medication history Explanation This patient's medication history is the most worrisome risk factor for development of osteoporosis. Prolonged and/or frequent use of corticosteroids, such as this patient's current and frequent use of prednisone, is associated with decreased bone density and development of osteoporosis. Typically, the inhaled steroids (such as her fluticasone) or nasal steroids deliver a minimal systemic dose and are not generally associated with osteoporosis. The patient's age is not a risk factor for osteoporosis. Increased age (and postmenopausal status for women) is associated with increased risk of osteoporosis. The patient's current symptoms of mild hip pain are not associated with osteoporosis. Typically, patients with osteoporosis are asymptomatic, even with fairly severe disease. Ethnic and racial groups shown to be highest risk for osteoporosis are Caucasians, Asians, and Native Americans. African-Americans seem to have a lower risk; research has not yet determined the reason. The patient's family history is negative for osteoporosis. The other conditions in her family history are not linked with heritable osteoporosis risk. The father-in-law with osteoporosis does not confer any genetic risk to this patient. The patient's social history is negative for alcohol and tobacco use. Both excessive alcohol use and tobacco use are risk factors for osteoporosis. Weight-bearing activity appears to improve bone mineral density, so this patient's physical activity also reduces, rather than increases, her risk for osteoporosis.

A 57-year-old woman presents with painful distal phalangeal joints (DIPJ) in both hands. Past medical history is positive for a 10-year history of arthritis that is treated with NSAIDs. On clinical exam, both hands present with enlargements of the DIPJs that are not warm or red, but are tender to palpation. What are these enlargements called? Answer Choices 1 Heberden's nodes 2 Bouchard's nodes 3 Mucinous cysts 4 Gout 5 Inflamed bursae

Herberden's nodes Explanation Heberden's nodes are detectable bony enlargements of the distal phalangeal joints (DIPJ) of the hands. The joint may be limited in function and tender to palpation. Bouchard's nodes are bony enlargements of the proximal phalangeal joints (PIPJ). The joint may be limited in function and tender to palpation. A mucoid (mucinous) cyst arises from the joint capsule in the distal or proximal phalangeal joints. It contains degenerative myxomatous fibrous tissue from degenerative arthritis affecting the joint. Gout generally affects the joints of the foot (primarily the 1st metatarsophalangeal joint). It appears as a swollen, highly painful joint that is warm to the touch. The joint aspiration shows monosodium urate crystals. An inflamed bursa is secondary to a trauma to joints associated with a bursa. Some of the joints that can be affected are the elbow, the shoulder, and the 1st metatarsophalangeal joint.

A 36-year-old woman presents with a 1-week history of flu symptoms. After you evaluate and treat her presenting issue, she asks about color change in her hands. For the past several months, her hands have been intermittently turning white and then blue. The fingers become red, and then they tingle and burn. She is concerned about these episodes. She feels that they primarily occur during periods of stress. The patient has a family history of diabetes (mother) and heart disease (father). She has hypertension, for which she takes 50 mg of atenolol daily. The patient's hands currently appear normal, but she is not having symptoms. Question What is the most likely underlying cause for this condition? Answer Choices 1 Hypothyroidism 2 Idiopathic 3 Medication effect 4 Peripheral vascular disease 5 Rheumatoid arthritis

Idiopathic Raynaud's disease, syndrome, or phenomenon, named for the French physician who first described the process in 1862, results from intermittent spasm of the arterioles, which leads to a disruption in blood flow and resultant color change - either blanching (white) or cyanosis (blue). The condition usually involves the vessels of the fingers and toes, but it can also involve the lips, nose, ears, and nipples. Primary Raynaud's, or Raynaud's disease, is idiopathic; it is the most common type, and it tends to be less severe than secondary Raynaud's. Raynaud's phenomenon, which occurs secondary to a specific disease or condition, is also called Raynaud's syndrome. Risk factors for primary Raynaud's are female gender, age less than 30, family history, and living in a cold climate. Risk factors for secondary Raynaud's include age (generally older than 30), existence of a medical condition (autoimmune disease, atherosclerosis, pulmonary disease, or thyroid disease), history of injury to the affected body part, exposure to chemicals, cigarette smoking, and living in a cold climate. Raynaud's is often triggered by cold or stress. Secondary conditions that may lead to this process include thyroid disease, autoimmune disease (rheumatoid arthritis, polymyositis and dermatomyositis, Sjögren's syndrome), atherosclerosis, polycythemia, and thyroid disease. Certain medications can also cause Raynaud's phenomenon. These include birth control pills, beta-blockers, migraine, and cancer medications. Some over the counter medicines and diet aids can cause it, as well. The diagnosis is made clinically, although there is a test involving nail fold capillaroscopy that may be positive in the setting of connective tissue disease. Most cases do not require treatment. Common medications used if treatment is indicated include calcium channel blockers, alpha-blockers, and nitroglycerin. While all the choices of causal conditions for this patient's Raynauds are correct, the most common cause is idiopathic (primary Raynaud's). A workup for underlying causes may be indicated.

A 43-year-old woman presents with stiffness and pain in her fingers. It takes her about an hour in the morning to be able to use her hands. The symptoms started approximately 3 months ago, and they have gradually worsened. Recently, she also realized that some of her finger joints are swollen. She has a 2-year history of knee pain when climbing stairs. The patient's medical history includes hypertension, hyperlipidemia, and coronary artery disease. Her current medication is aspirin (81 mg daily), celecoxib (600 mg daily), glucosamine (500 mg tid), niacin (3 g/day), simvastatin (10 mg qh), vitamin E, and a multivitamin supplement. Vital signs are temperature 98.5° F, blood pressure 145/85 mm Hg, pulse rate 80/min, and respiratory rate 20/min. Physical examination reveals a well-developed, well-nourished woman in no apparent distress. Physical examination is unremarkable except for swelling around the metacarpophalangeal and proximal interphalangeal joints of the 2nd to 4th fingers bilaterally. There are no skin alterations. Laboratory values are as follows: white blood cell count 26,900/mm3; hemoglobin 14.9 g/dL; hematocrit 44.4%; platelet count 152,000/mm3; and CRP 29 mg/L. X-rays taken of both hands and knees show juxta-articular osteopenia, bone erosions, and loss of articular cartilage. Question What is the most logical next step? Answer Choices 1 IgM RF (rheumatoid factor) 2 MRI of affected joints 3 Ultrasound of affected joints 4 HLA-B27 5 Electrophoresis

IgM RF Explanation A positive rheumatoid factor (RF) alone does not necessarily establish a diagnosis of rheumatoid arthritis. However, in combination with the symptoms, its presence is significant for the prognosis. Patients with high titers tend to have a more severe and progressive disease. 5% of the healthy population has a positive rheumatoid factor. It can be present in chronic liver disease, infectious mononucleosis, Sjögren's syndrome, SLE, tuberculosis, pulmonary fibrosis, leprosy, syphilis, visceral leishmaniasis, malaria, schistosomiasis, and subacute bacterial endocarditis. It also can be positive transiently in people after vaccination or transfusion. Ultrasound and MRI are diagnostic measurements if X-rays, which should be taken first, are unremarkable. Both methods can show incipient development of pannus as well as erosions around the bone-cartilage area, which is important for early diagnosis. In the early stages, X-rays only reveal evidence of tissue swelling and joint effusion. Within weeks of onset, juxta-articular bone demineralization can be seen. HLA-B27 is a histocompatibility antigen. It is found more often in Caucasians, and it is linked to diseases such as Reiter's syndrome and ankylosing spondylitis. Patients with rheumatoid arthritis are commonly HLA-DR4 positive. An electrophoresis would be indicated if scleroderma were suspected, since half the patients with that disease show hypogammaglobulinemia.

A 26-year-old African American woman presents with a 10-month history of recurrent, migratory joint pain, which has been known to occur in her knees, hips, and elbows. More recently, she has been noticing hair loss, low-grade fever, blurry vision, increased fatigue, weight loss, and anorexia. She also notes that her fingers turn pale and blue when she is "stressed out" and upon exposure to the cold weather. She denies a travel history, sick contacts, or a history of diabetes, thyroid dysfunction, or menstrual irregularities. She also denies chills, a change in diet, swollen glands, otalgia, sore throat, chest pain, cough, shortness of breath, abdominal pain, nausea, vomiting, or diarrhea. Her physical exam reveals normal vital signs. She has patchy alopecia and conjunctiva that are injected but without discharge. The lower extremity joints are tender, with mild erythema and effusions noted. The remainder of her exam is within normal limits. Laboratory results were remarkable for pancytopenia, normal thyroid and hepatic function, elevations in serum creatinine, a positive antinuclear and antiphospholipid antibodies, elevated rheumatoid factor, and red blood cell casts in the urine. Question What would be considered an inappropriate health care maintenance recommendation for this patient? Answer Choices 1 Increased exposure to sunlight will reduce cutaneous manifestations of this disease. 2 She should be aggressively screened and treated for coronary heart disease risk factors. 3 Compliance to anticoagulants is essential to prevent her higher risk of thrombosis. 4 Annual Influenza and pneumococcal vaccinations should be given every 5 years. 5 Preventive screenings for cervical cancer should be diligently pursued.

Increased exposure to sunlight will reduce cutaneous manifestations of this disease. This patient's most likely diagnosis is systemic lupus erythematosus with the antiphospholipid syndrome. Patients with SLE should be cautioned against sun exposure and should apply a protective lotion to the skin while out doors, since associated rashes may be exacerbated by sunlight. In later years, accelerated atherosclerosis, linked to chronic inflammation, becomes a major cause of death. The incidence of myocardial infarction is 5 times higher in persons with SLE than in the general population. Therefore, it is especially important for SLE patients to avoid smoking and to minimize other conventional risk factors for atherosclerosis (e.g., hypercholesterolemia, hypertension, obesity, and inactivity). The presence of antiphospholipid antibodies and a history of associated thrombotic events suggest the presence of the antiphospholipid syndrome; anticoagulation with warfarin, heparin, or aspirin are the treatments of choice. Patients with SLE should receive an influenza vaccination every year and pneumococcal vaccination every 5 years. Since SLE patients have a higher risk of developing malignancy (especially lymphoma, lung cancer, and cervical cancer), preventive cancer screening recommendations should be followed assiduously.

A 27-year-old Caucasian man returns to the emergency department with unbearable left lower leg pain; he does so approximately 6 hours after initial discharge. While playing lacrosse, the patient sustained a closed, mid-shaft tibial fracture. After casting and an anti-inflammatory, his pain was noted to be mild (2 out of 10 on 1 - 10 scale) at time of discharge. He reports his pain is increasing dramatically (it is now rated at 9 out of 10) and is unresponsive to his prescribed narcotic, acetaminophen, icing, and elevating his leg. He also describes a feeling of tingling and numbness throughout the lower left extremity. The patient arrived on crutches and appears in obvious pain. He is afebrile; he has a pulse of 105. The cast is intact and the remainder of the left leg and foot is examined. The patient is tender to palpation of the left foot and ankle, with swollen, firm tissue. The skin appears shiny. Pulses are normal, but capillary refill and sensation is decreased on the left foot compared to the right. With passive movement of the patient's left toes, he cries out in pain. There is no ecchymosis or visible skin lesions on the left foot or ankle. Question What explains the physiologic basis for this patient's severe pain? Answer Choices 1 Chronic peripheral vascular disease, leading to poor arterial blood flow 2 Demyelination of the neuron sheath, leading to muscle weakness and sensation of pain and paresthesias 3 Immobility of the affected limb, leading to venous stasis and thrombosis 4 Increased dopamine release in the brain, leading to activation of reward centers in the nucleus accumbens 5 Increased pressure within a confined tissue space, leading to venous obstruction

Increased pressure within a confined tissue space, leading to venous obstruction This patient is exhibiting a presentation for compartment syndrome, in which the basic process is increased pressure within a confined tissue space, leading to venous obstruction. As the pressure rises, muscle, and nerve necrosis may occur. Compartment syndrome is an emergent condition, which may arise after significant trauma, such as a fracture or burn. It must be quickly identified and treated to prevent irreversible tissue damage. Chronic peripheral vascular disease would be unlikely to cause this patient's severe, acute pain. He is also much younger than the typical patient with chronic peripheral vascular disease. Demyelination of the neuron sheath is a problem with pain, paresthesias, and muscle weakness in various neurological disorders, such as multiple sclerosis. The acute injury this patient suffered should not cause any demyelination. Immobility of this patient's limb could ultimately lead to a deep venous thrombosis (DVT). However, the patient's pain began worsening shortly after his injury, and it is associated with the swelling of the tissue on his leg. This patient is relatively low risk for a DVT because he is young, active, and has no comorbidities. Increased dopamine release in the brain leading to activation of the reward centers is the mechanism for this patient's narcotic pain medicines. However, this process is the not the cause of his pain. If the patient had a simple fracture without compartment syndrome, his pain would likely be responsive to analgesics (e.g., narcotics).

A 40-year-old man presents with ongoing back pain; there is increased stiffness in the morning, and the pain has been going on for a few months. The patient reports that it can take him up to 30 minutes after waking up for the discomfort to improve. Physical examination and diagnostic testing confirms the diagnosis of ankylosing spondylitis. Question In addition to regular exercise, what medication will best control the patient's symptoms? Answer Choices 1 Tylenol (Acetominophen) 2 Indocin (Indomethacin) 3 Prednisone 4 Aspirin 5 Oxycodone

Indocin Explanation NSAIDs, particularly indomethacin, are effective in controlling the symptoms of ankylosing spondylitis; NSAIDs are effective due to their anti-inflammatory properties as well as the relative safety of their long-term use. Tylenol is not an NSAID; it would not be effective in this case. Prednisone has anti-inflammatory properties, but it is not an ideal choice for chronic treatment of this condition; there are serious adverse effects associated with chronic use. Aspirin is a drug that has mostly antiplatelet properties, as well as some anti-inflammatory properties, which does not make it an ideal choice for treating this condition. Oxycodone is strictly a pain reliever; it has no anti-inflammatory properties, which are needed to treat this condition.

A 22-year-old man presents with a several-month history of lower back pain. The pain is worst in the morning and is associated with stiffness. His symptoms improve with activity. He has no known injury to the area, and attempts to alleviate the pain with a heat pack have been unsuccessful. His ESR and CRP are both elevated, and testing for HLA-B27 is positive. An X-ray is shown in the image. Question What is the first-line treatment for this patient? Answer Choices 1 Rest and continued supportive treatment with heating therapy 2 Acetaminophen 3 Indomethacin 4 Etanercept 5 Methotrexate

Indomethacin: Explanation The above patient has ankylosing spondylitis and should be treated initially with indomethacin or other nonsteroidal anti-inflammatory drugs (NSAIDs). Ankylosing spondylitis is an autoimmune disorder that leads to progressive stiffening and eventual fusion of the spine. It is most common in males in their 2nd and 3rd decade of life. Patients complain of pain and stiffness, which are typically worst in the morning and improve with activity. Patients typically have elevation in their ESR and CRP. HLA-B27 is positive in 90% of Caucasian patients and 50% of African-American patients. The above patient's presentation, laboratory, data and X-ray all support the diagnosis of ankylosing spondylitis. Rest and continued supportive treatment with heating therapy is incorrect. Patients with ankylosing spondylitis are encouraged to remain active to help maintain their flexibility and posture. Acetaminophen is incorrect. While acetaminophen may alleviate some of the pain associated with ankylosing spondylitis, it will not target the inflammation that contributes to the disease. Etanercept, a tumor necrosis factor (TNF) inhibitor, may be used in the treatment of ankylosing spondylitis. However, it is considered a second line agent for those whose disease is not responsive to NSAIDs. Methotrexate is sometimes used in the treatment of ankylosing spondylitis, but it would not be the preferred initial agent of choice. Methotrexate is more commonly used in the treatment of rheumatoid arthritis. This patient is unlikely to have rheumatoid arthritis given the location of his symptoms, his radiographic findings, and his positive HLA B27.

What is the first type of motion that is lost as a person develops progressively worsening osteoarthritis of the hip joint?

Internal Rotation the earliest sign of the development of osteoarthritis of the hip is often the loss of internal rotation. As the condition worsens, muscular contractures may develop which hold the affected limb in a flexed and externally rotated position, which has great consequence on the individual's gait and functioning level. Typically a person experiencing this type of osteoarthritis will eventually develop an antalgic gait where the time spent bearing weight on the affected limb is brief due to pain. Also, the gluteus medius (which is a hip abductor and helps stabilize the pelvis) may become weakened as the condition worsens, resulting in an abductor lurch as the trunk of the body sways out over the affected limb when attempting to walk.

A 54-year-old man presents with acute onset of excruciating pain in his right toe. The patient states the pain began shortly after dinner and has progressively worsened since then to the point where he is now unable to bear weight on the affected side. Aside from occasional backaches, he has never experienced pain like this before. He reports overall good health and aside from a multivitamin, he uses no medications or supplements. On exam, the patient's right foot is swollen, and the joint of the great toe is tense and inflamed. His temperature is 38° C, blood pressure is 155/85, and pulse is 103 beat per minute. Question What test would confirm the most likely diagnosis? Answer Choices 1 Serum uric acid level 2 Joint aspiration 3 White blood cell count 4 X-ray of the affected joint 5 Blood culture

Joint aspiration Explanation The patient's clinical presentation is most consistent with the diagnosis of gout. Gouty arthritis is a complication resulting from an elevated serum urate level. Elevated urate is most commonly caused by renal under excretion caused by drugs such as diuretics or salicylates, renal insufficiency, or less commonly inherited renal tubular defects in urate handling. About 10% of patients—so-called "overproducers"—generate excess urate. Excess urate production may result from enzymatic defects in the purine degradation pathway, as in the case of Lesch-Nyhan syndrome. Alternatively, excess urate may be caused by increased cellular turnover resulting from hemolysis, malignancy, or chemotherapy. Regardless of the cause, hyperuricemia leads to deposition of urate in joint spaces, with subsequent crystallization that causes inflammation and other clinical signs and symptoms as presented in this patient. Gouty arthritis most commonly affects a single joint of the lower extremity. Classically, it affects the metatarsophalangeal joint of the great toe. The patient experiences acute onset of joint swelling, pain, and erythema. Mildly elevated temperature and leukocytosis may also be present. Since the symptoms of gout can mimic other causes of acute joint pain, such as that caused by infection or autoimmune conditions such as rheumatoid arthritis, it important to establish a definitive diagnosis. This is best accomplished by obtaining a sample of synovial fluid from the affected joint space. In the case of gout, examination of the specimen using polarized light microscopy will reveal negatively birefringent, needle-shaped urate crystals. The shape of the crystals distinguishes gout from other crystal-related joint diseases. Pseudogout, which is caused by calcium pyrophosphate dehydrate deposition, produces crystals that are rhomboid-shaped with weakly positive birefringence. Arthritis caused by hydroxyapatite deposition appears as brown globules under the microscope. To rule out septic arthritis, synovial fluid specimens should be sent for Gram stain and culture as well. Although most patients with gout have elevated serum uric acid levels, this does not establish the diagnosis; therefore, it is not the test of choice for diagnostic confirmation. While a mildly elevated white blood cell count also frequently accompanies the presentation of gout, it is seen with many other causes of acute arthritis; therefore, it is not helpful for definitive diagnosis. An X-ray of the affected joint is likely to demonstrate inflammation, but this also is a non-specific finding. If an infectious arthritis is suspected, blood cultures may reveal the offending organism; however, culture of aspirated joint fluid will provide a more definitive result.

An avulsion fracture at the base of the fifth metatarsal is commonly called which of the following? A Bennett fracture B Boxer's fracture C Chauffer's fracture D Jones fracture E Lisfranc fracture

Jones Fracture D An avulsion fracture at the base of the fifth metatarsal, usually secondary to plantar flexion and inversion is called a Jones fracture. Also called a ballet or dancer's fracture, it is the most common metatarsal fracture. The fracture occurs at the proximal diaphysis. A Bennett fracture is an oblique fracture of the first metacarpal near the carpometacarpal joint. A boxer's fracture is a fracture of the fifth metacarpal. This is the most common fracture of the hand. A chauffer's fracture is an oblique fracture through the base of the radial styloid in the forearm. A Lisfranc fracture is actually a fracture and dislocation involving the tarsometatarsal joints.

A 13-year-old girl presents for her school physical. On examination, you notice the posterior curvature of her thoracic spine to be very prominent and bulging backwards. What type of deformity of the spine does she have? Answer Choices 1 Kyphosis 2 Scoliokyphosis 3 Scoliosis 4 Lordosis 5 Gibbus

Kyphosis Kyphosis is a posterior convex angulation of the thoracic spine as evaluated on a side view. Excessive and exaggerated angulation results in cosmetic problems, back pain, and cardio-respiratory problems. Kyphosis may be post-traumatic or non-traumatic type. Scheuermann's kyphosis, postural, congenital, and metabolic bone disease are a few of the non-traumatic causes of kyphosis. Scoliosis is the lateral curvature of the spinal column. This can be functional or structural when it compensates for other abnormalities. Scoliokyphosis is the combined lateral and posterior curvature of the spine. Lordosis is the anterior angulation of the spine in sagittal plane. It is the accentuation of the normal lumbar curve. It may be a compensation for other deformities such as kyphosis. Gibbus is a sharp angular deformity of the spine that could be due to a collapsed vertebra. The deformity may be protruding outward and may be caused by metastatic cancer or tuberculosisof the spine.

An 83-year-old malnourished woman presents for examination. She stands with her shoulders rounded and has an exaggerated thoracic convexity. What type of deformity of the spine does she have? Answer Choices 1 Scoliosis 2 Scoliokyphosis 3 Kyphosis 4 Lordosis 5 Gibbus

Kyphosis Kyphosis is the posterior (backward) curvature of the spine as viewed from the side. It is a rounded thoracic convexity. This is common in aging patients and in women. Scoliosis is the lateral curvature of the spinal column. This can be functional or structural when it compensates for other abnormalities. Scoliokyphosis is the combined lateral and posterior curvature of the spine. Lordosis is the curvature of the lumbar spine. It is the accentuation of the normal lumbar curve. It may be as a compensation for other deformities, e.g. kyphosis. Gibbus is an angular deformity of a collapsed vertebra. The deformity may be protruding outward and may be caused by metastatic cancer or tuberculosis of the spine.

An 8-year-old afebrile Caucasian boy presents with significant right hip and knee pain. The patient walks with a limp, and his mother denies any history of trauma. On physical examination, you note atrophy of the right quadriceps muscles and decreased range of motion (ROM) of the right hip, particularly with internal rotation and abduction. Leg length inequality is also observed on the right compared with the left. Frog-leg radiographs of the hip reveal cessation of growth at the capital femoral epiphysis, and a smaller femoral head epiphysis with widening of the articular space on the right side. There is also a linear radiolucency within the right femoral head epiphysis. White blood cell counts, C-reactive protein, and erythrocyte sedimentation rates are all normal. Question What is the most likely diagnosis? Answer Choices 1 Legg-Calvé-Perthes disease (LCPD) 2 Slipped capital femoral epiphysis (SCFE) 3 Developmental dysplasia of the hip (DDH) 4 Duchene muscular dystrophy 5 Osteomyelitis of the right hip

Legg-Calvé-Perthes disease (LCPD) Legg-Calvé-Perthes disease (LCPD) is an idiopathic osteonecrosis of the capital femoral epiphysis of the femoral head; it can lead to severe, degenerative arthritis if not treated promptly and aggressively. Patients with LCPD are usually male Caucasians between the ages of 3 and 10 (with a median age of 7). Patients may present with a limp, hip pain, or anterior knee pain without a history of trauma. Physical findings of LCPD include decreased hip motion (especially with internal rotation and abduction), muscle spasm, painful gait, leg length inequality, short stature, and thigh muscle atrophy. Plain film radiographs in affected patients reveal cessation of growth at the capital femoral epiphysis, and a smaller femoral head epiphysis with widening of the articular space on the right side. If a linear radiolucency is also seen within the right femoral head epiphysis, this is suggestive of a subchondral fracture. Treatment of LCPD includes minimal weight-bearing and protection of the joint; this is accomplished by keeping the femur abducted and internally rotated so that the femoral head is held inside the rounded portion of the acetabulum. Abduction and rotation of the femur is accomplished either by the use of orthotic devices (bracing) or surgery (osteotomy). Slipped capital femoral epiphysis (SCFE) is a condition that occurs when the femoral head is displaced posteriorly and inferiorly in relation to the femoral neck, placing the patient at risk for avascular necrosis unless there is orthopedic internal fixation. Patients with SCFE are usually between 10 - 16 years of age. They are more likely to be obese, male, and African-American. The clinical and radiographic findings in the patient are more consistent with LCPD than SCFE. Developmental dysplasia of the hip (DDH) is a condition that affects infants and is associated with ligamentous laxity. The classic examination finding of children with DDH is revealed with the Ortolani maneuver, whereby a palpable 'clunk' is felt when the hip is reduced in and out of the acetabulum on physical examination. Treatment of DDH consists of a Pavlik harness, especially if instability is present. Duchene muscular dystrophy is an X-linked condition that leads to a defective dystrophin protein, a protein that is integral to the structural stability of the muscle myofiber. Without dystrophin, the muscles of affected patients are susceptible to mechanical injury; they undergo repeated cycles of necrosis and regeneration with progressive muscular weakening. The clinical history and radiographic findings in the case above are more consistent with Legg-Calvé-Perthes disease. Osteomyelitis of the right hip is a possibility in the patient. However, osseous infections normally produce characteristic laboratory values (elevated white blood cell counts, C-reactive protein, and erythrocyte sedimentation rate) that are typically accompanied by fever and pain of the affected extremity. 3-phase technetium radionuclide bone scanning and MRI examinations can aid in the diagnosis of osteomyelitis. It is unlikely in this patient due to his normal WBC and normal ESR.

n 8-year-old afebrile Caucasian boy presents with significant right hip and knee pain. The patient walks with a limp, and his mother denies any history of trauma. On physical examination, you note atrophy of the right quadriceps muscles and decreased range of motion (ROM) of the right hip, particularly with internal rotation and abduction. Leg length inequality is also observed on the right compared with the left. Frog-leg radiographs of the hip reveal cessation of growth at the capital femoral epiphysis, and a smaller femoral head epiphysis with widening of the articular space on the right side. There is also a linear radiolucency within the right femoral head epiphysis. White blood cell counts, C-reactive protein, and erythrocyte sedimentation rates are all normal. Question What is the most likely diagnosis? Answer Choices 1 Legg-Calvé-Perthes disease (LCPD) 2 Slipped capital femoral epiphysis (SCFE) 3 Developmental dysplasia of the hip (DDH) 4 Duchene muscular dystrophy 5 Osteomyelitis of the right hip

Legg-Calvé-Perthes disease (LCPD) Legg-Calvé-Perthes disease (LCPD) is an idiopathic osteonecrosis of the capital femoral epiphysis of the femoral head; it can lead to severe, degenerative arthritis if not treated promptly and aggressively. Patients with LCPD are usually male Caucasians between the ages of 3 and 10 (with a median age of 7). Patients may present with a limp, hip pain, or anterior knee pain without a history of trauma. Physical findings of LCPD include decreased hip motion (especially with internal rotation and abduction), muscle spasm, painful gait, leg length inequality, short stature, and thigh muscle atrophy. Plain film radiographs in affected patients reveal cessation of growth at the capital femoral epiphysis, and a smaller femoral head epiphysis with widening of the articular space on the right side. If a linear radiolucency is also seen within the right femoral head epiphysis, this is suggestive of a subchondral fracture. Treatment of LCPD includes minimal weight-bearing and protection of the joint; this is accomplished by keeping the femur abducted and internally rotated so that the femoral head is held inside the rounded portion of the acetabulum. Abduction and rotation of the femur is accomplished either by the use of orthotic devices (bracing) or surgery (osteotomy). Slipped capital femoral epiphysis (SCFE) is a condition that occurs when the femoral head is displaced posteriorly and inferiorly in relation to the femoral neck, placing the patient at risk for avascular necrosis unless there is orthopedic internal fixation. Patients with SCFE are usually between 10 - 16 years of age. They are more likely to be obese, male, and African-American. The clinical and radiographic findings in the patient are more consistent with LCPD than SCFE. Developmental dysplasia of the hip (DDH) is a condition that affects infants and is associated with ligamentous laxity. The classic examination finding of children with DDH is revealed with the Ortolani maneuver, whereby a palpable 'clunk' is felt when the hip is reduced in and out of the acetabulum on physical examination. Treatment of DDH consists of a Pavlik harness, especially if instability is present. Duchene muscular dystrophy is an X-linked condition that leads to a defective dystrophin protein, a protein that is integral to the structural stability of the muscle myofiber. Without dystrophin, the muscles of affected patients are susceptible to mechanical injury; they undergo repeated cycles of necrosis and regeneration with progressive muscular weakening. The clinical history and radiographic findings in the case above are more consistent with Legg-Calvé-Perthes disease. Osteomyelitis of the right hip is a possibility in the patient. However, osseous infections normally produce characteristic laboratory values (elevated white blood cell counts, C-reactive protein, and erythrocyte sedimentation rate) that are typically accompanied by fever and pain of the affected extremity. 3-phase technetium radionuclide bone scanning and MRI examinations can aid in the diagnosis of osteomyelitis. It is unlikely in this patient due to his normal WBC and normal ESR.

A 34-year-old man presents with a 10-year history of progressive weakness in his arms and legs. His problems include difficulty brushing his hair and raising his arms about his head. He also is unable to rise from a chair without using his arms to push himself up from a seated position. Your exam notes decreased muscle bulk and objective weakness of the deltoid, biceps, triceps, hip flexors, and hamstrings. Hand strength and handgrip release is normal. He reports that his father developed similar symptoms as a young man, and the father now requires a wheelchair. Question What diagnosis best explains the patient's symptoms? Answer Choices 1 Limb-girdle muscular dystrophy 2 Duchenne muscular dystrophy 3 Pompe disease (alpha-glucosidase deficiency) 4 Myotonic dystrophy 5 Hemochromatosis

Limb-girdle muscular dystrophy Explanation The history and physical exam are most compatible with a limb-girdle muscular dystrophy that affects the proximal muscle of the shoulder and pelvic girdles. The condition is not usually present at birth, but develops in young adults. The symptoms are gradual and progress over many years, and some adults will end up requiring assistive devices. Multiple-inheritance patterns have been described, including the autosomal dominant inheritance that best fits this case. Duchenne muscular dystrophy has an onset in young boys, progresses to death (usually in the late teenage years), and has an X-linked inheritance pattern where male-to-male transmission is not possible. Pompe disease is a lysosomal storage disease due to deficient enzyme activity of alpha-glucosidase (also called acid maltase). Like most enzyme defect disorders, it is inherited in an autosomal recessive fashion, which makes it less likely in this case because 2 generations are affected. Pompe may present with similar clinical features to this case, including the presentation of respiratory complaints such as sleep apnea due to respiratory muscle weakness. Myotonic dystrophy is a variable form of muscular dystrophy that can present in adults with symptoms of delayed/impaired muscle relaxation (myotonia). Commonly, this affects the hands; difficulty with opening jars, releasing doorknobs, and handshakes will be reported. Other features of myotonic dystrophy include cataracts, premature balding, and diabetes mellitus. Hemochromatosis is a disorder of iron overload that does not typically cause overt or prominent muscular symptoms. Instead, complains of arthritis and fatigue are common. Hepatic dysfunction is common in latter stages of the disease.

A 7-year-old is diagnosed with an acute case of hematogenous osteomyelitis accompanied with fever and leukocytosis. Based on your knowledge of the disease, which bone is most likely to present with the infection? A Feet B Hands C Long bones D Pelvis E Vertebrae

Long bones C Fortunately, hematogenous osteomyelitis is not common in children, but when it does occur it primarily is found in the long bones. The femur, tibia and humerus are the most typical locations for osteomyelitis in children. The highly vascular metaphysis of long bones contribute to the potential for hematogenous spread of the implicated pathogen. Osteomyelitis can occur at any of the locations mentioned in the answer choices given, but at a significantly lower rate than in the long bones. The rate of occurrence at several selected locations is given below: Feet - 9% Femur 25% Hands - 6% Humerus - 13% Pelvis - 8% Radius/ulna - 6% Tibia/fibula - 28% Vertebrae - 2%

A 16-year-old boy presents following his striking a wooden door with a closed fist an hour ago when he was angry at his mother. He is neurovascularly intact and the skin is closed. There is an obvious deformity with a loss of small finger metacarpal knuckle. Radiographs reveal an oblique midshaft fracture of the 5th metacarpal with palmar angulation of 45 degrees. What physical exam technique must be performed to check for rotation of the fracture? Answer Choices 1 Look at the finger nails while fingers are extended for position in relationship with each other 2 Look for finger malposition when the fingers are flexed into the palm 3 Look for folds in the skin over the metacarpal head 4 Check for motion of the PIP and DIP joints 5 Look for palmar edema of the affected digit metacarpal

Look for finger malposition when the fingers are flexed into the palm Explanation With a fractured 5th metacarpal, there is an increased incidence of malrotation of the distal digit due to potential loss of metacarpal height and lack of ligamentous support of the metacarpal head by the inter-metacarpal ligaments. Therefore, rotation must be checked closely to ensure proper position of the distal fracture segment in relation to the proximal fracture fragment for functional healing to take place. This is best accomplished by asking the patient to make a partial fist (gently) and watching for finger malposition (normally all the digits of a closed fist point towards a single spot on the scaphoid) or "cross-over". Be sure to compare the injured hand to the uninjured one as some "normal cross-over" can be seen with the 5th metacarpal. Looking at the fingernails in the extended position may not show rotation as easily as in the flexed position. Although there may be 'folds of skin' over the metacarpal head, this won't show you the malrotation if there is any. A fracture of the 5th metacarpal will not stop motion of the PIP or DIP joints (although it may cause pain at the fracture site). Edema is a given accompaniment with a fracture, but it won't give you an indication of fracture rotation.

A 19-year-old man presents with pain and deformity of his right dominant shoulder after a sudden jerking movement to the same from a wrestling competitor approximately 1 hour ago. He states he felt a clunking sensation when it happened. He was unable to continue wrestling and has pain with movement of the right shoulder. On a physical exam, what findings would you expect? Answer Choices 1 Extreme tenderness to palpation over the AC (acromioclavicular) joint 2 Loss of normal surface contour of the shoulder 3 Full passive range of motion 4 A frozen shoulder 5 Full active motion of the shoulder

Loss of normal surface contour of the shoulder With the dislocation of the glenohumeral joint, there will be a loss of normal contour of the shoulder's surface anatomy. Tenderness of the AC joint will be found more with a shoulder separation (AC Separation). There will be a great decrease in active and passive range of motion secondary to pain. Frozen shoulder has an insidious onset.

Which physical complaint is the leading cause of lost work time and disability for patients under the age of 45? A Ankle pain B Hip pain C Knee pain D Low back pain E Shoulder pain

Low back pain D Low back pain is the most common causes of lost time from work and disability. Factors that can lead to back problems include repeated or heavy lifting and twisting or use of vibrating equipment along with poor fitness levels, smoking, poor job satisfaction and other psychosocial factors. Fortunately, 80% of patients experience significant recovery within one month. All of the other types of pain can occur based on multiple factors, but none of them occur with a frequency that rivals low back pain

A football player plants his foot and is hit by an opponent on the lateral aspect of his knee. What structure is most likely affected? Answer Choices 1 Lateral collateral ligament 2 Lateral meniscus 3 Medial collateral ligament 4 Posterior collateral ligament 5 Arcuate ligament

Medial collateral ligament A force applied to the knee will tear the ligament on the opposite side. Striking the lateral aspect of the knee will damage the medial collateral ligament. Neither the meniscus nor the cruciate ligament is likely to be injured.

An 18-year-old man presents with progressive, severe knee pain. On physical examination, you note focal soft tissue swelling and erythema over the distal metaphysis of the left femur. Plain film radiographs of the left knee are obtained. Question To what anatomic location does the neoplasm demonstrated on the radiograph most commonly metastasize? Answer Choices 1 Axial skeletal red marrow 2 Contralateral knee 3 Brain 4 Lungs 5 Liver

Lungs Classic osteosarcoma is the most common malignant bone tumor in the pediatric population, and it is thought to arise from primitive mesenchymal, bone-forming cells. Classically, osteosarcoma is located in a metaphyseal, intra-medullary location, with its histologic hallmark being the production of a malignant osteoid. Plain film radiographs most commonly demonstrate an aggressive, bone-forming lesion located about the knee, most often affecting the femoral or tibial metaphysis. The 3 most common sites of origin for osteosarcoma are the femur (40%), tibia (20%), and proximal humerus (10%). The mainstay of osteosarcoma therapy is surgical removal of the malignant lesion, with a special emphasis on limb-sparing, limb-preserving procedures that preserve function and locomotion. Metastasis to the lungs is the most common; metastases to other sites are extremely rare.

A 60-year-old man accompanied by his daughter presents with a history of low back and buttock pain; it is 6/10 in severity. The pain starts when he stands or walks, but he can walk faster and with lesser intensity of pain when he is leaning over the shopping cart. On examination, the pain increases on extension of spine. Question What is the best radiographic test for diagnosis and detailed evaluation of this condition? Answer Choices 1 MRI (Magnetic resonance imaging) 2 CT (computed tomography) myelography 3 CT scan 4 Electrodiagnostic studies 5 X-ray of lumbar spine

MRI Explanation The likely diagnosis is Lumbar Spinal Stenosis (LSS). Magnetic resonance imaging (MRI) is the correct answer, as it is the most appropriate, noninvasive test for imaging degenerative lumbar spinal stenosis1. It helps in obtaining a more detailed evaluation of spinal structures, including hypertrophy of the facet joints and/or ligament flavum with corresponding decrease in spinal canal diameter2. CT myelography is a useful study in patients who have a contraindication to MRI, for whom MRI findings are inconclusive, or in patients for whom there is a poor correlation between symptoms and MRI findings1. It requires the injection of contrast medium, which carries the risk of anaphylactic or allergic reaction. Therefore, it is associated with more risk than conventional MRI or CT scan. CT scan is a useful noninvasive study in patients who have a contraindication to MRI, for whom MRI findings are inconclusive, or for whom there is a poor correlation between symptoms and MRI findings, as well as in whom CT myelogram is deemed inappropriate1. Electrodiagnostic studies are best used when there is concern about additional neurologic compromise, such as peripheral polyneuropathy, rather than establishing diagnosis1. X-ray examination has great limitations in diagnosis of LSS. Therefore, it is not used for detailed evaluation of this condition2.

A 27-year-old man presents with knee pain that started while he was playing basketball the afternoon prior to presentation. The patient states that as he attempted to pass the ball to a teammate, he twisted his left knee with his left foot still planted on the floor. He describes a popping sensation at the time of the injury, but he could still bear weight on his leg, and he does not recall any immediate swelling. The morning of presentation, the patient noted swelling of the knee and pain that was significantly worse than it was yesterday. He has taken 600 milligrams of ibuprofen for the pain, but experienced only mild relief. Upon examination of the left knee, you note a limitation in range of motion, the presence of medial joint line tenderness, and an effusion. McMurray's test is positive. Question What is the optimal imaging modality or procedure for confirming your suspected diagnosis? Answer Choices 1 Anterio-posterior (AP) and lateral X-ray 2 Ultrasonography 3 Computed tomography (CT) scan 4 Magnetic resonance imaging (MRI) 5 Exploratory arthroscopy

MRI Explanation The patient in the scenario above most likely has a medial meniscal injury of the left knee. Meniscal tears are the most common knee injuries encountered in primary care, resulting from a twisting action exerted on the knee joint while the foot is still in a weight-bearing position. Magnetic Resonance Imaging (MRI) is still the optimal test for confirming the diagnosis of a meniscal injury. All intra-articular structures, including menisci, ligaments, and articulating surfaces, can be visualized in great detail. MRI is also very sensitive to abnormalities of bone marrow that may not be readily apparent on either radiographs or CT. Patients with meniscal injuries often report a tearing or popping sensation at the time of injury, which is followed by severe pain. Effusions associated with these injuries accumulate over hours in contrast to ligamentous injuries, in which hemorrhage causes immediate swelling. Therefore, the knee swelling and pain associated with meniscal tears are typically worse the day after the injury. With medial meniscus injuries, tenderness localized to the medial joint line may be noted on exam, and the range of motion of the affected knee may be limited. McMurray's test can be performed to help suggest the diagnosis. Radiographs are usually the initial imaging studies performed, if any, when a patient complains of knee pain (acute or chronic). However, plain films best show general wear and tear of joint spaces (arthritic changes) and bony deformities rather than intra-articular soft tissue damage/injury. With any acute injury of the knee, plain radiographs can be used to exclude fractures or other conditions. Anterio-posterior (AP) and lateral radiographs of the knee best visualize the tibio-femoral and tibio-fibular joints. The tangential (aka "sunrise") view allows the assessment of the patellofemoral joint and provides an unobstructed view of the patella. Weight-bearing radiographs are helpful in assessing the extent of arthritis in the knee joint and in planning various joint resurfacing procedures. Diagnostic ultrasonography is useful in imaging the soft tissues about the knee, muscle injury, bone healing, and foreign bodies. Recent technologic improvements have made this imaging modality increasingly more accurate while providing significant benefits over traditional modalities such as magnetic resonance imaging, particularly regarding cost, patient satisfaction, and ease of usage. Increasing use of diagnostic ultrasonography for musculoskeletal conditions is likely as acceptance grows. The primary indication for computed tomography (CT) of the knee is to assess the alignment and degree of displacement of fracture fragments, particularly at the articular surfaces. The other major indication for a knee CT is to assess the integrity of the bone around a prosthesis (e.g., total knee arthroplasty or 'TKA'). Arthroscopy of the knee is an invasive procedure and is rarely needed to make an accurate diagnosis. However, arthroscopic procedures are commonly used to treat disorders/injuries that have already been confirmed by other imaging (e.g., meniscal injuries, damaged cartilage, foreign bodies, reconstruction of torn ligaments, etc.). Occasionally, arthroscopy of the knee is performed for general diagnostic purposes when a patient has chronic knee pain that cannot be explained by any other modality and is not improving with conservative intervention.

A 73-year-old man presents with the inability to actively raise his left non-dominant arm to retrieve plates from the kitchen cabinet. This began a month ago after his shoulder pain improved. He had a history of pain in that shoulder for over 6 months that kept him from sleeping on the left side and the pain would wake him often. There was no specific injury he can recall although he felt a pop a month ago while taking out the trash. What is the diagnostic study of choice if surgery is indicated? Answer Choices 1 CT without contrast 2 Electromyelogram (EMG) 3 Ultrasound 4 MRI 5 Plain films

MRI Explanation This elderly man probably has a torn rotator cuff. If surgical treatment is being considered, MRI is the imaging study of choice because it can provide additional information on the status of the muscle and the size of the rotator cuff tear. Electromyelogram is used to assess the nerves. Ultrasound doesn't give as clear a picture of the tear due to the bony interference. Plain films can assess only the boney picture.

Following injury to a nerve at the wrist, the thumb is laterally rotated and adducted. The hand has a flattened appearance and is "ape-like." What nerve is damaged? Answer Choices 1 Ulnar nerve 2 Anterior interosseous nerve 3 Deep radial nerve 4 Musculocutaneous nerve 5 Median nerve

Median nerve In the palm, the median nerve supplies the 3 muscles [(i) abductor pollicis brevis, which abducts the thumb at carpometacarpal joint and metacarpophalangeal joint; (ii) flexor pollicis brevis, which enables the thumb to form one claw in the pincer-like action used in the picking up of objects; and (iii) opponens pollicis, which enable the thumb to adduct at the carpometacarpal and at the metacarpophalangeal joint] of the thenar eminence. All 3 are supplied by the median nerve. Injuries to the nerve make the thumb laterally rotated and adducted and makes the hand have a flattened appearance. The ulnar nerve supplies the 3 muscles of the hypothenar eminence, the palmaris brevis, the 3rd and 4th lumbrical muscles, all the interossei muscles, and the adductor pollicis muscle. Anterior interosseous nerve supplies to the flexor pollicis longus, the pronator quadratus, and the lateral half of the flexor digitorum profundus, as well as to the wrist, carpus, and distal radioulnar joints. Deep radial nerve supplies the supinator muscle and the posterior compartment of the forearm. Musculocutaneous nerve supplies the biceps, coracobrachialis, brachialis (= muscular branches), skin of the front and lateral aspects of the forearm down as far as the root of the thumb (= cutaneous branches), and the elbow joint (= articular branches).

A 60-year-old man accompanied by his daughter presents with a history of low back and buttock pain; it is 6/10 in severity. The pain starts when he stands or walks, but he can walk faster and with lesser intensity of pain when he is leaning over the shopping cart. On examination, the pain increases on extension of spine. Question What is the best radiographic test for diagnosis and detailed evaluation of this condition? Answer Choices 1 MRI (Magnetic resonance imaging) 2 CT (computed tomography) myelography 3 CT scan 4 Electrodiagnostic studies 5 X-ray of lumbar spine

MRI The likely diagnosis is Lumbar Spinal Stenosis (LSS). Magnetic resonance imaging (MRI) is the correct answer, as it is the most appropriate, noninvasive test for imaging degenerative lumbar spinal stenosis1. It helps in obtaining a more detailed evaluation of spinal structures, including hypertrophy of the facet joints and/or ligament flavum with corresponding decrease in spinal canal diameter2. CT myelography is a useful study in patients who have a contraindication to MRI, for whom MRI findings are inconclusive, or in patients for whom there is a poor correlation between symptoms and MRI findings1. It requires the injection of contrast medium, which carries the risk of anaphylactic or allergic reaction. Therefore, it is associated with more risk than conventional MRI or CT scan. CT scan is a useful noninvasive study in patients who have a contraindication to MRI, for whom MRI findings are inconclusive, or for whom there is a poor correlation between symptoms and MRI findings, as well as in whom CT myelogram is deemed inappropriate1. Electrodiagnostic studies are best used when there is concern about additional neurologic compromise, such as peripheral polyneuropathy, rather than establishing diagnosis1. X-ray examination has great limitations in diagnosis of LSS. Therefore, it is not used for detailed evaluation of this condition2.

A 47-year-old grossly obese woman presents with left non-dominant shoulder pain and limited motion. The pain began about a month ago, and her shoulder has progressively lost motion during that time; now she can't reach overhead with that arm. There is no history of trauma; it doesn't wake her at night; and she can sleep on the left side. Her past medical history is significant for Diabetes Type I and hypothyroidism. She is on regular insulin and NPH insulin as well as levothyroxine (Synthroid). What diagnostic study would best help confirm your suspected diagnosis? Answer Choices 1 AP and Axillary lateral plain radiographs 2 CT 3 EMG 4 Ultrasound 5 MRI arthrogram

MRI arthrogram Explanation This is adhesive capsulitis (frozen shoulder). MRI arthrogram can substantiate a frozen shoulder by demonstrating a contracted capsule and loss of the inferior pouch of the capsule. Although plain radiographs are used to rule out pathology such as osteophytes, loose bodies, calcific tendonitis, or tumors, it won't show soft tissue, which is what the etiology of capsulitis is. CT is good for bony pathology mostly and less effective with soft tissues. EMG is used for nerve study. Ultrasound is good if there is fluid in the joint, but there is not in this case.

A 65-year-old man with uncontrolled diabetes presents for evaluation of a foot injury. He ran into his dresser 3 days ago, and an ulceration has developed at the site of the injury. He admits to minimal localized pain, and he denies any associated fever or chills. On examination, the ulceration measures 2.5 cm in diameter, and a sterile probe can be easily advanced to the underlying bone. Question Based on your suspected diagnosis, what diagnostic test should be ordered because it has the highest level of sensitivity? Answer Choices 1 Plain radiograph 2 Magnetic resonance imaging 3 Computer tomography 4 Venous ultrasound 5 Nuclear medicine bone scan

MRI has been identified as the most sensitive imaging modality for the diagnosis of osteomyelitis, and it is particularly helpful in identifying soft tissue involvement. Plain radiographs are readily available, but they can be associated with a false negative result early in the course of the disease. CT and nuclear medicine bone scanning are both less sensitive than MRI. Nuclear medicine images are most helpful in identifying a location or multiple sites of infection. Venous ultrasound is not indicated for the diagnosis of osteomyelitis; it is used in the diagnosis of deep vein thrombosis.

A 67-year-old man presents with low back pain. The pain has been worsening over the last month; however, he states that he has had a 'bad back' for years. He denies recent or remote trauma. He has self-medicated with OTC analgesics without relief. The pain worsens at night and at rest, and he is unable to sleep. What is most likely causing the man's pain? Answer Choices 1 Aortic aneurysm 2 Lumbar stenosis 3 Musculoskeletal pain 4 Spondylolisthesis 5 Malignancy

Malignancy All are causes of low back pain and should be included in your differential diagnosis; however, back pain that is unrelieved by rest and worsens at night should raise suspicion of malignancy. Given the age and gender of this patient, metastatic prostate cancer should be considered.

A 38-year-old man presents with pain and the inability to extend his middle finger DIP joint following a sudden jamming-type injury when attempting to catch a football 3 hours ago. His radiographs are normal. Based on the patient history, what is the most likely diagnosis? Answer Choices 1 Swan neck deformity 2 Mallet finger 3 Boutonnière deformity 4 DIP dislocation 5 Trigger finger

Mallet finger Explanation A mallet finger deformity is due to the rupture of the extensor tendon at the base of the dorsal distal phalanx of any digit of the hand. Patients will report pain dorsally at the DIP joint with the inability to actively extend the DIP joint. Swan neck deformity may be the result of a mallet finger, but only over time. Since this injury is acute, swan neck deformity cannot be the answer. Boutonnière deformity is a result of rupture of the central portion of the extensor tendon at its insertion onto the middle phalanx. It results in flexion of the PIP joint and hyperextension of the DIP joint. DIP dislocation would be evident on radiographs. Trigger finger is the result of thickening of the A1 pulley of the synovial sheath or flexor tendon itself at the entrance to the synovial sheath. This causes the tendon to "catch" when the finger is flexed and then to "snap" like pulling a "trigger" when trying to extend the finger.

What absolute tissue pressure generally is used as a guideline for diagnosing compartment syndrome?

Many trauma surgery services use an absolute tissue pressure of approximately 30 mm Hg as the threshold for diagnosing compartment syndrome. Based on the entire clinical picture, patients with numbers in that range or higher will likely require surgical decompression with a fasciotomy, while lower numbers will probably be managed with a more conservative approach.

A 32-year-old man presents with pain on the inner aspect of the right elbow that began a few days ago. The pain was initially mild; however, for the past 2 days, the man says he has been experiencing during daily activities such as lifting objects, brushing his teeth, shaking hands, etc. He recently became a rock-climbing instructor, and he climbs 3-5 days a week. On examination, there is point tenderness at a specific point on his medial elbow. Pain increases on wrist flexion and forearm pronation against resistance. An X-ray of the elbow is normal. Question What is the most likely diagnosis? Answer Choices 1 Tenosynovitis 2 Lateral epicondylitis 3 Medial epicondylitis 4 Biceps tendinosis 5 Olecranon impingement

Medial epicondylitis Explanation The correct answer is golfer's elbow, also called medial epicondylitis. Golfer's elbow is a condition of worsening pain at the medial aspect of the elbow during forearm pronation and sudden wrist flexion. The condition is associated with repetitive movements such as rock climbing, racquet games, carpentry, typing, etc. Diagnosis is by eliciting pain on wrist flexion and forearm pronation against resistance. X-ray is usually normal. Tennis elbow, or lateral epicondylitis, is a painful condition of the elbow caused by repetitive use of the wrist extensors. It is a degenerative tendinosis of the extensor carpi radialis brevi muscle. The condition is associated with racquet games, carpentry, knitting, etc. Symptoms include pain at the lateral elbow, worsening with activity, especially squeezing movements, lifting objects with palm down, turning a doorknob, and flexing the wrist towards the forearm. Diagnosis may be made by the tennis elbow test. X-ray is usually normal. Tenosynovitis is the inflammation of the synovial sheath, most commonly occurring at the wrist, hands, or feet. There is pain and swelling at the joint. Radial tunnel syndrome refers to pain in the dorsal forearm due to compression of the deep branch of the radial nerve. Olecranon bursitis may or may not present with painless swelling at the olecranon tip after repeated friction to the elbow. There is no tenderness or decrease in range of motion on examination.

A 25-year-old woman presents with pain and tingling sensation in her right hand. Her symptoms are especially intense at night. By holding her hand flexed for about a minute, you can provoke the symptoms. The paresthesia is extended to the palmar area of the thumb, index and middle finger, as well as half of the ring finger; thumb adduction and apposition are weakened on the right. What nerve is affected? Answer Choices 1 Ulnar 2 Radial 3 Median 4 Musculocutaneous 5 Peroneal

Median The described symptoms are typical for carpal tunnel syndrome, a compression of the median nerve in the volar aspect of the wrist between the flexor tendons and the superficial transverse ligament. The syndrome is relatively common, affects more women than men, can be uni- or bilateral, and is frequently associated with occupations that require a lot of wrist flexion. Compression of the ulnar nerve is often caused by trauma. Repeatedly leaning on the elbow is an example of trauma that could cause it. The paresthesia affects the palmar area of the 5th and half of the 4th finger as well as the dorsal area of the 5th, 4th, and half of the 3rd finger. There can also be weakness of the thumb adductor, 5th finger abductor, and interossei muscles. Compression of the radial nerve (having an arm hang over the back of a chair for a long period of time is an example of something that could cause it) leads to paresthesia over the area of the first interosseus muscle and weakness of wrist and finger extensors. The musculocutaneous nerve supplies the upper arm flexors and the skin of the radial side of the forearm. Compression of the peroneal nerve in the area of the fibular neck is common in bedridden patients or those with badly administered leg casts. Dorsiflexion and eversion of the foot are weakened, and a sensory deficit can be found over the anterolateral area of the lower leg, the dorsum of the foot, or between the 1st and 2nd metatarsal.

A 65-year-old woman presents after tripping on a rug in her home and falling on her outstretched arm. She states that her "hand bent backwards" and she heard a "snap". Physical exam shows an obvious deformity over the distal radius, with radial shortening. What anatomical structure may also be injured by this classic Colles' fracture? Answer Choices 1 Radial nerve 2 Median nerve 3 Ulnar nerve 4 Radial artery 5 Ulnar artery

Median Nerve Due to the mechanism of the injury (i.e., forced wrist dorsiflexion), the distal fragment is angulated dorsally, and the proximal fragment is volarly displaced. Bony fragments or compression within the carpal tunnel may injure the median nerve. The radial nerve and artery course in close approximation to each other, as do the ulnar nerve and artery, but they are not typically at risk in an uncomplicated classic Colles' fracture.

A 25-year-old woman presents with pain and tingling sensation in her right hand. Her symptoms are especially intense at night. By holding her hand flexed for about a minute, you can provoke the symptoms. The paresthesia is extended to the palmar area of the thumb, index and middle finger, as well as half of the ring finger; thumb adduction and apposition are weakened on the right. What nerve is affected? Answer Choices 1 Ulnar 2 Radial 3 Median 4 Musculocutaneous 5 Peroneal

Median Nerve he described symptoms are typical for carpal tunnel syndrome, a compression of the median nerve in the volar aspect of the wrist between the flexor tendons and the superficial transverse ligament. The syndrome is relatively common, affects more women than men, can be uni- or bilateral, and is frequently associated with occupations that require a lot of wrist flexion. Compression of the ulnar nerve is often caused by trauma. Repeatedly leaning on the elbow is an example of trauma that could cause it. The paresthesia affects the palmar area of the 5th and half of the 4th finger as well as the dorsal area of the 5th, 4th, and half of the 3rd finger. There can also be weakness of the thumb adductor, 5th finger abductor, and interossei muscles. Compression of the radial nerve (having an arm hang over the back of a chair for a long period of time is an example of something that could cause it) leads to paresthesia over the area of the first interosseus muscle and weakness of wrist and finger extensors. The musculocutaneous nerve supplies the upper arm flexors and the skin of the radial side of the forearm. Compression of the peroneal nerve in the area of the fibular neck is common in bedridden patients or those with badly administered leg casts. Dorsiflexion and eversion of the foot are weakened, and a sensory deficit can be found over the anterolateral area of the lower leg, the dorsum of the foot, or between the 1st and 2nd metatarsal.

A 50-year-old obese woman presents with severe left knee pain. She states the pain began about 8 months ago but has gotten significantly worse in the last 3 months. The patient denies any trauma or event that initiated the pain. She notes stiffness in the knee first thing in the morning which only lasts around 5-10 minutes. The knee pain worsens with activity and is relieved with rest. The patient's medication list includes lisinopril 10 mg once daily for high blood pressure. She has a documented medication allergy to acetaminophen, which gives her hives. Physical examination reveals a female with a BMI of 40, limited range of motion of the left knee, and crepitus. Question Considering the most likely diagnosis for this patient, what pharmaceutical regimen would be recommended for her at this time? Answer Choices 1 Meloxicam 2 Allopurinol 3 Oxycodone 4 Alendronate 5 Risedronate

Meloxicam This patient most likely has degenerative joint disease, also known as osteoarthritis (OA). OA is the most common form of knee arthritis. Common characteristics of OA include onset after 40 years old (usually patients older than 55 years old) along with obesity, and typically these patients are genetically predisposed. Other components seen in patients with OA initially have an insidious onset of pain that will rapidly progress. Characteristics of this pain will that is recognized as being exacerbated by activity and relieved at rest. Morning stiffness is common in patients with OA, although it will resolve within a 30 minute time period. Common symptoms experienced by patients with OA include buckling or giving way of the affected joint due to the bony areas impinging upon each other. Physical examination findings include limited range of motion of the affected joint and crepitus; the patient above exhibits both findings. Patients may also have tenderness to palpation of the joint, joint effusion, or even palpable osteophytes. Initially pharmaceutical management will be to initiate acetaminophen, up to 4 grams a day. Since our patient has an acetaminophen allergy, this would not be an appropriate choice. NSAID therapy or a COX-2 inhibitor would be an appropriate next option, making meloxicam the correct answer in this scenario. Allopurinol would be appropriate in a patient scenario that has the diagnosis of gout. This would not apply to this patient. Narcotics such as oxycodone are an inappropriate choice at this time before trying anti-inflammatories. Both alendronate and risedronate are in the class of bisphosphonates and therefore are used in patients diagnosed with osteoporosis.

A 33-year-old woman presents for routine follow-up with an abnormal Computed Tomography (CT). Her past medical history includes a leg amputation 4 years ago for osteogenic sarcoma treated with neoadjunctive chemotherapy. She is married and a non-smoker. Her physical exam includes a BP-111/67 mm Hg, pulse-70/min, and a healed laparotomy incision for a colon resection from perforated diverticulitis. A chest CT demonstrates a new solitary, irregular, non-calcified 3 cm nodule. What is the most likely diagnosis of this nodule? Answer Choices 1 Primary lung cancer 2 Metastatic osteogenic sarcoma 3 Lymphoma 4 Pneumonia 5 Pulmonary thymoma

Metastatic osteogenic sarcoma Explanation Pulmonary metastases may present with symptoms similar to primary lung cancer. These include chest pain, cough, hemoptysis, and dyspnea. Usually metastatic lesions are detected with chest X-ray or CT. CT can characterize suspicious findings on chest X-ray and detail the number of metastasis. CT detects approximately 80% of pulmonary metastases detected at surgical exploration. Positron Emission Tomography (PET) scanning is currently being investigated as a modality to access for pulmonary metastases. In patients with a previous history of sarcoma, a new pulmonary nodule has a 92% probability of being a metastatic nodule. Patients who are candidates for resection with pulmonary metastasis include those who have the a controllable or controlled primary tumor, no extrapulmonary tumor, no better method of treatment available, and adequate medical status. Tumor pathology that benefits from resection under these circumstances includes sarcoma, germ cell tumors, colorectal tumors, breast tumors, hypernephroma, head and neck tumors, melanoma, and hepatocellular carcinoma. Approximate 5 year survival for colorectal is 37%, breast 37%, kidney 41%, sarcoma 31%, melanoma 21%, and germ cell tumors 68%. Surgical approaches include unilateral or bilateral thoracotomy, sternotomy, and video-assisted thoracic surgery. Prognostic factors include tumor doubling time, disease-free interval, number of metastasis, lymph node metastases, and complete resection. Primary pulmonary thymoma is rare. Treatment includes surgical resection and possibly post-operative radiation.

A 65-year-old woman presents with fatigue, loss of energy, lack of appetite, low-grade fever, muscle and joint aches, and stiffness. She has a glucose-6-phosphate dehydrogenase (G6PD) deficiency and a history of coronary artery disease. On examination, you find swollen, painful, and tender small joints in a symmetrical pattern. According to the history, physical, and X-rays, as well as the presence of rheumatoid factor and antibodies to cyclic citrullinated peptides, you conclude that she has a flare-up of rheumatoid arthritis. Question What long-term therapy should be recommended in order to modify the course of the disease? Answer Choices 1 Nonsteroidal anti-inflammatory drugs (NSAID) 2 Methotrexate 3 Sulfasalazine 4 Corticosteroids 5 Selective COX-2 inhibitors

Methotrexate Explanation Methotrexate is usually the initial synthetic disease-modifying antirheumatic drugs (DMARD) of choice for patients with rheumatoid arthritis. It is generally well-tolerated, and often produces a beneficial effect in 2 - 6 weeks. NSAIDs will help the pain, but they will not modify the course of rheumatoid arthritis. They should only be used in conjunction with DMARDs. Sulfasalazine is a second-line agent for rheumatoid arthritis. It is not indicated in this patient because it causes hemolysis in patients with a glucose-6-phosphate dehydrogenase (G6PD) deficiency. Low-dose corticosteroids will produce a prompt anti-inflammatory effect in rheumatoid arthritis and slow the rate of bony destruction; however, their multiple side effects limit their long-term use. They are given only to reduce disease activity until the slower acting DMARDs take effect, or as adjunctive therapy for an active disease that persists despite treatment with DMARDs. Selective COX-2 inhibitors are effective for treating the pain in rheumatoid arthritis, but they will not modify the course of disease; they can increase the risk of cardiovascular events, particularly in patients not taking aspirin.

A 43-year-old woman presents with stiffness and pain in her fingers. It takes her about an hour in the morning to be able to use her hands. The symptoms started approximately 3 months ago, and they have gradually worsened. Recently, she also realized that some of her finger joints are swollen. She has a 2-year history of knee pain when climbing stairs. The patient's medical history includes hypertension, hyperlipidemia, and coronary artery disease. Her current medication is aspirin (81 mg daily), celecoxib (600 mg daily), glucosamine (500 mg tid), niacin (3 g/day), simvastatin (10 mg qh), vitamin E, and a multivitamin supplement. Vital signs are temperature 98.5° F, blood pressure 145/85 mm Hg, pulse rate 80/min, and respiratory rate 20/min. Physical examination reveals a well-developed, well-nourished woman in no apparent distress. Physical examination is unremarkable except for swelling around the metacarpophalangeal and proximal interphalangeal joints of the 2nd to 4th fingers bilaterally. There are no skin alterations. Laboratory values are as follows: white blood cell count 26,900/mm3; hemoglobin 14.9 g/dL; hematocrit 44.4%; platelet count 152,000/mm3; and CRP 29 mg/L. X-rays taken of both hands and knees show juxta-articular osteopenia, bone erosions, and loss of articular cartilage. Question What is the most appropriate treatment option? Answer Choices 1 Methotrexate 2 Infliximab 3 Etanercept 4 Anakinra 5 Glucocorticoids

Methotrexate The correct response is methotrexate. Modern management of rheumatoid arthritis (RA) is based on the knowledge of the joint-destructive course of the disease. Early diagnosis and onset of therapy is of utmost importance. Destruction of joints can be reduced, and economic costs (incurred through morbidity and increased mortality) can be lowered. The goal of treatment is to prevent inflammatory damage to the joints, keep the joints functioning, and increase quality of life. There are 2 groups of basic therapeutics for the treatment of RA: BASIC THERAPEUTIC DRUGS DOSAGE Conventionals Methotrexate 7.5 - 25 mg/wk po, sc, im Sulfasalazine 2 - 3 g/day po Leflunomide 20 mg/day po Biologicals Infliximab 3 - 10 mg/kg IV Etanercept 25 mg/twice a week sc Adalimumab 40 mg/every 2 weeks sc Anakinra 100 mg/day sc Biologicals (i.e., infliximab, etanercept, adalimumab, and anakinra) should only be used after failing to treat RA with conventional basic therapeutics or in combination with them. The criterion standard is methotrexate; it prevents thymidine synthesis by blocking the enzyme dihydrofolate reductase. This leads to increased apoptosis of activated T-cells and decreased production of IL-1. Cyclosporine and hydrochloroquine are other conventionals; they are used in mild cases or in combination with methotrexate. Infliximab is a monoclonal anti-TNF-antibody and can be combined with methotrexate. 2/3 of patients who did not respond to conventional therapy respond to infliximab (with improvement of symptoms). Infliximab is usually administered IV at 0, 2, and 6 weeks, and every 8 weeks thereafter. The dosage can be increased from 3 mg/kg to 10 mg/kg. Side effects include headaches, respiratory infections, urinary tract infections, and sporadic allergic reactions. Before starting therapy with TNF-blockers, tuberculosis must be ruled out; therapy can lead to tuberculosis. Etanercept is a p75 TNF-α-receptor blocker; it is comparable to infliximab. In addition to the possible side effects of infliximab, reactions at the injection site have been reported. Anakinra is an interleukin-1-receptor blocker; it is injected subcutaneously every day. Increased infection rates and reactions at the injection site have been reported. Glucocorticoids should only be administered in combination with conventional basic therapeutics until the anti-inflammatory effect sets in. Dosage should be 7.5 - 10 mg/day, and can also be administered into the joint directly.

A 35-year-old man presents with right shoulder pain that is becoming progressively worse. He expresses concern that, although he visits the gym 3 times a week, over the past month he has not been able to increase the amount of weight he lifts secondary to the shoulder pain. He has not tried anything to alleviate the pain. The pain is at its worst at night while he is trying to sleep. He also reports pain while in the shower washing his hair or using the shoulder press machine at the gym. He denies any history of recent trauma or sports related injury; however, upon questioning he reports that about 1 month ago he and his wife painted their entire house in one weekend. Upon physical exam of the shoulders, no swelling, atrophy, redness, or bruising is noted. Point tenderness is noted over the right lateral deltoid muscle. Active ROM of the right shoulder at 80 degrees of abduction elicits pain. Patient has a negative drop arm test, negative apprehensive test, and a positive Neer impingement test of the right shoulder. Question What is the suggested first-line of treatment? Answer Choices 1 Rest with arm in sling 2 Modification of pain-producing activities, use of NSAIDs, and physical therapy 3 Corticosteroid injections 4 Surgical treatment and corticosteroid injections 5 Opioid analgesics

Modification of pain-producing activities, use of NSAIDs, and physical therapy The first-choice treatment is modification of activities that produce pain, use of nonsteroidal anti-inflammatory drugs (medications may help relieve symptoms), and physical therapy. The goal is pain control and inflammation reduction. Movement is needed to maintain flexibility and range of motion. Rest arm in a sling is incorrect because movement needs to occur in order to prevent frozen shoulder (adhesive capsulitis). Movement is needed to maintain flexibility and range of motion. Subacromial corticosteroid injections are considered to be the third line of treatment, not the first. Surgical treatment is considered for refractory shoulder cuff pain or tears if conservative management failed for more than 6 to 12 months (therefore, not the first line choice for treatment). Surgical treatment also depends on the patient's age, loss of function, weakness, and pain. Opioid analgesics might be used if the NSAIDs do not alleviate the pain; however, NSAIDs should be tried first.

What types of connective tissue are injured in a strain? A Bones and muscles B Fascia and joint capsules C Ligaments and joint capsules D Muscles and tendons E Tendons and bones

Muscles and tendons are injured in strains! A strain involves injury to the muscles and tendons that are responsible for active movement of various body parts. Fascia is a part of the muscle-tendon unit, so injury to fascia would be considered a strain as well. Injury to ligaments and joint capsules would be considered a sprain and damage to bone would be classified as a fracture.

A 42-year-old woman works full-time as a data entry clerk and often puts in many hours of overtime. She has started to notice numbness and tingling in her right thumb, index finger, middle finger, and half of her ring finger; symptoms are especially severe at night The numbness and tingling were intermittent for months, but they have become persistent during the past few days. Question What could be considered a predisposing factor for this patient's condition? Answer Choices 1 Myxedema 2 Hypoadrenalism 3 Hypoprolactinemia 4 Hypoglycemia 5 Cushing syndrome

Myxedema Explanation Myxedema is one of the metabolic predisposing causes for carpal tunnel syndrome. The patient's clinical presentation is suggestive of carpal tunnel syndrome. Carpal tunnel syndrome is a median nerve compressive neuropathy. Patients with myxedema or hypothyroidism have accumulation of myxedemateous tissue under the transverse carpal ligament, which causes compression of the median nerve in the carpal tunnel resulting in the manifestations of carpal tunnel syndrome. In carpal tunnel syndrome the median nerve is compressed in the wrist. The neurological distribution of her complaints corresponds to the distribution of the median nerve. The median nerve innervates the palmar surface of the thumb, index, middle, and radial half of the ring fingers and the motor branch innervates the thenar muscles. Electromyography and nerve conduction studies help in confirming the diagnosis of carpal tunnel syndrome. Patients can be managed with analgesics for pain, corticosteroids which can be administered orally or as local injection, and splint to prevent wrist flexion and further compression of the carpal tunnel. The other metabolic causes of carpal tunnel syndrome include acromegaly, diabetes mellitus, and amyloidosis. None of the other choices listed, which include hypoadrenalism, hypoprolactinemia, hypoglycemia, and Cushing syndrome, are associated with an increase in the occurrence of carpal tunnel syndrome.

A 70-year-old man with a history of type II diabetes presents with a 2-day history of a red, hot, painful, and swollen left toe. He cannot recall any recent injury or illness, but states that he has been eating a lot of protein-rich foods and consuming alcohol against medical advice. His only complaint is the painful foot. He denies fever, chills, headache, or problems with his right foot. An X-ray of the foot reveals no evidence of fracture, and analysis of the synovial fluid shows crystals that are negatively birefringent and needle-shaped. The patient begins treatment and is able to put pressure on the foot once again. Question What is the most likely initial treatment for the patient? Answer Choices 1 Colchicine 2 Allopurinol 3 Corticosteroids 4 NSAIDs 5 Antibiotics

NSAIDs This patient has gout, which is a form of inflammatory arthritis. It is the body's response to the deposition of uric acid crystals in the joints. Gout presents as acute monoarticular arthritis in 90% of patients. In early gout, usually only 1 or 2 joints are involved. Usually, they are the smaller, lower-extremity joints. Podagra, or inflammation of the first metatarsophalangeal joint, is the initial joint manifestation involved in about half of all cases. However, podagra may also be observed in patients with pseudogout, reactive arthritis, gonococcal arthritis, psoriatic arthritis, and sarcoidosis. The attacks usually begin abruptly and can reach maximum intensity in 6-12 hours. The joints are red, hot, and extremely tender. Untreated, the characteristics of gout change over time. The attacks become more polyarticular. Although more joints may become involved, inflammation in a given joint may become less intense. Attacks occur more frequently and last longer. Eventually, patients may develop a chronic polyarticular arthritis, which can be symmetrical and resemble rheumatoid arthritis. Tophi, which are collections of uric acid crystals in the soft tissues, occur frequently in untreated patients. They can be found in multiple locations, including the fingers, toes, and olecranon bursae; they can also be found along the olecranon, where they may appear to be rheumatoid nodules. Tophi tend to develop after 10 years in untreated patients who develop chronic gouty arthritis. Acute flares of gout can occur in situations that lead to increased levels of serum uric acid, such as the use of alcohol, overindulgence of certain foods rich in protein, trauma, hemorrhage, or the use of medications that elevate levels of uric acid. NSAIDS are the treatment of choice in most patients without underlying health problems. They have an anti-inflammatory effect that works by inhibiting cyclo-oxygenase, which acts to produce leukotrienes from arachidonic acid. Indomethacin is the drug of choice and is usually given at a dose of 25 - 50 mg, 4 times a day, unless the patient is elderly. However, other NSAIDs, such as ibuprofen, naproxen, sulindac, and ketoprofen, may be used. Aspirin is not recommended because it can elevate uric acid levels. Some of the side effects of NSAID therapy include gastropathy, nephropathy, and liver dysfunction. It can also cause fluid overload in patients with congestive heart failure. Therefore, their use should be limited in these patients. Colchicine is an antimitotic drug that is often used in patients who cannot take NSAIDs or corticosteroids. However, due to its side effects (such as nausea, vomiting, and diarrhea and following intravenous administration- bone marrow suppression, renal failure, and death), it is not used as a first-line treatment. The normal dosage is 0.5-0.6mg orally every hour until relief or side effects occur or until a maximum dosage of 6 mg is reached. Allopurinol is used in the prevention of future attacks. It is not used to treat acute flare-ups. Patients with frequent gout attacks may begin prophylactic treatment to prevent further episodes. Allopurinol is used in such cases. Allopurinol blocks xanthine oxidase, thereby reducing the production of uric acid. It should be used in patients who overproduce uric acid. It is the most effective agent to lower serum uric acid levels. The dosage is 200 - 300 mg daily. The side effects include rash, gastrointestinal symptoms, headache, urticaria, interstitial nephritis, and rarely hypersensitivity syndrome. It was originally developed as a chemotherapeutic agent. However, alcohol can interfere with its effectiveness. Sulfinpyrazone, a uricosuric agent, is an alternative agent to aid in the prevention of attacks. However, it is not used as frequently due to the fact that it can cause bone marrow suppression. Antibiotics are used when the symptoms are caused by an infectious agent. This is not the case in this patient.

A 50-year-old man was playing baseball with his company team last weekend and is now experiencing severe pain in his left shoulder. He states that he has been the team pitcher for several years now. He has developed pain progressively in his left shoulder over the last few months. He denies any numbness or tingling in his arm, hand, or fingers of his left upper extremity. Which exam would you expect to yield pain? Answer Choices 1 Neer test 2 Tinel sign 3 Phalen test 4 Bragard stretch test 5 Apley grind test

Neer test Explanation Neer test is the correct answer because this is the only exam listed that evaluates for a rotator cuff injury. The Neer test is performed by having the patient internally rotate the shoulder while the practitioner flexes the arm forward. Pain will be elicited as the supraspinatus muscle presses against the acromion. Tinel sign is incorrect because it evaluates for any irritated nerve such as in carpal tunnel syndrome. Phalen test is incorrect because it evaluates for carpal tunnel syndrome. Bragard stretch test is incorrect because it evaluates for nerve root irritation at the level of L4, L5, and S1. Apley grind test is incorrect because it evaluates for a torn meniscus.

A 54-year-old man presents with acute onset of excruciating pain in his right toe. The patient states the pain began shortly after dinner, and it has progressively worsened since then to the point where he is now unable to bear weight on the affected side. In addition, he explains that aside from occasional backaches, he has never experienced pain like this before. He reports overall good health and aside from a multivitamin, he uses no medications or supplements. On exam, the patient's right foot is swollen, and the joint of the great toe is tense and inflamed. His temperature is 38° C, blood pressure is 155/85, and pulse is 103 beat per minute. What is examination of synovial fluid from the affected joint space most likely to reveal? Answer Choices 1 Negatively birefringent, needle-shaped crystals under polarized light microscopy 2 Positively birefringent, needle-shaped crystal under polarized light microscopy 3 Negatively birefringent, rhomboid-shaped crystals under polarized light microscopy 4 Positively birefringent, rhomboid-shaped crystals under polarized light microscopy 5 Brownish clumps under polarized light microscopy

Negatively birefringent, needle-shaped crystals under polarized light microscopy The patient's clinical presentation is most consistent with the diagnosis of gout. Gouty arthritis is a complication resulting from an elevated serum urate level. Elevated urate is most commonly caused by renal underexcretion caused by drugs such as diuretics or salicylates, renal insufficiency, or (less commonly) inherited renal tubular defects in urate handling. About 10% of patients - so-called "overproducers" - generate excess urate. Excess urate production may result from enzymatic defects in the purine degradation pathway, as in the case of Lesch-Nyhan syndrome. Alternatively, excess urate may be caused by increased cellular turnover resulting from hemolysis, malignancy, or chemotherapy. Regardless of the cause, hyperuricemia leads to deposition of urate in joint spaces, with subsequent crystallization that causes inflammation and other clinical signs and symptoms as presented in this patient. Gouty arthritis most commonly affects a single joint of the lower extremity, classically the metatarsophalangeal joint of the great toe. The patient experiences acute onset of joint swelling, pain, and erythema. Mildly elevated temperature and leukocytosis may also be present. Since the symptoms of gout can mimic other causes of acute joint pain, such as those caused by infection or autoimmune conditions (e.g., rheumatoid arthritis), it important to establish a definitive diagnosis. This is best accomplished by obtaining a sample of synovial fluid from the affected joint space. In the case of gout, examination of the specimen using polarized light microscopy will reveal negatively birefringent, needle-shaped urate crystals. The shape of the crystals distinguishes gout from other crystal-related joint diseases. Pseudogout, which is caused by calcium pyrophosphate dehydrate deposition, produces crystals that are rhomboid-shaped, with weakly positive birefringence. Arthritis caused by hydroxyapatite deposition appears as brown globules under the microscope. To rule out septic arthritis, synovial fluid specimens should be sent for Gram stain and culture as well. Although most patients with gout have elevated serum uric acid levels, this does not establish the diagnosis and therefore is not the test of choice for diagnostic confirmation. While a mildly elevated white blood cell count also often accompanies the presentation of gout, it is seen with many other causes of acute arthritis, and therefore is not helpful for definitive diagnosis. An X-ray of the affected joint is likely to demonstrate inflammation, but this also is a non-specific finding. If an infectious arthritis is suspected, blood cultures may reveal the offending organism; however, culture of aspirated joint fluid will provide a more definitive result.

Septic arthritis in adults younger than 30 years is usually caused by A Neisseria gonorrhea B Staphylococcus aureus C Pseudomonas aeruginosa D Streptococcus pyogenes E Salmonella species

Neisseria gonorrhea A In patients younger than 30 years, gonococcus is the most common cause of septic arthritis. When all patients are considered, Staphylococcus aureus is the most common cause. Patients with prevalent joint disease and intravenous drug users are especially susceptible to Staphylococcus. Pseudomonas is also a common cause of septic arthritis in intravenous drug users. Salmonella is not a common cause of joint infection.

A 32-year-old man presents with pain in his back, buttocks, and posterior thighs for 2 days after lifting a heavy load at work. He denies pain sleeping, unless he rolls over in bed, and pain just standing or sitting still in a chair. He also denies any radicular symptoms or bladder or bowel dysfunction. Question What diagnostic tests does this patient require? Answer Choices 1 MRI of the LS spine 2 CT of the LS spine 3 Plain radiographs of the LS spine 4 Electromyelogram (EMG) 5 No diagnostic tests are required at this time

No diagnostic tests are required with an acute lower back pain patient initially. Plain films usually are not helpful to diagnosis acute back pain/strain. However, if the patient has pain at night, at rest, or has a history of significant trauma, then radiographs are indicated. MRI is best used for patients with findings suggestive of nerve impingement from a herniated disk. CT, though able to visualize herniated disk material, is best used for boney pathology. EMG is indicated if there is a long-standing nerve impingement.

A 52-year-old overweight woman has had pain in her right hand for the past month. She is employed as a pastry chef and has trouble making a fist. On exam, she is tender over the radial styloid. You have her flex her thumb into her palm and move the wrist into ulnar deviation. This movement recreates her pain. What is the initial treatment for this condition? Answer Choices 1 Nonsteroidal anti-inflammatory drugs (NSAIDs) and thumb spica splint 2 Corticosteroid injection into the tendon sheath and splinting 3 Surgical release of the tendon and oral steroids 4 Short arm casting for 6 weeks and oral steroids 5 Joint replacement surgery

Nonsteroidal anti-inflammatory drugs (NSAIDs) and thumb spica splint Explanation DeQuervain's disease is a type of tenosynovitis. The initial treatment is conservative, involving NSAIDs and splinting. If conservative therapy fails, then steroid injections (up to 3) may be tried. If injections fail to decrease symptoms, then surgery may be considered. No fracture is involved, so casting is not indicated. Tendon inflammation will not be corrected with joint arthroplasty.

A 43-year-old woman presents with pain, swelling, and limited motion of her right dominant elbow for the past half hour after slipping on a wet spot on the floor at work while carrying a tray of food. She landed directly on her bent elbow. She is unable to flex her elbow due to pain. There is edema and ecchymosis over the elbow itself. Which of the following physical exam findings is most possible with this injury? Answer Choices 1 Numbness of the thumb dorsal surface 2 Numbness of the back of the hand 3 Numbness of the little finger palmar surface 4 Decreased capillary refill of the middle finger 5 Absent radial pulse

Numbness of the little finger palmar surface Explanation This patient likely has a fracture of the olecranon. With this fracture, injury to the ulnar nerve is most possible. The ulnar nerve supplies sensation of the little finger palmar surface. Numbness of the dorsal thumb and back of the hand would be an injury to the radial nerve. Vascular compromise is rare with this injury, and thus capillary refill and radial pulse should be normal.

A 17-year-old female presents to your family practice office for the annual physical examination required by her cheerleading coach. Upon examination you note that her joints are more flexible than anticipated. You also note her long thin fingers. You listen to her heart and hear no murmurs. Her blood pressure is 105/65 mmHg, pulse 60 beats/min and regular, respirations of 15 breaths/min, and temperature 98.7˚F. As you are examining her she tells you that her "joints sprain and strain easily." Furthermore, you obtain family history and she tells you that some connective tissue disorder runs in her family. Before you can medically clear her you should do which of the following? A This patient does not require any further evaluation. B Obtain an echocardiogram. C Perform and EKG. D Perform a chest radiograph. E Refer her to a rheumatologist.

Obtain an echocardiogram B This patient has clear signs and symptoms that are suspicious for Marfan syndrome. The complications of Marfan syndrome include cardiovascular issues, especially valvular and aortic disease. An echocardiogram is an appropriate, non-invasive initial first-step to begin your investigation to rule out significant valvular and/or aortic root abnormalities.

A 56-year-old right hand dominant male presents with swelling to the right elbow. He explains to you that he has had this type of swelling before and was diagnosed with bursitis. Based on this description, where would the most likely site for the elbow bursitis be located? A Capitellum B Lateral epicondyle C Medial epicondyle D Olecranon E Radial head

Olecranon The olecranon bursa is very vulnerable to injury and chronic inflammation due to its superficial location on the extensor side of the elbow. Acute falls, chronic pressure, and other inflammatory processes such as rheumatoid arthritis or gout may contribute to olecranon bursitis. The medial and lateral epicondyles are known to be associated with chronic tendonopathies. The radial head is a common location for subluxation or fracture, while the capitellum at the distal end of the humerus is sometimes associated with fractures and articular cartilage damage.

A 54-year-old man presents with a 2-month history of left non-dominant shoulder pain. There is no history of trauma, but the pain began about a week after shoveling wet heavy snow from his 100-foot driveway. At first, the pain seemed to come and go from day to day, but has gotten progressively worse and more constant. Pain is worse with overhead use, and he cannot sleep on his left side; he wakes up if he rolls over onto his left shoulder. On exam, he is tender to palpation over the left humerus greater tuberosity. He has pain with Neer and Hawkins tests, but has a negative drop arm test. You prescribe physical therapy and non-steroidal anti-inflammatories, and you also advise taking a break from using the arm for heavy work.. Question At what point would you recommend a steroid injection? Answer Choices 1 Right away, as pain relief will make stretching more effective 2 Steroid injections should never be used for this problem 3 Only after failure of a home therapy program of at least 6 weeks 4 Only after 3 weeks of formal physical therapy treatment 5 Once a week for 6 weeks to reduce the inflammation

Only after failure of a home therapy program of at least 6 weeks When a home therapy program performed 3 to 4 times a day for 6 weeks fails to improve the symptoms of impingement syndrome, a subacromial injection of steroid could be considered. Posterior capsule stretching exercises should be performed at home prior to any steroid injection. Some patients require the supervision of a physical therapist to assure adequate stretching, but they still require at least 6 weeks of therapy prior to an injection. Steroid injections should not be repeated if the first one does not improve the symptoms for at least 4 to 6 weeks. Repeated steroid injections can weaken the tendons of the rotator cuff and potentially cause a rupture of the tendon.

An 11-year-old boy comes in to see you due to knee pain. He notes that he started having right knee pain about 3 months ago and has increasingly become more apparent. The patient plays basketball in the winter and baseball in the spring; he has done this for the last 2 years. When asked to point to where the pain is, the patient points to an enlarged area just inferior to the knee cap. Aggravating factors include participating in the sports mentioned, especially when he is doing excessive amounts of running and jumping. Alleviating factors include resting the knee. Physical examination reveals a prominence and soft tissue swelling over the right tibial tubercle as well as tenderness to palpation of the right patellar tendon. The remainder of the musculoskeletal exam is unremarkable. Question Given the most likely diagnosis, what would be the most appropriate pharmaceutical regimen that you can recommend at this time? Answer Choices 1 Oral corticosteroids 2 Oral nonsteroidal anti-inflammatory (NSAIDs) 3 Steroid injections of the joint 4 Oral antibiotics 5 Topical NSAIDs

Oral NSAIDs The condition that is being described in the above patient scenario is known as Osgood-Schlatter disease (OS). This is a condition that results from a tractions phenomenon; it results from repetitive quadriceps contraction through the patellar tendon at its insertion upon the skeletally immature tibial tubercle. By this description, the occurrence is found to be in adolescent patients, typically between 10-15 years old. Onset is typically gradual; the complaint of pain in the area of the tibial tubercle and/or patellar tendon region is common and especially noticeable after repetitive activities (especially running and jumping.) OS is seen to be the most frequent cause of knee pain in the pediatric population aged 10-15 years. OS rarely causes permanent impairment or disability, but rather loss of time from physical activity or athletics. Typical presentation will be a knee pain that is worsened with activity and improves at rest. Physical findings will include a prominence and soft-tissue swelling over the tibial tubercle as well as tenderness of the patellar tendon. Attempted flexion against resistance may initiate pain. Patients may also resist flexion of the affected knee because of inflammation and pain from the pull of the patellar tendon. Conservative care is commonly efficient and usually consists of rest and avoidance of the offending activity. NSAIDs could potentially be used to help alleviate the signs and symptoms but they have not been shown to shorten the course of OS. Topical NSAIDs would not be an appropriate option. Steroid injections as well as oral steroids should not be used as treatment of OS. Antibiotics are also not a viable option for treatment of OS.

A 13-year-old boy presents with a 5-month history of intermittent right knee pain. He notes that his pain is poorly localized over the anterior knee, but it is provoked by activities such as running, jumping, squatting, and climbing or descending stairs. As a result, he has had to curtail recreational games at recess and home. He denies any history of trauma, falls, injury, fever, chills, swelling, skin changes, hip pain, or ankle pain. His physical exam reveals a boy with an appropriate build. His right knee is notable for point tenderness over an enlarged and prominent tibial tubercle. There is associated focal swelling. An in-office X-ray is performed, demonstrating the following image. Question What is the most appropriate pharmacotherapeutic agent at this time? Answer Choices 1 Oral ibuprofen 2 Intraarticular triamcinolone 3 Oral gabapentin 4 Oral lorazepam 5 Oral prednisone

Oral ibuprofen Explanation This patient is presenting with signs and symptoms consistent with Osgood-Schlatter disease. Therapy for Osgood-Schlatter disease (OSD) is conservative. Initial treatment includes the application of ice for 20 minutes every 2 - 4 hours. The only medications that need to be prescribed are nonsteroidal anti-inflammatory drugs (NSAIDs), such as oral ibuprofen, for pain relief and reduction of local inflammation (any NSAID may be used). Along with nonsteroidal anti-inflammatory drugs (NSAIDs), analgesics (e.g., acetaminophen) may be given for pain relief and reduction of local inflammation. Steroid injections should not be used, as they may weaken the patellar ligament as well as thin and depigment skin. Oral corticosteroids (e.g., prednisone) are not necessary. Gabapentin is a gamma aminobutyric acid (GABA) analogue; structurally related to neurotransmitter GABA. It is indicated as adjunctive therapy for partial seizures with or without secondary generalization as well as for post-herpetic neuralgia and in amyotrophic lateral sclerosis. Off-label uses include treatments for insomnia, tremors, diabetic neuropathy, hot flashes, restless leg syndrome, and cocaine withdrawal. Lorazepam is indicated as treatment for anxiety, short term treatment of insomnia, for perioperative sedation, status epilepticus, and off-label for chemotherapy-induced nausea and chronic insomnia.

A 6-year-old boy has gradually increasing sharp pain to his left anterior hip that seems worse at night. He is moderately overweight, but he remains active and plays baseball. There is no history of trauma. There is no redness, swelling, or fever; there is no involvement of the other hip. Ibuprofen does seem to help. There is also no family history of any bone or joint disease. His weight is at the 95th percentile with height at the 50th percentile. On exam, there is mild tenderness to palpation over his left anterior hip, but the rest of the exam is normal. Laboratory studies show a normal complete blood count, hemoglobin, sedimentation rate, platelets, and C reactive protein. Plain X-rays show a small round lytic lesion surrounded by mild reactive bone formation. Question Based on these findings, what is the most likely diagnosis? Answer Choices 1 Legg-Calve-Perthes disease 2 Osteomyelitis 3 Primary lymphoma of bone 4 Osteoid osteoma 5 Ewings sarcoma

Osteoid osteoma Osteoid osteoma is a common benign bone tumor consisting of an ovoid or spherical nidus of osteoid-rich tissue with interconnected bone trabeculae on highly vascularized connective tissue. It ranges from 0.5 to 2 cm in size. It does not progress or have malignant potential. In fact, some lesions may regress spontaneously. Multinucleated giant cells, osteoclasts, and axons are frequently observed. There may be bone sclerosis around the central nidus. Most are cortical or periosteal, but 20% arise within the marrow. Most patients are 5 to 20 years old. Most commonly, the proximal femur and tibia are involved, although any bone may be affected. Pain is usually the presenting symptom and is frequently relieved with non-steroidal anti-inflammatory drugs (NSAIDs). Tenderness upon palpation may be elicited on exam; patients may also have disuse atrophy, painful scoliosis, or limb-length discrepancy. X-rays are the initial examination of choice to reveal the central nidus as a small spherical or oval lytic lesion surrounded by soft-tissue edema and possible reactive bone formation. Computed Tomography (CT) is also used for precise localization and as guidance for biopsy and ablation. The primary goal of treatment is pain control with NSAIDs; however, if pain is not controlled or complications such as neuropathy, synovitis, growth disturbance, or scoliosis develop, the lesion should be removed. Several techniques are available for ablation, including percutaneous radiofrequency, ethanol, laser, or thermocoagulation therapy under CT guidance. Surgical options include en bloc excision, curettage, or CT guided removal. Although small lytic lesions can be caused by infections or other benign neoplasms of the bone and leukemia, the history, physical findings, small size of the lesion, and location in the diaphysis make osteoid osteoma the most likely diagnosis. Ewing sarcoma is a highly malignant primary bone tumor derived from bone marrow. Males are affected more frequently than females, and it most commonly presents in children and adolescents aged 4-15 years. It is especially infrequent in African Americans and Asians. Metaphyseal or diaphyseal areas of long and flat bones are both affected. The earliest symptom is intermittent pain that becomes intense and may radiate to the limbs, particularly with tumors in the vertebral or pelvic region. X-rays will show a long, permeative lytic lesion in the metadiaphysis and diaphysis of the bone with a prominent soft tissue mass extending from the bone, or it will show a lesion with poorly defined margins that is destroying the bone. Occult metastatic disease is usually present, requiring multi-drug chemotherapy as well as local disease control with surgery and/or radiation. Length of treatment varies depending on location and staging, but most patients undergo chemotherapy for 6-12 months and radiation for 5-8 weeks. Legg-Calve-Perthes disease, a childhood hip disorder, results from infarction with idiopathic avascular necrosis of the bony epiphysis of the femoral head. It occurs bilaterally in 10-20% of children, and they usually present at 4-8 years of age. Children present with limp, pain, and reduced hip range of motion. Possible causes include trauma, developmental, inflammatory, and coagulation abnormalities. Risk factors include male gender, low socioeconomic group, presence of inguinal hernia or genitourinary tract anomalies, and low birth weight. Early radiological signs most commonly show a small femoral epiphysis, sclerosis of the femoral head with sequestration and collapse with fragmentation, slight widening of the joint space due to thickening of the cartilage, failure of epiphyseal growth, and the presence of joint fluid or joint laxity. Magnetic Resonance Imaging is more sensitive in detecting early disease. Treatment aims at maintaining containment of the femoral head within the acetabulum with abduction splints, casts, or surgically with an osteotomy of the proximal femur. Osteomyelitis is an infective process that may involve all osseous components including the metaphyses of long bones, particularly the tibia or femur. Hematogenous spread most commonly occurs in infants and young children. Neonates have group B streptococci and Escherichia coli as the most commonly involved organisms, with 50% also having a septic joint. Beyond the neonatal period, the most common organisms involved are Staphylococcus aureus, group A streptococci, and Haemophilus influenzae type b. Infections in this age group may gain access from traumatic injuries or surgical procedures. Joint involvement is rare beyond the neonatal period. Untreated or inadequately treated infections can become chronic, with staphylococcus isolated most frequently. Clinical presentation of hematogenous osteomyelitis can be variable, ranging from acute pain with decreased movement of the affected limb and adjacent joint to possible swelling and redness. Systemic signs can include fever, malaise, and irritability. Following trauma onset may be insidious; despite localized pain, edema, and redness, the frequent absence of systemic signs can misleadingly suggest a superficial infection. Laboratory studies may show an elevated or normal leukocyte count where sedimentation rate and C-reactive protein are typically elevated. A blood culture is positive in 50% of cases. Diagnosis is based on the history and physical exam findings supported by a positive technetium-99 methylene diphosphonate bone scan. X-ray changes show swelling of tissue around the infection and periosteal reaction or new bone formation, but these may not be evident until 1 to 2 weeks after onset of symptoms. Eventually, bone destruction with lytic lesions will be visible. Successful treatment requires an appropriate antibiotic at sufficiently high dose and duration. For staphylococcal infections, the most common cause of osteomyelitis, a penicillinase-resistant semisynthetic penicillin is the drug of choice. Primary lymphoma of bone (PLB) is a rare malignant neoplastic disorder. Most cases result from non-Hodgkin lymphoma. PLB tumors produce osteoclast-stimulating factors that cause lytic bone destruction. It is rare in children younger than 10 years of age and fairly evenly distributed in the 2nd through 8th decades. The usual presenting symptom is prolonged pain with possible swelling. Diagnostic criteria include a primary focus in a single bone, histologic confirmation, and no evidence of distant soft tissue or lymph node involvement. The most common location is the thoracic spine, but diffuse spinal involvement is also quite frequent. Treatment involves radiation therapy and chemotherapy. Surgery is indicated only when a large mass causes acute cord compression.

Which of the following is a type of primary malignant tumor of the bone? A Enchondroma B Leiomyoma C Lipoma D Melanoma E Osteosarcoma

Osteosarcoma Osteosarcoma is a type of primary bone cancer. Fortunately this is a rare cancer, accounting for only 1% of the cancers diagnosed each year in the USA for patients of all ages. Osteosarcomas are more common in the pediatric population, accounting for 5% of all childhood cancers and 56% of all cancers of the bone in patients less than 20 years old. Osteosarcomas have a bimodal occurrence rate with the highest prevalence between ages 11-13 and over the age of 65. Enchondromas are common benign cartilaginous tumors that develop in the medulla (marrow cavity) of bone. A leiomyoma is another name for a benign uterine fibroid. A lipoma is a benign, soft, freely movable, generally nontender mass in the soft tissue sometimes referred to as a fatty tumor. These are generally inconsequential, but may be a marker for spina bifida if found in the lumbar region. Melanoma can be found in bone, but it would be considered a metastatic lesion and not a primary malignant bone tumor.

A 72-year-old man presents with pain, rated 5/10, in his right upper arm. He heard a pop and then felt sudden pain when it occurred. On exam, there is ecchymosis of the upper and middle arm; there is also a visible, palpable bulge proximal to the elbow. He reports a longstanding history of impingement syndrome in the same arm. What treatment is indicated for this patient? Answer Choices 1 Physical therapy 2 Cortisone injection 3 Arthroscopic debridement 4 Surgical repair 5 Narcotic analgesics

PT Explanation Range of motion and strengthening exercises are the best treatment for a ruptured proximal biceps tendon in an elderly patient as there is minimal loss of function. Surgical repair of the torn tendon is indicated in young, athletic patients. Cortisone injections would not be helpful, and debridement is not necessary. Pain is usually mild; therefore, narcotics are not needed.

A 16-year-old girl presents with severe right knee pain; it started during a softball game 3 days ago after the patient swung to hit the ball. She reports hearing a popping sound before the pain began. There is no significant past medical history. Examination of the right knee reveals a positive 'apprehension sign'; Lachman, McMurray, valgus, and varus tests are negative. X-rays show a lateral tilt of the right patellar and subluxation. Question What is the most likely diagnosis? Answer Choices 1 Anterior cruciate ligament tear 2 Medial collateral ligament tear 3 Medial meniscal tear 4 Patellofemoral pain syndrome 5 Patellar dislocation

Patellar dislocation Explanation Patellar dislocation is commonly seen with a history of twisting of the torso when the foot is planted. The positive apprehension sign, in conjunction with the X-ray findings, are diagnostic for this disorder only. Anterior cruciate ligament tear is incorrect; there would be increased anterior laxity with Lachman test, which was negative. Medial collateral ligament tear is incorrect; there would be pain and laxity with valgus testing, which was negative. Medial meniscal tear is incorrect; the McMurray test was negative, and it would have elicited pain at the medial joint line. Patellofemoral pain syndrome is incorrect; it does not include instability or dislocation of the patellar, which is evident on X-ray.

A 34-year-old woman presents Monday morning with knee pain. She states that she was playing with her 2 children in their backyard over the weekend and fell onto solid ground, landing directly on her knees. She noted immediate and significant right knee pain. She notes significant swelling; it accompanies the knee pain which she rates as an 8/10 on a numerical pain scale. Physical examination reveals significant obvious joint effusion and exquisite focal tenderness to palpation over the patellar area of the right knee; the left knee has no obvious abnormalities. She is able to appropriately extend both legs against gravity. Results of the anterior drawer, McMurray, and varus/valgus stress testing are within normal limits. Question Given these history and physical examination findings, what is the most likely diagnosis? Answer Choices 1 Bone cyst 2 Patellar fracture 3 Ligament tear 4 Tibial fracture 5 Femoral shaft fracture

Patellar fracture Explanation The patient above has most likely experienced a right patellar fracture. Many fractures of the patella are the result of direct forces, such as striking the dashboard in a motor vehicle accident, or in our patient's scenario, falling onto a flexed knee. Patellar fractures should be high on one's differential diagnosis list if a patient presents with acutely swollen knee and patella pain following trauma to that area. There will be presence of jointeffusion and focal tenderness to palpation of the patella. Another feature is that the patient will be unable to extend the knee against gravity, which requires surgical repair if present. Our patient had all these signs and symptoms, minus the inability to extend the knee. A bone cyst is described as a fluid-filled hole that develops inside a bone. These most commonly occur in children and young adults and very rarely cause significant symptoms. The cause of bone cysts is unclear, but they usually will not cause serious health issues. This does not match what is described in the original question. Tibial fractures are caused by very similar mechanisms of injury and present with similar signs and symptoms. If a tibial fracture has occurred, there is more often than not a ligamentous or meniscal injury in conjunction with the tibial fracture. With these issues present, the patient would not even be able to bear the various tests to assess these structures, and the patient would have positive findings when completing the anterior drawer and/or Lachman test (assesses ACL), the McMurray test (assessing meniscal injury), or the varus and valgus stress tests, which assess the medial and lateral collateral ligaments. Femoral fractures are not commonly caused by the type of injury described in the patient scenario. The femur is the longest, strongest, and heaviest bone in the body, and along the same lines is the principle load-bearing bone in the lower extremity. If a femoral shaft fracture does occur, it is almost always the result of high energy forces, such as in motor vehicle collisions, being struck by a motor vehicle, motorcycle accidents, falls from height of 3 meters or more, or gunshot wounds. Presentation is typically substantial in its signs and symptoms: pain, swelling, deformity, or even shortening of the thigh. This is not consistent with the clinical scenario above. Ligament tear is low in the differential diagnosis as well. The patient never mentioned a "pop" at the time of injury, which is extremely common and indicative of a ligament injury. With this injury, patients will have acute swelling and a feeling that the knee will give out and is unstable. Almost all will develop knee effusion from hemarthrosis. The anterior drawer is seen as 1 of the 3 tests that are most sensitive and specific when diagnosing ACL tears during the physical examination, and in the case of the patient above, there were no abnormal results.

A 32-year-old woman with no significant past medical history presents with a 3-month history of right anterior knee pain which is described as a dull and aching pain that is 'right under the kneecap'. Provocative activities include bending movements, descending stairs, and performing squatting maneuvers. Pain is relieved during rest. She notes that she is an avid outdoorswoman whose hobbies include running and hiking; her symptoms began following a running session. She denies a history of falls, prior surgeries or instrumentation, fever, chills, malaise, myalgias, changes in weight, joint swelling, skin changes or rashes, or other joint pains. Her physical exam is normal with the exception of a tender undersurface of the patella, with crepitus upon passive range of motion of the right knee. There is abnormal patellar tracking upon right knee flexion and apprehension of the patient upon passive manipulation of the patella. Additionally, there is a positive patellar grind test. McMurrary's, Lachman, the anterior and posterior drawer, Apley's compression and distraction tests, and varus/valgus tests are all negative. There is no joint line tenderness, effusion, or restriction of range of motion of the right knee. Question What is the most likely diagnosis? Answer Choices 1 Right knee meniscal tear 2 Tear of the medial collateral ligament 3 Anterior cruciate ligament tear 4 Iliotibial band syndrome 5 Patellofemoral pain syndrome

Patellofemoral pain syndrome Explanation This patient exhibits signs and symptoms of patellofemoral pain syndrome, also known as chondromalacia patellae, miserable malalignment syndrome, and runner's knee. This syndrome describes any pain involving the patellofemoral joint. The pain affects any or all of the anterior knee structures, including the medial and lateral aspects of the patella, as well as the quadriceps and patellar tendon insertions. The patella engages the femoral trochlear groove with approximately 30 degrees of knee flexion. Forces on the patellofemoral joint increase up to 3 times body weight as the knee flexes to 90 degrees (e.g., climbing stairs), and 5 times body weight when going into full knee flexion (e.g., squatting). Abnormal patellar tracking during flexion can lead to abnormal articular cartilage wear and pain. When the patient has ligamentous hyperlaxity, the patella can sublux out of the groove, usually laterally. Patellofemoral pain is also associated with muscle strength and flexibility imbalances, as well as altered hip and ankle biomechanics. Patients usually complain of pain in the anterior knee with bending movements and less commonly in full extension. Pain from this condition is localized under the kneecap but can sometimes refer to the posterior knee or over the medial or lateral inferior patella. Symptoms may begin after a trauma or after repetitive physical activity, such as running and jumping. When maltracking, palpable and sometimes audible crepitus can occur. Intra-articular swelling usually does not occur unless there are articular cartilage defects or if osteoarthritis changes develop. Patellar mobility can be assessed by medially and laterally deviating the patella (deviation by one-quarter of the diameter of the kneecap is consider normal; greater than one-half the diameter suggests excessive mobility). The apprehension sign suggests instability of the patellofemoral joint and is positive when the patient becomes apprehensive when the patella is deviated laterally. The patellar grind test is performed by grasping the knee superior to the patella and pushing it downward with the patient supine and the knee extended, thus pushing the patella inferiorly. The patient is asked to contract the quadriceps muscle to oppose this downward translation, with reproduction of pain or grinding being the positive sign for chondromalacia of the patella. The Iliotibial band syndrome typically presents with a lateral "snapping" with flexion and extension of the knee. Patients with ACL tears hear an audible 'pop' and usually fall down following the injury. They also have acute swelling, difficulty with weight-bearing, and complain of instability. Positive Lachman and Anterior Draw tests are observed. Medial collateral ligamentous injuries usually occur as a result of a valgus stress to the partially flexed knee. It can also occur with a blow to the lateral leg. The MCL is commonly injured with acute ACL injuries. The patient may have a limited range of motion due to pain, especially during the first 2 weeks following the injury. A positive valgus stress test is expected. Injuries to a meniscus can lead to pain, clicking, and locking sensation. Most meniscus injuries occur with acute injuries (usually in younger patients) or repeated microtrauma, such as squatting or twisting in older patients. Clinical findings include an antalgic gait, difficulty with squatting, catching, or locking of the meniscal fragment, effusion, and joint line tenderness. Patients can usually point out the area of maximal tenderness along the joint line. The McMurray test is usually present.

A 19-year-old boy presents with pain and deformity of his right dominant shoulder after a sudden jerking movement to the same from a wrestling competitor approximately 1 hour ago. He states he felt a clunking sensation when it happened. He was unable to continue wrestling and has pain with movement of the right shoulder. After returning from radiology, you review the films and diagnose a glenohumeral dislocation. After reducing the dislocation, what part of the physical exam MUST be done again? Answer Choices 1 Check the full active range of motion 2 Perform an apprehension test to make sure the joint is stable 3 Perform a neurologic check of the affected upper extremity 4 Check the motion of the elbow 5 Inspect the skin for open wounds

Perform a neurologic check of the affected upper extremity Explanation Axillary nerve palsy may develop with reduction of a glenohumeral dislocation. It is imperative that both pre- and post-reduction neurological exams be performed. Never do an apprehension test following a reduction or you may have to repeat the reduction. The arm should be placed in a sling after confirming the reduction with repeat radiographs (AP and transscapular views of the shoulder). Vigorous shoulder motion should be delayed for 3 months in anyone under 30 years old, but may begin at 6 weeks for those over 30 years old. Motion check of the elbow is important to do, but is unlikely to change from pre-reduction to post-reduction of the shoulder. Inspection of the skin for open wounds should have been performed at presentation and is unlikely to change with a reduction maneuver.

A 17-year-old male high school football player (punter) presents after being tackled and slammed onto his right dominant shoulder forcefully 2 hours ago during a game. He had immediate pain, but was able to continue punting. He has full active and passive range of motion, but some pain (6/10) with abduction. There is no obvious deformity and the skin over the shoulder is intact and not tented. What radiographic procedure should be ordered? Answer Choices 1 Plain AP and lateral of the affected shoulder 2 Plain AP of both shoulders 3 Plain AP of both shoulders with and without weights 4 Plain AP of the affected shoulder with and without weights 5 Plain AP of the affected shoulder with weights only

Plain AP of both shoulders with and without weights Plain AP of both shoulders with and without weights will likely show Type II and above AC separations. Type I is a sprain of the AC ligaments only and likely will not be seen on radiographs.

A 17-year-old male high school football player (punter) presents after being tackled and slammed onto his right dominant shoulder forcefully 2 hours ago during a game. He had immediate pain, but was able to continue punting. He has full active and passive range of motion, but some pain (6/10) with abduction. There is no obvious deformity and the skin over the shoulder is intact and not tented. What radiographic procedure should be ordered? Answer Choices 1 Plain AP and lateral of the affected shoulder 2 Plain AP of both shoulders 3 Plain AP of both shoulders with and without weights 4 Plain AP of the affected shoulder with and without weights 5 Plain AP of the affected shoulder with weights only

Plain AP of both shoulders with and without weights Explanation Plain AP of both shoulders with and without weights will likely show Type II and above AC separations. Type I is a sprain of the AC ligaments only and likely will not be seen on radiographs.

A 24-year-old man presents due to increasing pain in his right groin and buttock and difficulty walking. The pain has been present and worsening for about 1 month. He further reports that he had quite significantly injured himself, also near his right hip in a fall, while snowboarding about 3 months ago. He did not seek care immediately and had difficulty ambulating for a week, then saw a chiropractor to "put his lower back back in". The pain and gait problems eventually improved. Now he is concerned because the pain has returned with no new history of trauma. He is otherwise healthy and takes no regular medicines. He denies any chronic health conditions and has an unremarkable family history for musculoskeletal and rheumatologic conditions. On physical exam, the patient was noted to walk into the exam room with a slight limp. Examination of the hip, buttock, and groin region reveals no edema, erythema, or ecchymosis. The exam does not produce one particular point of tenderness with palpation, but pain is elicited with right hip motions, particularly internal rotation. Question What diagnostic study, if found to show the "crescent sign", would confirm this patient's condition? Answer Choices 1 Arthroscopy 2 Bone biopsy 3 Nuclear bone scan 4 Plain radiography (X-ray) 5 Ultrasound

Plain X-ray This patient's condition is suspicious for avascular necrosis (or the more preferred term, osteonecrosis) of the hip. Avascular necrosis (AVN) is a relatively common complication, following a traumatic hip dislocation, when the vascular supply to the femoral head is damaged and bone death occurs. There are many theories about the exact mechanism, and other causes, such as vascular, congenital, and autoimmune diseases, have been implicated with osteonecrosis. Plain radiography (X-ray) is the preferred initial test, and if the "crescent sign" is seen, AVN is confirmed. The crescent sign is a lucent (white) appearing crescent-shaped region of the femoral head, which indicates bone structure collapse upon itself. Arthroscopyis a surgical procedure for viewing the joint through a small endoscope, introduced via a small incision. Arthroscopy can be done for a variety of hip joint disorders (as well as other joints), but does not have a strong role in osteonecrosis evaluation. The crescent sign is not associated with arthroscopy. Bone biopsy plays a role in the diagnosis and staging the extent of a suspected osteonecrosis. Characteristic findings can include dead bone cells and degenerative changes. A nuclear bone scan can be very helpful in diagnosing and staging osteonecrosis. In all but the mildest cases of osteonecrosis, a bone scan will show increased uptake at the bone site (often the femoral head). Ultrasound does not typically play any role in AVN.

A 47-year-old man presents with a history of worsening burning left heel pain for the last few weeks. He states that it is worse in the morning after getting out of bed and beginning to walk; it improves with exercise. He denies any foot numbness, tingling, or known injury. He states that he recently began a jogging regimen. On examination, Thompson test is negative. Question What is the most likely diagnosis? Answer Choices 1 Heel contusion 2 Calcaneal fracture 3 Tarsal tunnel syndrome 4 Achilles tendon rupture 5 Plantar fasciitis

Plantar fasciitis close Explanation The correct answer is plantar fasciitis, as it occurs as a result of excess stress on the Achilles tendon that attaches to the plantar fascia. Classic presentation is burning heel pain, which is worse when bearing weight after prolonged periods of rest (e.g., sleep). Heel contusion is a soft tissue injury resulting from a trauma. Heel contusions cause pain, but the pain is unlikely to be as severe as what is described in this case. Calcaneal fracture typically occurs after an injury, and the pain would likely be more consistent with weight bearing. Tarsal tunnel syndrome is due to entrapment of the posterior tibial nerve, and it presents with medial malleolus pain that may radiate. Pain is typically worse during sleep, walking, and dorsiflexion. Paresthesias are common. Achilles tendon rupture is often described as the feeling of being kicked or shot in the area of the Achilles tendon; subsequently, jogging and walking normally would not be possible, as the tendon is crucial for this movement.

A 72-year-old female presents with a 4-month history of pain and stiffness in her shoulders and hips. She identifies the pain being worse in the morning and aggravated with getting in and out of the car along with difficulty brushing her hair. She also reports malaise and a 10-pound weight loss over the past few months. Her blood work shows an erythrocyte sedimentation rate (ESR) of 74 mm/h. What is the most likely diagnosis? A Gouty arthritis B Bursitis C Fibromyalgia D Polymyalgia rheumatica E Septic arthritis

Polymyalgia Rhuematica D Polymyalgia rheumatica (D) is a clinical diagnosis based on pain and stiffness of the shoulder and pelvic girdle areas, frequently in association with fever, malaise, and weight loss. Because of the stiffness and pain in the shoulders, hips, and lower back, patients have trouble combing their hair, putting on a coat, or rising from a chair. Elevated ESRs are present in most cases being > 50 mm/h. The age group most affected is over 50 years old. Fibromyalgia (C) is not specific for only the shoulders and hips. Bursitis (B) would usually be specific to one location. Gouty arthritis (A) is usually monoarticular and is not commonly seen in the hips and shoulders on presentation. Septic arthritis (E) would be joint specific and more acute.

A 52-year-old woman with a body mass index of 35 presents for evaluation of back pain. The pain becomes worse at night and is accompanied by stiffness. For the past week, she has been experiencing night sweats and fever. She relates that she had a positive PPD test a year ago and did not follow up as directed. An X-ray of her lumbar spine reveals osteopenia and cortical breakdown of vertebral bodies L4 and L5. Question What is the most likely diagnosis? Answer Choices 1 Pott Disease 2 Cauda equina syndrome 3 Osteoarthritis 4 Compression fracture secondary to osteoporosis 5 Compression fracture secondary to obesity

Post disease The clinical picture is suggestive of tuberculosis of the spine (also known as Pott disease). The fact that she had a positive PPD and did not follow her treatment appropriately indicates that she has active tuberculosis of the spine. Clinical findings include fever, sweating, or prostration. Pain may be mild at onset, is usually worse at night, and may be accompanied by stiffness. Imaging of the lower spine may find bone atrophy, narrowing of the cortex, and enlargement of the medullary canal. Cauda equina syndrome will cause back pain but will have accompanied loss of neurologic function (bowel and bladder) and paresthesias, which are not present in this patient. Radiographic findings with osteoarthritis include narrowing of the joint space, osteophyte formation, sharpened articular margins, and lipping of marginal bone, which are not seen in this patient. Clinical findings of compression fracture secondary to obesity or osteoporosis include loss of height and backache of varying degrees of severity, which are not present in this patient.

A 66-year-old man presents with monoarticular arthritis. He has stage 3 chronic kidney disease. His affected joint is warm to the touch. You suspect gout. He is allergic to aspirin. What anti-inflammatory should you prescribe? Answer Choices 1 Acetominophen 2 Aspirin 3 Metformin 4 Ibuprofen 5 Prednisone

Prednisone In this case, you are looking for an anti-inflammatory agent that will not cause a decline in renal function. Prednisone is recommended. Aspirin has anti-inflammatory properties, but it is contraindicated because the patient is allergic to it. Acetaminophen could be use for analgesia, but it is unlikely to decrease inflammation. Metformin is a medication used for the treatment of diabetes. Ibuprofen is an anti-inflammatory and anti-pyretic agent. Ibuprofen will likely cause a decline in renal function; it and other non-steroidal medications block prostaglandin-mediated vascular relaxation, effectively vasoconstricting and decreasing glomerular filtration rate and renal function. Approximately 30% of patients exposed to non-steroidal medications will have a decline in renal function. Patients with pre-existing kidney disease may be more susceptible to a decline in function, as they may be more dependent on the above-described prostaglandin-medicated vascular tone.

A 55-year-old woman presents with weakness, bone pain, and lethargy. Serological tests reveal the presence of Russell bodies. The accumulation of what intracellular material is most likely responsible for the presence of Russell bodies? Answer Choices 1 Glycogen 2 Carbohydrate 3 Endogenous pigment 4 Exogenous pigment 5 Protein

Protein Russell bodies are the accumulation of immunoglobulin in plasma cells; immunoglobulins are proteins. Occasionally, there can be abnormal cellular accumulations of glycogen and carbohydrates; this can be seen in glycogen storage diseases. Endogenous pigments would include hemosiderin and lipofuscin. Exogenous pigments can be seen with numerous pneumoconioses.

A 65-year-old man presents with complaints of acute onset of pain and swelling of the right great toe. He denies recent alcohol ingestion or trauma to the area. On physical examination, the patient is afebrile, and the first metatarsophalangeal joint is erythematous, swollen, and warm to the touch. Laboratory evaluation reveals a WBC (white blood cells) count of 12,000/μL and a normal differential. Serum uric acid level is found to be 5 mg/dL. Synovial fluid analysis reveals the presence of rhomboid-shaped crystals. Which of the following is the most likely diagnosis? A acute gout B pseudogout C psoriatic arthritis D infectious arthritis E rheumatoid arthritis

Pseudogout Pseudogout presents similarly to acute gout and is best diagnosed by the finding of the rhomboid-shaped crystals of calcium pyrophosphate in joint aspirates. Joints commonly involved in pseudogout are the knees and wrists and other joints such as the metacarpophalangeals, hips, shoulders, ankles, and elbows. The diagnosis of pseudogout is further supported by the finding of a normal serum uric acid level. Acute gout would more likely be associated with an elevated serum uric acid level. Psoriatic arthritis commonly presents with asymmetrical oligoarticular involvement of two to four joints, and in a higher percentage of patients, there is known presence of the dermatological expression of psoriasis. Infectious arthritis is ruled out with the findings of an afebrile patient and WBC count of 12,000/μL. In acute infectious arthritis, the WBCs would be expected to be elevated in the range of 50,000 to 200,000/μL. Rheumatoid arthritis usually presents with symmetrical polyarticular involvement of three or more joints.

An 8-year-old boy sustained a puncture wound to his right foot 4 days ago. He was playing and stepped on a nail that went through his sneaker. His mother said the wound bled profusely but the nail did not go completely through his foot. They washed the wound at home with soap and water, wrapped it in a bandage, and did not seek further care. This morning, he complained that it was very painful and his mother noted that his foot looked red and swollen. On exam, his temperature is 99°F; pulse is 114 BPM, and his BP is 104/68 mm Hg. The plantar surface of his right foot has a small 2 mm scabbed entry wound that is surrounded by a 5 - 6 cm area that is erythematous, swollen, and quite tender. There is a scant amount of thin, seropurulent material from the entry wound on examination. You are worried about possible osteochondritis. What bacteria would most likely cause this complication? Answer Choices 1 Streptococci 2 Eikenella 3 Pasturella 4 Clostridium 5 Pseudomonas

Pseudomonas Pseudomonas aeruginosa causes more than 90% of foot osteochondritis in plantar puncture wounds. It should be considered as a likely pathogen when there is a penetrating injury through footwear, especially sneakers or from soil and manure contamination. It is believed that the penetrating object may push particles of foam contaminated with bacteria into the wound. In addition, anaerobes, Klebsiella, Bacteroides, Serratia, and Salmonella sp. are less common organisms in this setting that may also cause secondary infections. Streptococci (as well as Staphylococci) are the most common causes of secondary infection of ordinary puncture wounds. Secondary infections will occur in 6% - 10% of non-bite puncture wounds in addition to retention of a foreign body. Eikenella should be considered as a possible dog bite pathogen after staphylococci and streptococci. Pasturella is a common dog and cat bite pathogen. P. multocida is responsible for 60%-80% of infections following a cat bite. Though dog bites are 8 - 10 times more common than cat bites, the rate of infection is much greater from cat bites, approximately 10% - 20% for dog bites and 50% from cat bites. Clostridium is a common anaerobic bacterium that may complicate human bites along with Streptococci and Staphylococci. Eikenella corrodens should also be considered and is found primarily in adults but can also be a significant pathogen in children. Due to high risk of secondary infection, prophylactic antibiotic to cover Gram-positive and anaerobic bacteria are generally recommended following deep bite wounds through the dermis or to the hand, along with close follow up.

A 29 y/o male IV drug abuser shows up at your emergency facility transported by Miami Fire Rescue. He has had a 1 day history of fever, and a very swollen, and painful left knee. The swelling started yesterday just before the chills and fever. Physical examination show limited passive and active motion in that knee and an otherwise unremarkable physical examination. 45 cc of purulent fluid. Gram stain shows gram-positive cocci in clusters. Q1. The MOST likely diagnosis is A. Septic arthritis (gram +/-) B. Gonococcal arthritis C. Crystalline arthritis D. Psoriatic arthritis E. Reactive Arthritis(Reiters Syndrome) Q2. In this patient which of the following is the likely cause of his septic joint infection? A. Staphylococcus aureus B. Streptococcus pyogene C. Bacillus cereus D. Campylobacter Q3. Treatment should include hospital admission & which of the following? A. High dose antibiotics by mouth and joint aspiration on an as needed basis to relive pain and allow motion B. High dose IV antibiotics and splinting the affected joint to prevent spread of the infection by the vascular system (hematogenous spread) C. IV antibiotics of standard dosing for an adult D. Daily joint aspiration, followed by intraarticular injection of antibiotics and IV antibiotics

Q1 A. Septic Arthritis Q2. A. Staph aureus Q3. D. Daily joint aspiration, followed by intraarticular injection of antibiotics and IV antibiotics

A 50-year-old woman has had Raynaud's phenomenon of the hands for 15 years. The condition has become worse during the last year, and she has developed arthralgias and arthritis involving the hands and wrists as well as mild sclerodactyly and difficulty swallowing solid foods. Physical Exam of the face shows shiny, taut skin and no facial wrinkles Laboratory studies reveal a positive serum antinuclear antibody assay at a dilution of 1:160. Anticentromere antibodies are present in high titers; antiribonucleoprotein antibodies are not detectable Q1. The most likely diagnosis of this woman's disorder is A. Systemic sclerosis B. MCT disease C. Overlap syndrome D. Dermatomyositis E. SLE Q2. If this patient with diffuse cutaneous scleroderma (systemic sclerosis) develops severe hypertension she is at significant risk of dying from A. Thrombotic stroke B. Central nervous system hemorrhage C. Renal failure D. Pulmonary hypertension E. Pulmonary fibrosis

Q1 A. systemic sclerosis (Scleroderma) Q2 C. Renal Failure ( HTN--> renal failure is the leading cause of death) shiny taut skin in women who has no facial wrinkles Limited disease: CREST Syndrome raynaud's skin involvement limited to fingers, extremities distal to elbows and face Diffuse disease: rapid development of symmetric skin thickening or proximal and distal extremities, face and trunk.

A 20-year-old woman has developed low-grade fever, a malar rash, &n arthralgias of the hands over several months. High titers of anti-DNA antibodies are noted, & complement levels are low. The patient's white blood cell count is 3000/mL, and platelet count is 90,000/mL. The patient is on no medications and has no signs of active infection. Urinalysis is performed that shows hematuria and proteinuria. Q1. This patient probably has A. Early RA B. SS C. SLE D. Wegener's Granulomatosus E. Mixed connective-tissue disease (MCTD) Q2. Which of the following is NOT one of the criteria of SLE by the American College of Rheumatology? A. discoid rash B. endocarditis C. oral ulcers D. serositis E. renal disorder Q3. Which of the following statements is correct about SLE A. If glomerulonephritis, severe thrombocytopenia, or hemolytic anemia develops, high-dose glucocorticoid therapy would be indicated B. CNS symptoms will occur within 10 years C. All patients can be expected to develop Raynaud's phenomenon when exposed to cold D. Having the disease is an absolute contraindication to pregnancy E. All are correct

Q1. C. SLE Q2. B. Endocarditis Q3. A. . If glomerulonephritis, severe thrombocytopenia, or hemolytic anemia develops, high-dose glucocorticoid therapy would be indicated

A 60-year-old female complains of dry mouth and a gritty sensation in her eyes. She states it is sometimes difficult to speak for more than a few minutes. There is no history of diabetes mellitus or neurologic disease. The patient is on no medications. On exam, the buccal mucosa appears dry and the salivary glands are enlarged bilaterally. Q1. This patient probably has A. Sjogren's Syndrome B. SS C. SLE D. Wegener's Granulomatosus E. Mixed connective-tissue disease (MCTD) Q2. The next step in evaluation is A. Lip biopsy B. Schirmer test and measurement of autoantibodies C. IgG antibody to mumps virus D. Use of corticosteroids

Q1. Sjorgren Syndrome Q2. Schirmer test and measurement of autoantibodies Sjorgren's is MC in mid aged women It is a progressive lymphocytic destruction of exocrine glands most frequently resulting in symptomatic eye and mouth dryness

A 28-year-old man is brought in after being involved in a motor vehicle accident. The patient was the driver in the car and was hit head on during a wrong way crash. After initial survey of ABCs is done and the patient is ruled stable, a secondary evaluation reveals that the patient's left ankle is painful. During inspection, it is found that there is localized pain and swelling over the medial aspect of the left ankle. The patient is able to walk with this ankle, but a mild limp is present. Pulses are palpable. Question Based on the history and physical examination, what is the best clinical intervention at this time? Answer Choices 1 RICE approach 2 Immediate physical therapy 3 Emergent orthopedic referral 4 Observation only 5 Placement of a cast

RICE approach Explanation Eversion ankle sprains will present with localized pain and swelling over the medial aspect of the ankle only. In this type of sprain there is a tear of the deltoid ligaments on the medial side of the ankle. The patient will likely have difficulty weight bearing, but he or she would still attempt to walk, although he or she would most likely limp. Sprains in general are related to injuries involving the stretching or tearing of a ligament and may be approached as a Grade I (Mild), Grade II (Moderate), or Grade III (Severe). Most likely the injury above is a Grade I Mild ankle sprain. Treatment of this type of injury would consist of using the RICE method: rest, ice, compression, and elevation. Following these actions would most likely allow this injury to heal efficiently and effectively. I mmediate physical therapy on its own would not be optimal treatment at this point in time; if the patient appropriately uses the RICE method, improvement may be seen as soon as 24 -48 hours after the initial injury. Observation only would most likely not allow the injury to heal as well. An emergent orthopedic referral is not needed at this time; there are no visible or substantial musculoskeletal injuries and the pulses are present. A cast placement is not necessary unless there is the presence of a fracture or a Grade III type of strain.

A 24-year-old woman enjoys participating in an amateur karate class. While taking part in a tournament, she uses her right upper arm to block a high roundhouse kick and is injured. She develops a right wrist drop. What nerve was injured? Answer Choices 1 Radial nerve 2 Ulnar nerve 3 Median nerve 4 Axillary nerve 5 Musculocutaneous nerve

Radial nerve A radial nerve injury can cause a wrist drop. The radial nerve innervates the skin and posterior surface of the upper extremity. The radial nerve innervates the muscles of the back of arm, forearm, and hand. If the muscles of the posterior compartment of the forearm are paralyzed, as occurs with radial nerve damage, there will be wrist drop. The ulnar nerve innervates the skin of the medial third of the hand. The ulnar nerve innervates some of the flexors in the anterior forearm and most of the intrinsic muscles of the hand. The median nerve innervates the skin on the lateral 2/3 of the hand and fingers on the palmar side and the dorsum of digits 2, 3, and some of 4. Most of the muscles of the anterior forearm and some of the muscles of the hand are innervated by the median nerve. The axillary nerve innervates some to the skin in the shoulder region and the back of the arm. The deltoid and teres minor are innervated by the axillary nerve. The musculocutaneous nerve innervates some of the skin on the lateral forearm. The muscles innervated by the musculocutaneous nerve are the brachialis, biceps brachii, and coracobrachialis.

Parents bring their 4-year-old daughter in because of knock-knee. She is otherwise healthy, and her height is in the 50th percentile for age. On examination, she has about 10 degrees of valgus. What should you recommend? Answer Choices 1 X-rays of the lower extremities 2 Reassurance that this is normal for age 3 Orthopedic shoes 4 Long leg braces 5 Lab tests for markers of bone turnover

Reassurance that this is normal for age Children are normally bowlegged (genu varum) at birth. By 12 to 18 months of age, the legs become straight. Then children develop knock-knee (genu valgum), which is maximal by about age 3-4 years. This averages 10-15 degrees at this age. The legs then spontaneously "straighten" to 5-10 degrees of valgum, which is the average for adults. This child is within the normal range, and the parents should be reassured. Because virtually all of these children improve spontaneously, they do not need special shoes or braces. Because this child is not short, she does not require additional work-up such as X-rays of the lower extremities or laboratory tests for serum phosphorus, vitamin D levels, calcium, and parathyroid hormone levels. If she was short and had other physical findings, rickets would be in the differential diagnosis and further work-up would be indicated.

A 33-year-old man with no significant past medical history presents with a 2-month history of persistent right knee pain. The knee pain is located in the anterior part of the knee, but according to the patient, "behind the knee cap." Pain is worse as he descends stairs, performs squatting maneuvers, and sits for excessive periods of time. He is an avid runner and states that running also increases pain. He denies any trauma, falls, accidents, or prior surgeries. He further denies any fever, chills, insect bites, rashes, effusions, grinding, popping, or clicking sensations in the knee. He denies any hip or ankle pain. The physical exam reveals tenderness to palpation along the medial undersurface of the right patella and a positive patellar apprehension test. The anterior and posterior draw tests, McMurray's test, and Apley's compression and distraction tests are all negative. Question What is the most appropriate clinical intervention for this patient at this time? Answer Choices 1 Recommend weight gain 2Reduce running exercises 3 Prescribe narcotic analgesia 4 Referral for knee arthroscopy 5 Cast immobilization

Reduce running exercises Explanation This patient's most likely diagnosis is patellofemoral pain syndrome. Clinical interventions include relative rest, a reduction of running pace and mileage, and avoidance of squats, lunges, and running uphill and downhill. Other interventions include quadriceps strengthening and NSAID medications. Surgical interventions are indicated if symptoms persist beyond 6 months, if a rehabilitation program fails, and if other causes of knee pain have been excluded. Weight loss is recommended to decrease the stress on the patellofemoral joint. Knee orthotics may be beneficial.

A 73-year-old man presents with the inability to actively raise his left non-dominant arm to retrieve plates from the kitchen cabinet. He has a long-standing history of chronic shoulder impingement syndrome. On exam he is found to have a (+) positive drop arm test. What treatment is recommended for the most likely diagnosis? Answer Choices 1 An immediate subacromial steroid injection 2 Schedule for surgical repair immediately 3 Order MRI 4 Refer for physical therapy 5 Place in shoulder immobilizer

Refer for physical therapy Explanation Refer this elderly patient with rotator cuff tear to physical therapy to regain range of motion by stretching and strengthening the remaining rotator cuff. If the patient is younger and has a sudden rotator cuff tear, they should be referred to an orthopedic surgeon for immediate repair. There is little chance this gentleman will need repair. Although MRI will show the tear, little clinical information will be gained unless he becomes a candidate for surgery. Injection of a steroid may improve his painful symptoms, but it also increases the risk of worsening the tear by weakening the tendons. Do NOT place this elderly patient in a shoulder immobilizer, as doing so will decrease the chance of obtaining return of function by making the shoulder joint stiff.

A 12-year-old girl is referred to you following a screening for scoliosis at school by the school nurse. The nurse used a scoliometer that revealed a reading >5 degrees and felt radiographs were needed at this point. The girl is totally asymptomatic of any back pain or motion or activity limitations. You order a standing AP and lateral long cassette spine and use the Cobb method to measure your curvatures. You measure and find a thoraco-lumbar single curve of 25 degrees with a Risser stage of 3. Question What is recommended for this patient? Answer Choices 1 Brace immediately 2 Recheck radiographs in 1 year 3 Repeat radiographs in 3 months 4 Refer for spinal surgical stabilization 5 No need for further follow up since she is now skeletally mature

Repeat radiographs in 3 months Explanation For progressive curves in a patient with Risser stage of 3, it is recommended that you repeat radiographs in 3 months. 1 year is too long to wait in an actively growing child. Bracing is reserved for curves >30° in a patient with Risser stage of 3. Surgical referral is not needed until idiopathic curves reach 50° or for curves of 40° to 50° that are likely to progress. This patient should be followed up with every 3 months until skeletally mature.

A 12-year-old boy is brought in by his mother following a fall on his right outstretched hand from the top half of a slide an hour ago. He denies any loss of consciousness. There is deformity, pain, and decreased motion of the elbow due to pain, as well as pain down the forearm into the wrist area. There is no shoulder or neck pain. Which of the following radiographs should be ordered? Answer Choices 1 Right elbow AP and Lateral 2 Right elbow AP and Lateral, Left elbow AP and Lateral 3 Right elbow AP, Lateral, and Obliques 4 Right elbow AP and Lateral, Right forearm AP and Lateral, Right wrist AP & Lateral 5 Right and Left elbow AP and Lateral and right wrist AP and Lateral

Right and Left elbow AP and Lateral and right wrist AP and Lateral Explanation In pediatric patients, there are multiple growth centers in the elbow. Therefore, it is recommended to take the contralateral elbow to compare the anatomic positions of the growth centers along with the injured elbow AP & Laterals. The Wrist AP & Lateral should also be taken because of the possible injury to the wrist.

A 28-year-old woman presents with severe ankle pain. An evaluation reveals the patient is unable to move her ankle. She states that it is exquisitely tender, and the pain worsens with movement or weight bearing. She refuses to walk with the foot or put any weight on the leg. Upon inspection, the right ankle is found to be remarkably painful to palpation. The skin is a healthy color and there are no signs of lacerations or tenting. Pulses are palpable. The entire right ankle is swollen and appears dislocated. Question Given the history and physical examination, what is this patient most likely suffering from? Answer Choices 1 Right inversion ankle sprain 2 Right eversion ankle sprain 3 Right ankle strain 4 Right ankle fracture 5 Right ankle hematoma

Right ankle fracture Explanation The correct answer for this patient scenario is a right ankle fracture. Since there is significant swelling bilaterally, both the tibia and the fibula are very highly likely to be fractured. Motor vehicle accidents are one of the most common causes of ankle fractures in adults. Key features of an ankle fracture include pain, swelling/signs of bruising after the traumatic event, apparent deformity, and the inability to bear weight; the patient in this case fits all of these components. Inversion ankle sprains will present with localized pain and swelling over the lateral aspect of the ankle only. The patient would have difficulty weight bearing and would still attempt to walk, although the patient would most likely limp. Eversion ankle sprains are also termed high ankle sprains. This injury is commonly acquired after the foot being turned out or externally rotated and everted, as during a tackle. Pain associated with this sprain is typically worse than with other sprains. Mild swelling or even an effusion may be present. No dislocation is apparent. Ankle strains involve the stretching or tearing at the junction where the muscle meets the tendon (sprains have to do with stretching or tearing of the ligament). Acute strains will present with pain, muscle spasms, loss of strength, and potentially limited range of motion. Strains take place with the muscle is stretched and suddenly contracts, as is the case in running or jumping. Hematomas typically cause some discomfort or pain, but they rarely cause such severe issues as dislocation of the ankle or the inability to bear weight.

A 21-year-old man presents with a 3-month history of pain in his left shoulder. He is a left-handed pitcher for his college baseball team. The pain began when he started spring training a few months ago, and it has gradually worsened since that time. In addition to the pain, the patient is now having difficulty lifting his left arm above his head. He denies any injury to the shoulder itself, and he states that ibuprofen provides some relief. On physical exam, the patient has tenderness to palpation of the lateral left shoulder just under the acromion, limited abduction of the left shoulder, a negative 'drop arm' sign, and a negative 'crossover test'. Question What condition best explains the patient's symptoms? Answer Choices 1 Bicipital tendinitis 2 Acromioclavicular arthritis 3 Rotator cuff tendinitis 4 Rotator cuff tear 5 Adhesive capsulitis

Rotator cuff tendinitis Explanation The patient above is most likely suffering from rotator cuff tendinitis. Distinguishing between the various soft tissue disorders that can cause shoulder pain (tendinitis, subacromial bursitis, and tears) can be difficult because these conditions can cause similar symptoms and can often co-exist. However, knowledge of musculoskeletal anatomy and physiology can help in making a more precise diagnosis. The muscles of the rotator cuff include the supraspinatus, infraspinatus, teres minor, and subscapularis, otherwise known as the SITS muscles. The tendons of 3 of the rotator cuff muscles form a musculotendinous unit where they insert into the proximal humerus. Repetitive movements of the shoulder, such as throwing or swimming, can lead to inflammation of the tendons of the rotator cuff muscles. The most commonly affected muscle is the supraspinatus, which inserts directly under the acromion. Therefore, a patient with a rotator cuff tendinitis, in addition to the inability to lift the arm above shoulder level (abduction and external rotation), will have tenderness with palpation of the area just under the acromion on the affected side. The "drop arm" sign is elicited by having the patient fully abduct the affected arm to shoulder level, or up to 90 degrees, and then lower it slowly. If the patient is unable to hold the arm fully abducted, this is a "positive drop arm" test. Though the "drop arm" sign is used to assess the status of the rotator cuff, a positive drop arm sign indicates an actual complete tear in the rotator cuff, not just tendinitis. The patient in the presentation above had a "negative drop arm" test. Bicipital tendinitis, inflammation of the long head of the biceps tendon, can also present with anterior shoulder pain that may resemble rotator cuff tendinitis and may coexist with it. However, in bicipital tendinitis, tenderness is usually maximal in the bicipital groove along the anterior aspect of the humerus. Inflammation or arthritis of the acromioclavicular (AC) joint of the shoulder is not a common cause of shoulder pain. However, when it is present, it is usually the result of direct injury to the shoulder with resulting degenerative changes. This condition usually presents with localized tenderness over the AC joint or pain with adduction of the affected shoulder. The "crossover test" is also used to assess the AC joint. Therefore, a "positive crossover test" is when adduction of the patient's affected arm across the chest elicits pain. The patient above had a "negative crossover test." Adhesive capsulitis usually occurs in patients 50 - 70 years of age, and it refers to fibrosis of the glenohumeral joint capsule. It is manifested by diffuse, dull, aching pain in the shoulder and progressive restriction of active and passive range of motion. There is usually no localized tenderness on exam.

A 35-year-old man presents with right shoulder pain that is becoming progressively worse. Although he visits the gym 3 times a week, over the past month he has not been able to increase the amount of weight he lifts secondary to the shoulder pain. He has not tried anything to alleviate the pain. He reports that the pain is at its worst at night while he is trying to sleep. He also reports pain while in the shower washing his hair, or when using the shoulder press machine at the gym. He denies any history of recent trauma or sports related injury. However, upon questioning, he reports that about 1 month ago he and his wife painted their entire house in 1 weekend. On exam, there is no notable swelling, atrophy, redness, or bruising of the shoulders. Point tenderness is noted over the right lateral deltoid muscle. Active ROM of the right shoulder at 80 degrees of abduction elicits pain. Patient has a negative drop arm test, negative apprehensive test, and a positive Neer impingement test of the right shoulder. Question What is the most likely diagnosis? Answer Choices 1 Acromioclavicular sprain 2 Adhesive capsulitis 3 Fracture of the clavicle 4 Glenohumeral dislocation 5 Rotator cuff tendonitis (tendinopathy)

Rotator cuff tendonitis (tendinopathy) The most likely diagnosis is rotator cuff tendonitis (tendinopathy). It is associated with pain and difficulty abducting or rotating the arm. Degree of trauma depends on the type of injury. With full thickness tears, the degree of trauma is significant (e.g. a major fall, MVA, or shoulder dislocation). Partial thickness tears can be caused by lesser degree of trauma, and overuse injuries can cause tears, as well. In addition, tendonitis injuries are often associated with overuse. Symptoms are usually pain and weakness made worse by overhead activities, overhead abduction, and external rotation. Pain is usually located over the anterior and lateral aspects of the shoulder and may radiate into the lateral deltoid. The pain may worsen with sleeping on the affected extremity. The patient may describe a "catch" as the arm is brought into the overhead position. In addition, a typical painful arc usually occurs between 70 degrees and 110 degrees of abduction along with a positive impingement test, which is forced forward elevation of the arm. The pain results as the greater tuberosity impinges on the acromion. The drop test is associated with rotator cuff tears. Acromioclavicular sprain is common in younger persons and athletes. The classic cause is the direct blow to the acromion with the humerus in the adducted position. This drives the acromion medially and inferiorly. Falling on an outstretched hand or elbow can also cause this injury. A cross body adduction test is positive with this condition. Adhesive capsulitis is often associated with patients with diabetes and thyroid disorders. It can result from immobility following a shoulder injury. Patients have slow, gradual onset of shoulder pain that can be severe, especially at night. The pain is generalized and referred to the superolateral aspect of the shoulder and upper arm. Examination reveals no swelling at the glenohumeral joint. Passive and active ROM is greatly limited in all movements. Fracture of the clavicle is one of the most common injuries. It is usually caused by a fall on the lateral aspect of the shoulder. Less commonly, it is caused by a direct blow or by falling on the outstretched hand. Although complications are uncommon, they can occur. With this type of injury, lung and neurovascular compromise can occur. AP X-ray is the best way to ascertain a clavicle fracture. Glenohumeral dislocations in younger individuals are most commonly caused by direct trauma and sports injuries. Patients with this type of injury are usually unwilling to move the affected arm, and they tend to cradle it with the unaffected arm. Anterior shoulder dislocations account for about 80-90% of all dislocations. An excessive external rotation or abduction force usually causes this type of injury. A bulge may be noticeable where the humeral head rests, with an emptiness beneath the acromion. Posterior shoulder dislocation usually happens when the humeral head is driven posteriorly with force. Usually, the patient experiences a positive apprehensive test.

A 23-year-old woman presents with a rash and swollen joints. She had been healthy previously, and the only medication she takes is acetaminophen. A review of systems includes recent fevers and a 5-pound weight loss in the past month; she is also experiencing photosensitivity and hematuria. She denies oral ulcers, nasal congestion/discharge, ear pain, pleuritic symptoms, chest pain, neural symptoms, bruising, and bleeding. On physical exam, her temperature is 101 degrees Fahrenheit; her blood pressure is 130/85 mmHg. She has a malar rash as well as diffusely swollen, warm, and tender joints. Her cardiopulmonary exam is normal. She has no costovertebral angle tenderness. Trace bipedal edema is noted. Diagnostic studies reveal: Erythrocyte sedimentation rate 50mm/hour Hemoglobin 11 g/dL Anti-double-stranded DNA antibodies Positive Antihistone antibody Positive Platelet count 200,000 cells/ml White blood cell count 12,000 cells/mm3 Chest x-ray Normal Urinalysis No bacterial growth. Numerous dysmorphic red blood cells. 10 white blood cells/high power field. Red blood cell casts. 510 mg albumin/g creatinine. Complement 3, 4 Both reduced Anti-Neutrophil Cytoplasmic Antibody Negative Question What is the most likely diagnosis? Answer Choices 1 Drug-induced lupus 2 Systemic lupus erythematosus 3 Wegener granulomatosis 4 Microscopic polyangiitis 5 Henoch-Schönlein purpura

SLE Explanation This woman most likely has Systemic lupus erythematosus (SLE), given her symtpoms of hematuria, photosensitivy and arthritis, plus the findings of urinary red cell casts , proteinuria, and hypocomplementemia Penal involvement is frequent in SLE, and it may include a variety of glomerular diseases. Hypocomplementemia may occur due to complement consumption in the glomeruli. Focal proliferative and diffusely proliferative glomerulonephritis are 2 forms of glomerulonephritis that might be seen in this patient if biopsied. If proven, she might be treated with immunosuppressive therapy, such as prednisone and cyclophosphamide. Patients with SLE nephritis may recover, relapse, progress to a different form of lupus nephritis, or progress to end stage renal disease. Drug-induced lupus does not typically cause renal or neural involvement. A variety of drugs cause a lupus-like picture with positive anti-histone antibodies, but no anti-double-stranded DNA antibodies are seen; moreover, normocomplementemia is noted in this condition. Wegener's granulomatosis is a small vessel vasculitis of unknown pathogenesis, which causes inflammation and necrotizing granulomas in the upper and lower respiratory tract and kidneys as well as a rash. Palpable reddish-purple lesions on the lower extremities and positive anti-neutrophil cytoplasmic antibody (ANCA) should raise suspicion of vasculitis. Renal involvement is very common; necrotizing glomerulonephritis with negative immune staining may be seen on renal biopsy. Microscopic polyangiitis/polyarteritis is a small vessel vasculitis and is a differential cause of rash, arthralgias, and renal failure. Arthritis (true joint swelling noted on physical exam) is not usually noted. Patients may be ANCA positive. Henoch-Schönlein purpura (HSP) is a vasculitic syndrome seen more often in children than adults; it may cause petechiae, purpura, gastrointestinal bleeding, and hematuria/glomerulonephritis. Complement counts and anti-double-stranded DNA are normal, and elevated IgA antibody levels may be seen. In a classification scheme revised by the Systemic Lupus International Classification Clinic( SLICC) in 2012, patients who have "4 SLICC criteria including at least one immunological criterion( ie elevated ANA) and at least one clinical criteria( ie malar rash) or biopsy-proven nephritis compatibile with systemic lupus in the present of either ANA or antids-DNA" can be classified as having systemic lupus erythematosus( Petri). Clinical and immunological criteria from reference 10 are listed below. Please see reference 10 for complete details.

Membranous glomerulonephritis is associated with which of the following conditions? Answer Choices 1 Heroin abuse 2 Patients with AIDS 3 Systemic lupus erythematosus 4 Vesicoureteral reflux 5 Massive obesity

SLE Explanation The causes of membranous glomerulonephritis are enumerated as follows: (Renal vein thrombosis is frequent.) * Chronic infections include malaria, hepatitis B * Systemic diseases like SLE * Heavy metals like Hg, Gold intoxications * Drugs like penicillamine, captopril * Solid tumor like melanoma, lung cancer, etc. The causes of focal segmental glomerulosclerosis are enumerated as follows: (Renal vein thrombosis is infrequent.) * Vesicoureteral reflux * HIV infected patients * Obesity * Heroin addict * Solitary or ablated kidney

A 5-year-old boy presents with an ankle injury. While at his school's playground, he was playing on the swings and decided to jump off. An X-ray of the ankle demonstrates a fracture line through the metaphysic plate of the superior aspect of the distal fibula, ending bfore the epiphyseal plate. What type of Salter-Harris fracture does this child have? Answer Choices 1 Salter-Harris type I 2 Salter-Harris type II 3 Salter-Harris type III 4 Salter-Harris type IV 5 Salter-Harris type V

Salter Harris type 2 The injury described is classified as a Salter-Harris type II growth plate fracture. In a Salter-Harris type II fracture, the fracture lies above the growth plate, sparing the epiphysis. Salter-Harris is a system used to classify factors affecting the growth plate, or 'physis'. 5 types have been characterized: Type I is a transverse fracture through the growth plate (physis). Type III is a fracture through the growth plate and epiphysis, sparing the metaphysis. Type IV is a fracture through all 3 elements of the bone: the growth plate, metaphysis, and epiphysis. Type V is a compression fracture of the growth plate resulting in a decrease in the perceived space between the epiphysis and diaphysis on X-ray.

A 91-year-old man is referred to you by an orthopedic surgeon for follow-up after hospitalization. He had been admitted 14 days prior after he fell down a flight of stairs and was found to have fractured his left femoral neck. After operative fixation, he developed pneumonia, requiring intensive care unit admission and ventilatory support for 3 days. He had previously been well for his age; he was able to walk slowly for more than a half mile, and he not taking any medications before his accident. Now, he is unable to walk unaided due to marked weakness. He denies any pain. On physical examination, he is afebrile and has a mini-mental state examination (MMSE) score of 28/30. His pulse is 80 bpm and regular. Heart sounds are normal with no murmurs, gallops, or rubs. Examination of the chest, abdomen, CNS, and skin are unremarkable. He had no significant signs of joint disease other than the recent hip fracture. Musculoskeletal exam is remarkable for quadriceps femoris muscle wasting bilaterally, and he was not able to rise against gravity from a chair. Motor strength is 3/5 in all other muscle groups. Diagnostic investigations are as follows: Hemoglobin 11.9 g/dL (13-18) MCV 89 fL (80-96) Total white cell count 9.1 × 109/L (4-11) Serum urea 50 mg/dl (15-45) Serum sodium 139 mEq/L (137-144) Serum potassium 3.4 mEq/L (3.5-4.9) Serum albumin 29 g/L (37-49) Serum total bilirubin 1 mg/dL(0.3 to 1.9 ) Serum ALT 20 U/L (5-35) Serum alkaline phosphatase 305 U/L (45-105) Serum CK 74 U/L (24-195) ESR 27 mm/h (<15) ECG normal Question What principle is the underlying reason for this patient's weakness? Answer Choices 1 Anemia 2 Disuse atrophy 3 Hypokalemia 4 Protein depletion 5 Sarcopenia

Sarcopenia Explanation In a 91-year-old man, the most likely background reason for his current generalized weakness is sarcopenia. Sarcopenia (from the Greek meaning "poverty of flesh") is the degenerative loss of skeletal muscle mass and strength associated with aging. Reduction in muscle mass occurs universally with aging and becomes particularly noticeable above the age of 80. By definition, sarcopenia is a reduction in muscle mass to less than 2 standard deviations below the mean for a young healthy reference group. Of important note, sarcopenia is a component of the frailty syndrome. The frailty syndrome is a collection of symptoms or markers, primarily due to the aging-related loss and dysfunction of skeletal muscle and bone that place older adults at an increased risk of adverse events such as death, disability, and institutionalization. Its main three components are sarcopenia, osteoporosis, and muscle weakness. Hypokalemia can in fact cause muscle weakness; however, in this patient, the mild degree of hypokalemia reported would not be enough, in itself, to explain his severe weakness, though it might be a contributing factor and thus should be corrected. The low serum albumin is evidence that the patient is truly protein depleted, as severe acute illness causes protein wasting. However, although this might be making some contribution through exacerbating the muscle wasting, it is not the principal cause in this case. For example, by comparison, a younger patient having suffered a similar illness and with the same serum albumin and potassium levels would not be expected to be too weak to rise from a chair. Some acute disuse atrophy could be contributing in this patient, though the illness was only of 14-days duration. From the patient's history, it would appear that he is normally sufficiently active to have prevented severe disuse atrophy over a prolonged time period. Elderly patients who are bedridden or chair-bound would be more likely to fall into that category. The patient is not sufficiently anemic to affect static muscle power.

A 4-year-old girl presents after falling on an outstretched right dominant arm with her elbow fully extended from a slide (about 4 feet fall) 30 minutes ago. There was no loss of consciousness, but there was immediate extreme pain, with an obvious deformity at the elbow. On presentation, the girl is holding her right elbow, and she will not allow you to passively move it. What would you expect to find on exam? Answer Choices 1 No swelling is associated with this injury 2 Full passive range of motion 3 Severe pain with flexion of the elbow 4 No pain with extension of the elbow 5 Loss of axillary nerve sensation and motor innervation

Severe pain with flexion of the elbow The correct response is severe pain with flexion of the elbow. This is the classic age and mechanism of injury for a supracondylar humerus fracture. Pain is worse with flexion or extension, especially in displaced fractures, as indicated by the deformity in this child's case. The axillary nerve would not be injured here since it is proximal to the elbow and rarely involved. This injury typically has a large effusion or edema and ecchymosis, and many times is deformed (if fracture is displaced) and has crepitus. Patients will not usually allow you to passively move the elbow.

A 47-year-old grossly obese woman presents with left non-dominant shoulder pain and limited range of motion. The pain began about a month ago, and her shoulder has progressively lost range of motion since it started. She cannot reach overhead with that arm. There is no history of trauma, it does not wake her at night, and she can sleep on the left side. Her past medical history is significant for Diabetes Type I and hypothyroidism. She is on regular insulin and NPH insulin as well as Synthroid (levothyroxine). On exam, she is tender to palpation at the deltoid insertion and diffusely over the entire shoulder. She has pain with motion; it is worse at the end point of range. Her active range of motion is from 0° to 60° (abduction), 0° to 70° (forward flexion), and 0° to 20° (external rotation). Her passive range of motion is only 10° better in all directions. Question What should the patient expect with regards to recovery with this disorder? Answer Choices 1 She will be better in a month if she does her physical therapy 2 She will be better only after you inject her with steroids 3 She will be this way the rest of her life 4 She needs surgery to get better 5 She will need 1 to 2 years to recover full motion

She will need 1 to 2 years to recover full motion Explanation You should advise patients with Adhesive Capsulitis that they should expect, on average, a 1 to 2 year recovery period to regain full motion and for pain to fully subside. Although physical therapy is an adjunct to recovery, it may shorten the recovery; however, it will only shorten it to about a year. Most patients will recover fully over this time period and will not have to put up with it for life. Some orthopedic surgeons will perform manipulation under anesthesia, but only after a full attempt for self-recovery of at least a year. Steroids will help with the pain, but not the motion recovery.

A 12-year-old African-American boy presents with a 12-hour history of fever, chills, and knee pain. Plain radiograph is suspicious for osteomyelitis, and blood cultures are positive for Salmonellae. Question What is the patient's past medical history most likely to reveal? Answer Choices 1 Acute lymphoblastic leukemia 2 Osgood-Schlatter disease 3 Sickle cell anemia 4 Osteogenesis imperfecta 5 Type I diabetes mellitus

Sickle cell anemia Salmonellae has been identified as the most common cause of hematogenous osteomyelitis in the setting of hemoglobinopathies, such as sickle cell anemia. Diabetes mellitus is also associated with hematogenous osteomyelitis, but in these patients, the causative organism is commonly S. aureus, not salmonellae. Acute lymphoblastic leukemia, Osgood-Schlatter disease, and osteogenesis imperfect do not have a specific association with osteomyelitis.

Following a fall, a 6-year-old boy has restricted movement of the right hand. On examination, the right shoulder appears swollen. X-ray shows a fracture in the proximal humerus. However, at the site of fracture, there is a lucent defect extending up to the epiphyseal plate. The bone is not expanded. The patient undergoes a surgical procedure. Grossly, the lesion is actually cystic and contains clear yellow fluid. Microscopically, the cyst wall is made of fibrous tissue with occasional giant cells. What is the most likely diagnosis in this case? Answer Choices 1 Aneurysmal bone cyst 2 Simple cyst of bone 3 Ganglion 4 Synovial cyst 5 Telangiectatic osteogenic sarcoma

Simple cyst of bone Explanation An aneurysmal bone cyst is a non-neoplastic locally aggressive lesion of the bone typically seen in first 2 decades of life. There is a slight female preponderance for developing them. It involves the metaphyses of the long bones. X-ray shows characteristic purely lytic lesion involving the metaphyses of a long bone with an eccentric, blow out appearance and extension into the soft tissue, which is limited by a thin shell of new bone. Grossly, it shows cavities separated by septa. The spaces contain blood or serum. Microscopically, there are cysts of various sizes separated by septa. The septa are composed of loosely arranged spindle cells and benign giant cells. The spaces are lined by cuboidal cells. The septa show capillary proliferation. Areas may show relatively solid proliferation of cells. It is treated by simple curettage but they can recur. Simple cyst of the bone is seen mainly in the proximal humerus and proximal femur in children. The patients generally present with a pathological fracture. X-ray shows a lucent defect situated centrally that extends up to the epiphyseal plate. Bone involved is not expanded. Grossly, the cyst has a thin fibrous wall and it is filled with clear yellow fluid. Microscopically, the cyst wall is thin, fibrous, septa, with occasional giant cells. A peculiar calcified matrix like cementum may be present. Treatment is by aspiration of the cyst followed by injection of methylprednisolone acetate. If this fails, then surgery is advised. Telangiectatic osteogenic sarcoma is a histological variant of osteogenic sarcoma. It is characterized by prominent blood-filled cysts that appear similar to aneurysmal bone cysts radiographically and pathologically. Pathological fractures are very frequent. However, the septum that separates the bloody cysts contains malignant stroma. They are associated with a more aggressive course than conventional osteogenic sarcoma. A ganglion is a small cyst that is always located near a joint capsule or tendon sheath. It is commonly seen around the wrist joint. It arises because of cystic or myxoid degeneration of connective tissue. The cysts lack a true cell lining. The lesion may be multilocular, and it enlarges by coalescence of adjacent areas of myxoid change. Synovial cysts are caused by the herniation of synovium through a joint capsule. Sometimes, it occurs due to massive enlargement of the bursa. The cyst will have a synovial lining, and it may be hyperplastic and contain inflammatory cells and fibrin. The synovial cyst that forms in the popliteal space is called as Baker's cyst. These cysts are associated with joint diseases leading to increased intra-articular pressure, such as degenerative joint disease, neuropathic arthropathy, and rheumatoid arthritis.

A 56-year-old man wakes up in the morning to find that he has a swollen, red, painful big toe on his left foot. 2 days earlier, he had been on a cruise to the Bahamas and spent much of the time eating and drinking. He normally has 1 glass of wine with dinner on the weekends, but while on the cruise, his alcohol consumption increased substantially. He also did a great deal of walking in an attempt to make up for his excesses. He goes in to his physician's office, and tests are run. An X-ray shows no acute fracture, and his vital signs are within normal limits. Blood work shows an increase in uric acid, but it is otherwise normal. He begins treatment and feels better within 24 hours. Question What statement is true? Answer Choices 1 Attacks are polyarticular in nature 2 Smaller, lower-extremity joints are usually involved first 3 Diet has no effect on this disease 4 Whites are affected more frequently than blacks 5 It occurs frequently in men under the age of 25

Smaller, lower-extremity joints are usually involved first Gout presents as acute monoarticular arthritis in 90% of patients. In early gout, usually only 1 or 2 joints are involved. Usually, they are the smaller, lower-extremity joints. Podagra, or inflammation of the first metatarsophalangeal joint, is the initial joint manifestation involved in about half of all cases. However, podagra may also be observed in patients with pseudogout, reactive arthritis, gonococcal arthritis, psoriatic arthritis, and sarcoidosis. The attacks usually begin abruptly and can reach maximum intensity in 6-12 hours. The joints are red, hot, and extremely tender. Untreated, the characteristics of gout change over time. The attacks become more polyarticular. Although more joints may become involved, inflammation in a given joint may become less intense. Attacks occur more frequently and last longer. Eventually, patients may develop a chronic polyarticular arthritis, which can be symmetrical and resemble rheumatoid arthritis. Tophi, which are collections of uric acid crystals in the soft tissues, occur frequently in untreated patients. They can be found in multiple locations, including the fingers, toes, in the olecranon bursae, and along the olecranon (where they may appear to be rheumatoid nodules). Tophi tend to develop after 10 years in untreated patients who develop chronic gouty arthritis. African Americans are usually affected more than Caucasians. Symptoms occur more commonly in men than in women. They usually present in the age group of 30 - 50 years. In women, it occurs more commonly in the postmenopausal age group

You are evaluating a 32-year-old man who presents with an 11-day history of progressive, atraumatic, left hip pain. His PMH is unremarkable with the exception of intravenous drug use. He complains of increasing pain that originally was aggravated with movement, but now he has pain at rest. He is ambulatory with a limp and exam shows no edema, crepitus, rash, or erythema. Pain is reproduced with hip ROM. Distal CMS is fully intact. Vital signs show a BP of 124/93, HR 134, RR 20, Temp 39.4, and O2 sat of 96% RA. Laboratory studies show a leukocytosis and elevated ESR. Question What finding would you expect on his hip X-ray? Answer Choices 1 A transverse, non-displaced fracture of greater trochanter 2 Soft tissue swelling with periosteal elevation 3 Severe bone demineralization 4 Tophi 5 An osteochondroma

Soft tissue swelling with periosteal elevation The answer is soft tissue swelling with periosteal elevation.1 This patient presents with osteomyelitis of the hip. Osteomyelitis is infection of the bone, with accompanying inflammation and eventual bone destruction.1 Risk factors for osteomyelitis include IV drug use, implanted devices and hard wear, immunocompromised states, open fractures, and chronic joint destruction, such as in patients with rheumatoid arthritis.2 Acutely, patients may have fever, malaise, a mild leukocytosis, and elevated inflammatory markers such as ESR and CRP.3 Additional work up should include blood cultures, bone aspiration, and imaging studies. The initial imaging modality should be an X-ray. Early in the process, the X-ray may be completely normal or show soft tissue swelling with periosteal elevation. Late findings will include bone demineralization, lytic regions, and eventual bone sclerosis.3 A fracture of the greater trochanter in a 32-year-old without trauma is extremely unlikely. Severe bone demineralization is a late finding of osteomyelitis. Roughly 30%-50% of bone demineralization is necessary for this to be seen on x-ray.3 In a patient with an 11-day history of symptoms, it is unlikely you will find this on x-ray at this time. Tophi are deposits of urate crystals seen in patients with longstanding gout.2 An osteochondroma is a benign cartilage growth.4

A newborn boy was observed to have a small cyst located midline in the lumbar region of his back. No neurological symptoms were observed, and the sac contained only meninges and cerebral spinal fluid. X-ray revealed the absence of a vertebral arch at L5. What is this type of anomaly? Answer Choices 1 Spina bifida with meningocele 2 Spina bifida occulta 3 Spina bifida with meningomyelocele 4 Spina bifida with myeloschisis 5 Spinal dermal sinus

Spina bifida with meningocele Explanation The failure of the 2 halves of the vertebral arches to fuse during development results in an anomaly referred to as spina bifida. The most minor, clinically insignificant type (spina bifida occulta) is asymptomatic and may be revealed only with imaging of the vertebra involved. Sometimes a small dimple or tuft of hair overlying the defect may be present at the level of the malformation, usually in the lumbar or sacral regions. More serious anomalies, categorized as spina bifida cystica, show a cyst-like protrusion at the level of the defect. When the cyst or sac contains meninges and cerebrospinal fluid, it is referred to as spina bifida with meningocele. If the spinal cord and/or nerve roots are included with the meninges and CSF in the sac, it is referred to as spina bifida with meningomyelocele. The most severe type of vertebral arch defect is one in which not only did the arches fail to fuse in the midline, but the neural plate failed to fold and fuse at the level of the anomaly, resulting in a flattened plate of nervous tissue, called spina bifida with myeloschisis. Spina bifida cystica show varying degrees of neurological deficits.

An 8-year-old boy who was born with myelomeningocele and had subsequent repair and shunting has had worsening symptoms of bladder incontinence and consequent increased frequency of urinary tract infections. He has been using a wheel chair much more frequently lately because his back, legs and right hip have been hurting and he has felt a popping sensation in his hip. Based on these symptoms, what is the most likely diagnosis? Answer Choices 1 Natural progression of myelomeningocele 2 Shunt malfunction 3 Spinal cord tethering 4 Syringomyelia 5 Chiari II malformation

Spinal cord tethering Explanation Spinal cord tethering may cause progressive neurologic deficits, including decreased muscle strength or increased tone, deteriorating gait, change in sensory level, urologic dysfunction or orthopedic deformities such as hip dislocation, pes cavus or scoliosis. Although anatomical evidence of some degree of tethering is evident in almost all myelomeningocele patients, surgical untethering is reserved only for those having progressive signs and symptoms. Diagnosis is thus based on clinical grounds and not imaging studies. Natural progression of myelomeningocele is partly dependent on the level of the lesion. In children without severe developmental delays and hypotonia, 89% of preadolescents or 100% with low lumbar or sacral and 63% with higher lesions are ambulators when they receive multidisciplinary orthopedic and neurosurgical management. After adolescence ambulation decreases to 50% as it becomes more energy efficient to use a wheel chair. Orthopedic procedures are common. 85% of school age children can be free of urinary incontinence in social situations. Long term mortality can be 35-50% by adulthood. Shunt malfunction signs and symptoms include cognitive changes such as change in school performance or worsening behavior, onset or change in frequency of seizures, decrease in muscle strength or loss of previously acquired motor skill or increase in spasticity, change in ambulation, change in urinary or bowel function, change in lower cranial nerve function, pain in the back especially around the myelomeningocele repair site or worsening in scoliosis or lower extremity orthopedic deformities. Syringomyelia is a dilated spinal cord central canal filled with CSF. This is present in up to 80% of patients with myelomeningocele but only 2.5% show symptoms such as upper extremity weakness, spasticity or ascending motor loss in the lower extremities. Extension into the brain stem (syringobulbia) can cause lower cranial nerve and brainstem dysfunction. Chiari II malformation consists of the caudal displacement of the cerebellar tonsils and vermis, caudal medulla and sometimes the fourth ventricles into the cervical spinal canal. Although present in most children with myelomeningocele, it causes severe enough symptoms in 15-35% of patients to require surgical treatment. Symptoms are most common during infancy with disorders of swallowing and may include choking on foods and liquids, nasal regurgitation during drinking and frequent vomiting or significant gastroesophageal reflux. In addition, repeated aspiration pneumonia, dysarthria, apnea or cyanotic spells, inspiratory stridor and a hoarse or high-pitched cry. Older children and adults with symptoms have weakness or spasticity of the upper extremities, headache or neck pain, cerebellar dysfunction, oculomotor changes, and scoliosis.

A 4-year-old girl presents after a fall from a 4-foot slide 30 minutes ago. She fell with her right dominant arm outstretched and her elbow fully extended. There was no loss of consciousness, but there was extreme pain. She immediately grabbed her right forearm and her right elbow. There is an obvious deformity at the elbow. What is the most likely diagnosis? Answer Choices 1 Proximal humerus fracture 2 Distal radius fracture 3 Supracondylar humerus fracture 4 Lateral epicondyle humerus fracture 5 Medial condyle humerus fracture

Supracondylar humerus fracture Supracondylar fractures of the distal humerus are the most common elbow fracture in children between the ages of 2 and 12. The typical mechanism of injury for this type of fracture is a fall on an outstretched arm, usually from a height. Proximal humerus and distal radius fractures will not show deformity at the elbow unless also associated with injury at the elbow. Fracture of the lateral epicondyle of the humerus accounts for 1/5th of elbow fractures in children. Fracture of the medial condyle of the humerus is also uncommon.

A 76-year-old male presents to the hospital with a hot, swollen, and painful right knee that appears to have an effusion. After your examination, you decide to perform an arthrocentesis. The fluid aspirate is turbid, cloudy, and most definitely looks like it is infectious material. Based on this clinical scenario, what would be most likely bacterial organism present in an elderly septic arthritis? A Hemophilus influenzae B Neisseria gonorrhoeae C Pseudomonas aeruginosa D Staphylococcus aureus E Streptococcus pneumonia

Staph aureus D Septic arthritis is an infection within a joint space. The infection can be caused by multiple pathogens including bacteria, viruses, fungi and mycobacteria, but in older adults the most common organism implicated is Staphylococcus aureus. These bacteria can get into the joint space through multiple mechanisms including direct inoculation after a penetrating injury, hematogenous spread from an infection elsewhere in the body, or from extension from a contiguous bone infection. All of the bacteria mentioned as possible answers are possible causes of septic arthritis, but there prevalence varies within different patient populations. Hemophilus influenza and Streptococcus pneumonia are commonly seen in children less than five years of age that develop septic arthritis. Neisseria gonorrhoeae is most common in sexually active adolescents and young adults. Pseudomonas aeruginosa is a common cause of septic arthritis in IV drug abusers

A 52-year-old man presents to the emergency department with fever, chills, and left foot pain. His foot has been hurting for the past few weeks, but has worsened over the last few days. There is no known injury to the area, but the patient notes that the pain began after he started wearing a new pair of shoes. Past medical history includes hypertension and type 2 diabetes, but he has not followed up with his physician recently. Upon examination, there is an ulcer on his heel with surrounding erythema and edema. X-ray reveals erosion and periosteal changes at the site of the ulcer. Question Which organism is the most common cause of this infection? Answer Choices 1 Staphylococcus aureus 2 Escherichia coli 3 Pseudomonas aeruginosa 4 Serratia marcescens 5 Salmonella enterica

Staphylococcus aureus is correct. The patient has osteomyelitis. Staphylococcus auerus is the most common cause of osteomyelitis of the foot in diabetic patients. Escherichia coli, Pseudomonas aeruginosa, Serratia marcescens, and Salmonella enterica are incorrect. While all these organisms may cause osteomyelitis, Staphylococcus aureus is the most common cause

A patient complains of pain during medial rotation of the humerus but not during abduction or extension of the shoulder. What muscle is injured? Answer Choices 1 Pectoralis major 2 Teres minor 3 Latissimus dorsi 4 Deltoid 5 Subscapularis

Subscapularis The pectoralis major flexes, adducts, medially rotates, and is a minor extensor of the shoulder. The teres minor is a lateral rotator. The latissimus dorsi extends and medially rotates. The deltoid abducts and can assist in flexion, extension, and medial and lateral rotation. Only the subscapularis medially rotates and is not involved in extension and abduction. Therefore, this is the injured muscle.

A 22-year-old woman presents for evaluation of knee pain. The patient appears in moderate distress and is unable to fully bear weight as she moves to the exam table. Her physical exam reveals a swollen, tender left knee. She is tender at the lateral femoral condyle, lateral tibial plateau, and tibiofemoral joint line. Lachman and anterior drawer tests are positive. Posterior drawer, Apley grind, patellar grind, and McMurray tests are negative. There are no sag or apprehension signs. Valgus and varus stress tests are normal. An X-ray is obtained and shows no fractures. Question What inciting event is most likely in this patient's history? Answer Choices 1 Dashboard injury to knee during a motor vehicle accident 2 Fever and prior Neisseria gonorrhoeae infection 3 Frequent squatting and climbing stairs 4 Sudden landing after jumping during basketball 5 Lateral blow to knee

Sudden landing after jumping during basketball Explanation This patient's physical exam findings demonstrate a deficiency in the anterior knee, specifically the anterior cruciate ligament (ACL). ACL injuries most often occur acutely while playing sports, such as basketball, skiing, and tennis; however, they can occur with other mechanisms as well. Female gender is a risk factor. The patient may give a history of hearing a "pop", along with acute pain as well as buckling or instability of the knee. Of the choices listed, sudden landing after jumping during basketball is the most likely to produce an ACL injury. A dashboard injury to the knee during a motor vehicle accident could produce several injuries, but it would be most likely to produce a posterior dislocation and disrupt the posterior cruciate ligament (PCL). A PCL injury would likely yield physical exam findings of a positive posterior drawer and sag sign (in which the anterior tibia sags posteriorly). Fever and prior Neisseria gonorrhoeae infection with knee pain suggests septic arthritis as the cause. Septic arthritis would produce swelling and tenderness, but it would not be expected to demonstrate instability, such as this patient's positive anterior drawer and Lachman tests. Frequent squatting and climbing stairs can contribute to knee pain, and would be suggestive of a patellofemoral syndrome or possibly osteoarthritis (although the latter is unlikely in a 22-year-old patient). If this patient's knee pain had been caused by squatting and stair-climbing, her physical exam might have demonstrated a positive apprehension test and patellar grind test. A lateral blow to the knee is associated with disruption of the medial and/or lateral collateral ligaments. Here, we would expect the physical exam to have a positive valgus and/or varus stress test. A lateral knee injury would not typically cause ACL injury.

A 17-year-old male high school football player presents after being tackled and slammed onto his right dominant shoulder forcefully 2 hours ago during a game. He had immediate pain, but was able to continue punting. He has full active and passive range of motion, but some pain (4/10) with abduction. There is no obvious deformity and the skin over the shoulder is intact and not tented. What would you expect to find on a physical exam? Answer Choices 1 Tenderness over the greater tuberosity 2 Tenderness over the AC (acromioclavicular) joint 3 Crepitus with motion 4 Decreased radial pulse 5 Paresthesia over the elbow

Tenderness over the AC (acromioclavicular) joint Explanation Acromioclavicular (AC) separations typically occur as a result of a direct blow to the very lateral shoulder, usually a fall onto the shoulder. This AC separation is probably a Type I. Type I will have tenderness over the AC joint, no deformity and fair motion with some pain but the patient may still be able to play football with the shoulders padded and limited contact. Pain occurs with abduction mainly. They will typically hold their arm adducted and supported. Tenderness over the greater tuberosity would be seen with rotator cuff pathology or proximal humerus fractures. Crepitus would be seen with fracture or Type III AC separations. Nerve and/or vascular injuries are very rare with this injury.

A 21-year-old man presents with a 3-month history of pain in his left shoulder. He is a left-handed pitcher for his college baseball team and states that the pain began at spring training and has gradually worsened since that time. In addition to the pain, the patient is now having difficulty lifting his left arm above his head. He denies any injury to the shoulder itself and states that ibuprofen provides some relief. Question What physical exam finding in this patient would be most indicative of rotator-cuff tendinitis? Answer Choices 1 A positive "crossover" test 2 Tenderness to palpation along the anterior aspect of the left humerus 3 Tenderness to palpation just under the acromion 4 A positive "Speed's test" 5 Palpable crepitus during range-of-motion maneuvers

Tenderness to palpation along the anterior aspect of the left humerus The muscles of the rotator cuff include the supraspinatus, infraspinatus, teres minor, and subscapularis, otherwise known as the SITS muscles. Three of these muscles - the supraspinatus, infraspinatus, and teres minor - insert on the greater tuberosity of the humerus. Repetitive movements of the shoulder, as with throwing or swimming, can lead to inflammation of the tendons of the rotator cuff muscles; the most commonly affected muscle is the supraspinatus, which inserts directly under the acromion. Classically, a patient with rotator cuff tendinitis will have tenderness to palpation just under the acromion on the affected side as well inability to lift the arm above shoulder level (abduction and external rotation). The "crossover test" is used to assess the acromioclavicular (AC) joint of the shoulder. Inflammation or arthritis of the AC joint usually presents with localized tenderness or pain with adduction of the affected shoulder. A "positive crossover test" is when adduction of the patient's affected arm across the chest elicits pain. Bicipital tendinitis, inflammation of the long head of the biceps tendon, can also present with anterior shoulder pain that may resemble rotator cuff tendinitis and may coexist with it. However, in bicipital tendinitis, tenderness is usually maximal in the bicipital groove along the anterior aspect of the humerus. The Speed's test is used to evaluate for biceps tendon injury. The patient keeps the elbow extended and arm supinated. In this position, the patient elevates the arm against resistance. Pain with elevation is indicative of a inflammation or tear in the biceps tendon. Palpable crepitus during range of motion maneuvers is suggestive of osteoarthritis of the joint, not tendinitis, though both can co-exist.

A Caucasian man presents with edema and pain in his right shoulder. He tells you he plays softball every weekend and does a lot of hoeing and shoveling around his house. The shoulder had been sore, but now it hurts to the point where he tries to avoid using it. The drop-arm test is negative, but the patient is unable to lift his arm up to 90 degrees without pain. What is the most likely diagnosis? Answer Choices 1 Bursitis 2 Fibromyalgia 3 Gout 4 Tears in the rotator cuff 5 Tendinitis

Tendinitis Explanation Tendinitis is an inflammation of the tendon tissue, or the tendon sheath (tenosynovitis). The cause is often unknown. As the vascularity of tendons decreases with age, the incident of Tendinitis will increase. Symptoms usually include painful tendons on movement. Swelling can be seen, or just felt, on palpation. Along the tendon, localized tenderness of variable severity is present on palpation. Generally, there is not the warmth or redness of Bursitis. During the drop arm test the client will be asked to fully abduct the arm (straight out laterally from the side), then slowly lower the arm to his side. If he is unable to hold his arm against gravity, the drop arm test is positive. The drop arm test detects whether or not there are any tears in the Rotator Cuff. If there are tears in the rotator cuff, or supraspinatus tendon, the arm will drop to the side from a position of 90 degrees. The client will not be able to lower it smoothly in rotator cuff tear, and will not be able to raise it to 90 degrees in tendonitis. Bursitis is an inflammation of the bursa. A bursa is a closed sac containing a small amount of synovial fluid (usually < 1 cm). A true bursitis has fluid that can be aspirated. Tendinitis is often mislabeled as bursitis. Bursitis around the knee is more common in those who kneel often (like maids and clergymen). To distinguish between Tendinitis and Bursitis, note that Bursitis will often not only feel painful on use, but have swelling, redness, and warmth. Bursae are sac-like cavities, or potential cavities, that contain fluid. Analysis of bursal fluid can tell the care giver the reason for the inflammation. Fibromyalgia is a soft tissue, non-articular pain disorder characterized by chronic, generalized musculoskeletal aches, pains, and stiffness that occur primarily in muscles and their attachments. It is associated with specific sites of exaggerated tenderness. Gout occurs most commonly in males (95%). Presentation is usually acute, with a swollen, red big toe. Females may develop gouty arthritis in the perimenopausal phase, as estrogen levels drop. In many cases, if the disorder is left untreated, urate crystals are deposited, usually in and around the joints of the extremities. The needle-shaped monosodium urate (MSU) crystals can be deposited in the parenchyma of organs such as the kidney. In some clients, deposits of uric acid form within the urinary collecting tubules.

You are evaluating a 28-year-old man who is brought in by ambulance for a right knee injury sustained during an MVA. The patient is in obvious distress, holding his right knee and screaming. You are told by EMS that the patient was the restrained driver of a car that was rear-ended at high speed. You are unable to obtain any additional history. Examination already shows the development of a right knee effusion. The skin is intact, and there does not appear to be any deformity. Examination of laxity is limited, secondary to the patient's pain. It appears that distal sensation is intact but you cannot palpate a dorsals pedis or posterior tibial pulse. You order an emergent MRI for further evaluation. Question In the meantime, what test would be best to help confirm the patient's diagnosis? Answer Choices 1 Apley grind test 2 Bilateral ankle-brachial indices 3 Lachman's test 4 Valgus stress 5 McMurray test

The correct answer is bilateral ankle-brachial indices. This patients history and exam are concerning for a knee dislocation. Anterior knee dislocation is the most common type, and often due to a high velocity injury such as an MVA.1 While rare, dislocations of the knee have significant complications involving both structural damage to the knee (disruption of tendons, ligaments, and cartilage), as well as neurovascular injuries (injury to the popliteal and/or tibial arteries, and damage to the peroneal nerve most commonly).1 The finding of absent, or weak, distal pulses is concerning for an injury to the popliteal artery. While an MRI will be the definitive test, the finding of an ankle-brachial index of <0.8 (1.0-1.4 being normal) is very concerning and warrants immediate involvement of an orthopedic or vascular surgeon.2 The Apley grind test is used to evaluate a meniscus injury. It is done by having the patient lay prone with their knee bent at 90 degrees. The examiner then applies downward pressure by pushing down on the bottom of the patient's foot while rotating the foot. The test is positive if pain is reproduced.3 The Lachman test is performed by having the patient supine and the knee flexed at 30 degrees. One hand firmly holds the distal femur, and the second hand pulls the proximal tibia anteriorly. Excessive anterior movement of the tibia indicates an injury to the anterior cruciate ligament.3 Valgus stress is done with the patient in the supine position. One hand supports the lateral aspect of the knee while the other hand grasps the patient's ankle and applies valgus stress (move the ankle joint to the lateral side). This test is used to assess if there is laxity of the medial collateral ligament.3 The McMurray test is used to evaluate for a meniscus injury. It is performed on a supine patient by bringing the knee up to 90 degrees of flexion. The examiners hand is then placed of the medial aspect of the knee, and the other hand externally rotates the foot and then extends the leg. If a "click" is appreciated, the test is positive for a medial meniscus injury. The opposite is done to assess for a lateral meniscus injury.3

A 33-year-old woman presents with tingling and numbness in the palms, thumbs, and index fingers bilaterally. Her symptoms are worse during the night. Recently, she noticed that she has difficulty grasping small objects. She works in a poultry farm and is involved in assembly packing. On examination, Tinel's sign and Phalen's test are positive. Question What nerve is compressed? Answer Choices 1 Ulnar nerve 2 Median nerve 3 Radial nerve 4 Musculocutaneous nerve 5 Posterior interosseous nerve

The correct diagnosis is carpal tunnel syndrome (CTS), in which the median nerve is compressed. CTS is a compressive neuropathy of the median nerve in the carpal tunnel at the wrist. The carpal tunnel is bound by carpal bones on the 3 sides and is located at the base of the palm. The median nerve runs within it. Several conditions that cause compression of the nerve, such as acromegaly, hypothyroidism, diabetes, gout, pseudogout, repetitive work such as assembly packing, and prolonged use of vibratory tools, can cause CTS. Treatment consists of anti-inflammatory drugs, exercises, splinting, and carpal tunnel release surgery. Ulnar nerve compression causes numbness and tingling in the one and a half fingers on the ulnar aspect of the hand. Radial nerve compression causes pain in the upper extensor part of the forearm and weakness of extension of the wrist, fingers, and thumb. In posterior interosseous nerve compression, there is usually no sensory nerve deficit; motor paralysis of the extensors can occur. Musculocutaneous nerve compression is rare and causes symptoms in the forearm, not the hand.

A 65-year-old woman presents with morning stiffness and pain in her shoulders. She reports that she has lost a few pounds over the past month and has had intermittent fevers. The physical examination reveals tenderness over her deltoids with reduced shoulder abduction. What laboratory findings would be expected? Answer Choices 1 Elevated C-reactive protein 2 Abnormal electromyogram 3 Elevated serum creatine kinase level 4 Abnormal muscle histology on biopsy 5 Microcytic microchromic anemia

The correct response is an elevated C-reactive protein. The clinical presentation is suggestive of polymyalgia rheumatica; patients usually present with pain and stiffness of the pectoral and pelvic girdles. They may also have constitutional symptoms, such as weight loss and fever. On examination, the affected shoulder or thigh muscles are tender, and there is a painful restriction of hip and shoulder movements. Laboratory results in these patients usually reveal an elevated erythrocyte sedimentation rate (ESR) and elevated C-reactive protein. The serum creatine kinase is within normal limits, and a normochromic normocytic anemia may be present. Electromyography is normal and a muscle biopsy, if done, is also normal. Management is with corticosteroids.

A 34-year-old man was the driver in a single car motor vehicle accident. Preliminary radiologic studies show a comminuted fracture of the right tibia. Currently, the patient is describing a substantially increasing amount of pain felt in the injured extremity. He describes the pain as being a 10/10; it seems as if it is becoming worse with each passing minute. With anguish on his face, he describes it as a deep, achy, burning pain. You quickly examine the right leg; you note pallor, a tense 'wood-like' feeling of the extremity, diminished sensation, and muscle weakness. Question Given the most likely diagnosis, what would be the most appropriate clinical intervention at this time? Answer Choices 1 Observation only 2 Intravenous heparin therapy 3 Fasciotomy 4 Placement of a long leg cast 5 Intravenous antibiotic therapy

The correct response is fasciotomy. This patient is suffering from acute compartment syndrome. Compartment syndrome should always be considered an urgent and extremely emergent situation due to the consequences that can ensue. Acute compartment syndrome (ACS) most often develops after the patient has suffered some type of significant trauma and especially involving a long bone fracture. Fractures account for more than 75% of cases of ACS. More often, ACS is seen in patients under the age of 35 years of age; it also has a higher incidence in young men, especially after they experience fracture of the tibial diaphysis or distal radius; risks of developing ACS is even more increased if the patient suffers from a comminuted fracture. Our patient described above fits the scenarios that show a higher risk of developing ACS. Many times the signs and symptoms of ACS occur in a stepwise fashion; an important clue is a rapid progression of symptoms and signs in a short amount of time. Symptoms include pain out of proportion to the apparent injury, a persistent deep ache or burning pain, or even paresthesias. Signs that would be evident include pain with passive stretch of muscles in the affected compartment, a tense compartment with a firm 'wood-like' feeling, pallor, diminished sensation, muscle weakness and a late finding of paralysis. Adequate decompressive fasciotomy is the procedure that must be performed. Just prior to surgery the affected leg in the patient above should be immobilized with the ankle in slight plantar flexion; doing so helps decrease the deep compartment pressure and does not cause increased pressure in the anterior compartment. Urgent treatment of compartment syndrome can cause is a necessity to avoid the detrimental consequences that will occur as a result of this pathology. Such complications include permanent loss of function. Observation only is the inappropriate choice for this reason. All bandages and casts must be removed to prevent worsening of the ACS, therefore placement of a cast is incorrect; it has been estimated that complete removal of a cast will lead to a decrease of 85% in pressure from the patient's baseline. Both intravenous heparin and/or antibiotic therapy are inappropriate options at this time.

A 49-year-old Caucasian man is well known to your practice; he presents with pain in his left lower extremity. Upon further questioning, the patient describes doing construction work 3 months prior when he jumped onto his feet from a height of about 5 ft. Since this episode he has noted issues of increasing left sided hip and knee pain. He describes the pain as radiating into the left groin and front middle thigh area. The pain is relieved with sitting, and it is aggravated by walking and climbing up stairs. The patient denies any paresthesias, numbness, bowel/bladder dysfunction, fever, night sweats, or chills. Radiographic interpretation includes the presence of a crescent sign as well as marked irregularity of the left femoral head with sclerosis. Question Given these findings, what would you expect to see on a radiographic image taken of the left hip? Answer Choices 1 Subchondral collapse 2 Lytic lesions 3 Femoral head fracture 4 Bone cyst 5 Bone tumor

The correct response is subchondral collapse. Avascular necrosis, sometimes also referred to as osteonecrosis, is osseous cell death resulting from vascular compromise. Common sites that are affected include the proximal or distal femoral head or even the ankle, shoulder, or elbow. Common causes include but are not limited to corticosteroid use, alcoholism, trauma, systemic lupus erythematosus, pancreatitis, gout, or even sickle cell disease. Radiographic findings will include mild density changes early on, sclerosis and a more progressed disease state will lead to the pathognomonic crescent sign. When repair begins at the interface between necrotic and viable bone this produces a sclerotic margin. Eventual mechanical failure of trabecular bone at this interface results in progressive micro fracture and collapse of the adjacent dead subchondral trabeculae and this leads to the subchondral radiolucent area along the fracture line, which is the crescent sign. Later disease will show joint-space narrowing and degenerative changes in the acetabulum. Lytic lesions are most commonly seen in patients with multiple myeloma. A fractured femoral head may be seen as the result of a high-impact trauma involving significant force, falls in older patients, or various illnesses or disease states that significantly affect bone integrity, such as vitamin D deficiency, systemic lupus erythematosus, or cancer. Both of these potential answers are not as likely. A bone cyst is described as a fluid-filled hole that develops inside a bone. These most commonly occur in children and young adults and very rarely cause significant symptoms. The cause of bone cysts is unclear; usually they will not cause serious health issues. This does not match what is described in the original question. A bone tumor is an area of abnormal growth of cells within the bone and may either be malignant or benign. Causes of bone tumors are generally unknown but are seen commonly in patients with a history of genetic abnormalities, radiation, or injury. More often than not, no specific cause is found; again, this is most likely not the correct answer.

What is seen during the early stages of osteoarthritis? Answer Choices 1 Loss of articular cartilage 2 Thickening of articular cartilage 3 Decrease in proteoglycan synthesis 4 Decrease in water content of the joint 5 Lack of hypertrophic repair of articular cartilage

Thickening of articular cartilage Explanation Most descriptions of osteoarthritis emphasize the loss of articular cartilage that occurs as a result of this disease. However, during early stages of this disease, the cartilage is thicker than normal. There is an increase in water content of the joint as well as an overall increase in proteoglycan synthesis and hypertrophic repair of articular cartilage.

A 67-year-old man presents with the complaint of a "tender bulge in his right dominant arm" following an attempt at lifting his couch 2 days ago. He admits to having mild chronic shoulder pain and taking ibuprofen (Motrin) for relief. While attempting to lift the couch, he felt a "snap" in his shoulder and noticed a "bulging" in his anterior arm. What anatomic structure is most likely injured? Answer Choices 1 Distal Biceps Brachii tendon rupture 2 Long head of the Biceps Brachii tendon rupture 3 Short head of the Biceps Brachii tendon rupture 4 Supraspinatus tendon rupture 5 Triceps Brachii tendon rupture

The long head of the biceps brachia is the correct answer, it passes through the bicipital groove of the humeral head and attaches on the superior rim of the glenoid. It functions to suppress the humeral head on the glenoid surface, but it is exposed to friction rubbing over time; therefore, it is subject to rupture, even with what seems like minimal trauma. The distal biceps rupture may occur, but it is less common and more painful, and it requires much more force to rupture. The short head of the biceps brachii attaches to the coracoid process of the scapula and is less likely to rupture since there is minimal irritation to this tendon from use. The supraspinatus is a rotator cuff muscle and would not cause a "bulge" when it ruptures. The triceps brachii muscle is on the posterior arm and would not cause "bulge" in the anterior arm if it ruptured.

A 16-year-old gymnast presents with severe left knee pain of acute onset following an injury she sustained while performing training exercises 1 hour ago. She has no significant past medical history. During a planted pivoting motion of her leg, she felt her "knee pop out of place." Her physical exam reveals left knee hemarthrosis, localized tenderness over the medial patellar retinaculum, and an inability to flex her left knee. There is an abnormal, asymmetric displacement deformity of the left knee and pain and apprehension when the patella is pushed laterally with the knee slightly in flexion. However, she is able to fully extend her lower extremity at the left knee. Question What is true regarding this patient's diagnosis? Answer Choices 1 The most likely direction of patellar dislocation is laterally. 2 McMurray's and Apley's compression tests are expected to be positive. 3 Asymmetric knee cartilage loss and narrow joint space are present. 4 This patient experienced a tear of quadriceps tendon. 5 Chronic, repetitive kneeling on the left knee is the underlying cause.

The most likely direction of patellar dislocation is laterally. This patient presents with the manifestations consistent with a patellar dislocation. It is most common in females in the second decade of life. The typical mechanism involved is one of pivoting the knee on a fixed lower leg. Another mechanism includes a valgus force, and external rotation of the tibia is applied to a flexed leg. Further historical findings include a perception of the "knee popping out of place,"; the displaced patella is typicall dislocated laterally. Examination demonstrates tenderness over the medial retinaculum and adductor tubercle, which is the origin of the medial patellofemoral ligament. The patient also may have a hemarthrosis, pain, and apprehension when the patella is pushed laterally with the knee slightly bent. Narrowing of the joint space and asymmetric cartilage loss due to chronic trauma are common in osteoarthritis. Quadriceps tendon tears occur most commonly in patients over 40 years old. Risk factors include anabolic steroid use, local steroid injection, diabetes mellitus, inflammatory arthropathy, and chronic renal failure. Typically, patients with tendon tears present with a history of a sudden "pop" while stressing the extensor mechanism. Patients have pain at the site of injury, difficulty with weight bearing, knee joint effusion, tenderness at the upper pole of the patella, and a palpable defect proximal to the superior pole of the patella. Complete tears result in loss of active knee extension; partial tears can still have knee extension. Chronic, repetitive kneeling is a common cause of prepatellar bursitis.

A 27-year-old man presents with knee instability. During a game of basketball, the man jumped up for the ball and landed incorrectly. He felt and heard a "pop". He experienced instant swelling and pain; when he attempted to apply weight, it felt like his knee was going to "give out". The patient states, "If I don't wear a knee brace, especially when I'm walking down stairs, it feels my knee is going to give out." On exam, he has a positive Lachman sign and anterior drawer sign. McMurray's sign, posterior drawer sign, and varus and valgus stress test are all negative. Question What type of injury does this patient have? Answer Choices 1 Posterior cruciate ligament 2 Anterior cruciate ligament 3 Medial cruciate ligament 4 Lateral cruciate ligament 5 Medial meniscal

The patient has an anterior cruciate ligament (ACL) injury. Applying the Lachman test and anterior drawer sign will increase the translation between the femur and tibia; an intact ACL would have decreased translation and an endpoint. It is very important to exam the unaffected knee first in order to compare it to the affected joint. Patients typically say they felt something "pop", and they have difficulty with ambulation. An MRI can confirm the diagnosis and the extent of the tear. A prompt referral to the orthopedist is warranted. A posterior cruciate ligament injury is suspected if there is a positive sag sign and posterior drawer sign. Usually, the mechanism of injury is when the knee is flexed and there is associated force (e.g. hitting a dashboard during a car accident). A medial cruciate ligament injury is suspected when a valgus stress produces a knee laxity. It presents with tenderness to palpation of the medial aspect of the knee. If there is gross laxity, then an ACL injury should also be suspected because the ACL also provides some medial stability. A mechanism of injury would be a football player being tackled from the lateral side of the knee. A lateral cruciate ligament injury is suspected with a positive varus stress. These are more common with high velocity injuries because the opposing leg provides some protection. A meniscal injury usually presents with a positive McMurray's sign and Thessaly test; the patient typically complains of joint line tenderness or a knee locking sensation. Meniscal injuries usually occur together with ACL tears because the mechanisms of injury are the relatively the same.

A 33-year-old man presents with a red finger on his right hand. He has a history of severe diabetes Type I. Further details from the patient include that he got a severe gash over the knuckle of the right pointer finger while working on an engine motor 2 days ago. He washed it out and covered it as soon as he could. Since this occurred, he has noted a significant increase in pain, redness, and severe restriction of movement in this finger; these characteristics have worsened vastly in the last 12 hours. He admits to "feeling warm" for the past day as well as having significant sweating at night, but he has not taken his temperature. He has tried to take some acetaminophen 500 mg 2 tablets every 8 hours; there has not been any noticeable relief. Pertinent physical examination findings include an oral temperature of 101.5° F, significant erythema extending distally to the DIP joint and proximally toward the dorsal surface of the right hand, severe restriction of flexion, as well as an inability to point the joint when asked. Question Based on the history and physical examination, what is the most likely diagnosis? Answer Choices 1 Osteoarthritis 2 Gouty arthritis 3 Osteopenia 4 Septic arthritis 5 Psoriatic arthritis

The patient in the above scenario is highly likely to be suffering from a case of non-gonococcal acute bacterial arthritis, which is otherwise known as septic arthritis (SA). SA cases present with acute pain, swelling and heat of the affected joint that can be worsening with each passing hour. Chills and fever are common in around 80% of patients who present with SA. Restricted motion and loss of function are also found. In terms of the physical examination, fever is present in 1/3 - 1/2 of patients with SA; erythema, restricted movement and warmth may also be present. The patient's past medical history of having type I diabetes also increases the risk and likelihood of SA. Degenerative joint disease, also known as osteoarthritis (OA), is an incorrect response. Components seen in patients with OA include progressive onset of pain that is exacerbated by activity and relieved at rest. OA signs and symptoms are usually not found in an acute time period, but rather over months to years of progressing pain of the affected joint. Morning stiffness is common in patients with OA, although it will resolve within a 30 minute time period. Physical examination findings include limited range of motion of that affected joint and crepitus, to which the patient above has both. Patients may also have tenderness to palpation of the joint, joint effusion, or even palpable osteophytes. Psoriatic arthritis is an incorrect response. Psoriatic arthritis has distinct signs and symptoms including, but not limited to, psoriasis preceding the arthritis, 'sausage' appearance of finger and toes, symmetric involvement, as well as nail pitting. Acute gouty arthritis has an extremely sudden onset and very commonly occurs at night. Precipitating factors include excessive alcohol intake, changes in medications, or fasting episodes for surgical procedures. A specific injury does not initiate the signs and symptoms of gouty arthritis. The most commonly affected joint is the metatarsophalangeal joint of the great toe, which is sometimes referred to as 'podagra'; attacks can occur in the feet, ankles, or knees as well. The affected joints are progressively swollen and exquisitely tender; also, the overlying skin can be tense, warm, and dusky red. The patient will commonly also have a fever of 39°C or higher. Although some factors are similar to the patient in this case, he has many more similarities to the diagnosis of SA. Osteopenia is much more common diagnosis in older women that have experienced menopause; occurrence is higher in Caucasian women that possess other risk factors, such as tobacco or alcohol abuse, excess/deficiency in hormones, malignancy, and even some genetic disorders. Both disease states are typically asymptomatic until an actual fracture occurs. Loss of height may also be seen.

A 37-year-old woman presents with knee pain after scrubbing her kitchen floor on her hands and knees. Examination shows that she has prepatellar bursitis, which is also known as housemaid's knee. Where is the prepatellar bursa located? Answer Choices 1 Posterior to the patella, between the patella and the infrapatellar fat pad 2 Anterior to the patellar tendon, between the tendon and the skin 3 Anterior to the patella, between the skin and the patella 4 Posterior to the patellar tendon, between the tendon and infrapatellar fat pad 5 Between the superficial infrapatellar bursa and the infrapatellar fat pad

The prepatellar bursa is located anterior to the patella, between the skin and the patella. The superficial infrapatellar bursa is located anterior to the patellar tendon, between the tendon and the skin. The infrapatellar fat pad is located posterior to the patellar tendon.

A forearm fracture just distal to the elbow damages the radial nerve. What will the patient experience? Answer Choices 1 Wrist drop from the loss of wrist extension 2 Weakness of the pronator muscles 3 Loss of sensations on the ventral surface of the forearm 4 Loss of thumb opposition 5 No ability to abduct the fingers

The radial nerve innervates the wrist extensors, so damage to the nerve will not allow wrist extension, which leads to wrist drop. Pronator and opposition muscles, as well as sensory aspects of the ventral forearm, are all mediated by the median nerve. Finger abduction is controlled by the ulnar nerve.

A 32-year-old woman presents with a 3-month history of right hand dominant pain and paresthesia of the thumb, index, and middle fingers. She works in a retail sales office and spends much of the day typing. What associated physical finding may indicate an advanced case of this disorder? Answer Choices 1 Thenar atrophy 2 Hypothenar atrophy 3 Weakness on extension of the DIP joints 4 Decreased gap to 2 mm on 2-point discrimination test 5 Weak radial artery pulse

Thenar atrophy When compression of the median nerve is severe and long-standing, atrophy of the thenar muscles, which are supplied by the motor branch of the median nerve, is possible and indicates possible denervation of the muscles. Hypothenar atrophy would indicate ulnar nerve long-standing compression, usually because of a space-occupying lesion, such as ganglion, lipoma, ulnar artery aneurysm, or muscle anomaly. Weakness on extension of the DIP joints would indicate injury or entrapment of the radial nerve. Widening of the gap, NOT decrease of the gap, in 2-point discrimination testing would be expected. The radial artery is not within the carpal tunnel and would unlikely be decreased unless there was injury to the wrist.

A 58-year-old menopausal woman presents for a wellness exam. She has no current complaints and denies vasomotor symptoms, but reports she has recently been caring for her mother after the mother suffered a hip fracture. The patient's past medical history is remarkable for stable rheumatoid arthritis since age 28 years, with weekly etanercept. Her rheumatologist has periodically prescribed a few weeks of prednisone for multiple flares in the past. She takes no other medications, has no allergies, and has never had surgery. Her family history is significant for osteoporosis in an older sister. She has 4 grown children, lives with her husband, and is a homemaker. She denies use of alcohol and drugs, but admits to smoking 1 pack-per-day of cigarettes. On physical exam, her vitals and exam are all normal. In addition to smoking cessation, some preventive screenings are recommended and results are shown in the table. Dual-energy x-ray absorptiometry (DXA) T-score of -3.22 Question What intervention is most appropriate, based upon this patient's history and DXA score? Answer Choices 1 Increase use of prednisone to daily and stop entanercept 2 Initiate systemic combination hormone therapy 3 Repeat DXA in 6 months 4 Send for radionuclide bone scan 5 Start bisphosphonate therapy

This patient has several risk factors for osteoporosis (family history, smoker, rheumatoid arthritis, and prolonged use of glucocorticoids throughout life) and should be screened. It is important for primary care providers who perform "wellness" exams to identify risk factors and make recommendations for health, even when the patient is asymptomatic. Most patients with osteoporosis do not present with any symptoms. A T-score equal to or less than -2.5 defines osteoporosis. Some conditions, such as a parathyroid disorder, can contribute to secondary osteoporosis. This patient's labs have ruled that out. Treatment should be offered and first-line for this patient is to start bisphosphonate therapy. Additionally, this patient (and all patients with osteoporosis) should have adequate dietary or supplemented calcium and vitamin D, avoid smoking, be counseled on fall prevention, and achieve regular weight-bearing activity. Many chronic conditions, such as rheumatoid arthritis (RA), are treated periodically with steroids to reduce inflammation and gain control. However, a cumulative use of >/= 3 months of glucocorticoids, such as prednisone, is a risk factor for osteoporosis. This patient's RA is stable on entanercept, a tumor necrosis factor (TNF) inhibitor, which is not linked with osteoporosis. It would be very inappropriate to increase the use of prednisone to daily and stop entanercept for this patient, given her DXA result. This patient is menopausal, which is another risk factor for osteoporosis. Without natural estrogen production, bone mineral density decreases. However, it would be contraindicated to initiate systemic combination hormone therapy in this woman, as she is a smoker. Systemic hormone therapy is a reasonable treatment for osteoporosis if an individual does not have contraindications and cannot tolerate first-line treatments, such as bisphosphonates. Alternately, in a nonsmoker who is experiencing severe vasomotor symptoms and is without other estrogen contraindications, combination hormone therapy could be used primarily for the menopausal symptoms and secondarily for its role in treating osteoporosis. The history and test results are adequate to diagnose osteoporosis and begin treatment. Waiting to simply repeat the DXA in 6 months is not appropriate. Additionally, there is a small margin of error in all tests. In order to detect true changes in bone mineral density and overcome inherent differences in tests, the interval between DXA scans should be at least 1-2 years. As mentioned above, no further testing is needed to confirm the diagnosis and start treatment. It is not necessary to send this patient for a radionuclide bone scan. A bone scan may be done when a fracture is strongly suspected and not shown on other imaging, or when looking for malignancy. Bone scans are typically not necessary in the evaluation of osteoporosis.

A 24-year-old man presents with radial-sided wrist pain following a fall on his outstretched hand 3 days ago. He did not seek immediate medical attention due to a lack of swelling. Despite self-treatment with ice and with analgesics, his wrist still hurts, especially with ulnar deviation and while trying to open a jar or grip the steering wheel of his car. On examination, his tenderness seems to be in the anatomic snuffbox of the wrist. Radiographs are normal in all views. What is the proper treatment for this man at this time? Answer Choices 1 Elastic bandage for support, continue ice and analgesics, follow up PRN 2 Wrist volar splint, continue ice and analgesics, follow up in a month 3 Thumb spica splint and refer to an orthopedic surgeon 4 Immediate referral to an orthopedic surgeon 5 Routine referral to an orthopedic surgeon

Thumb spica splint and refer to an orthopedic surgeon If a patient with a suspected scaphoid fracture has radiographs that are read as 'normal', but clinically there is a fracture, place the patient in a long arm thumb spica splint and refer the patient to an orthopedic surgeon. Elastic bandage and volar splints are not adequate immobilization for a possible scaphoid fracture. Immediate referral is not warranted because you have not identified a positive diagnosis, and routine referral is not warranted for the same reason. However, if after a positive diagnosis there is also evidence of non-union or worsening fracture gap, a referral to an orthopedic surgeon is definitely warranted.

The most commonly fractured long bone in both adults and children is which of the following? A Femur B Fibula C Humerus D Radius E Tibia

Tibia The tibia is the most commonly fractured long bone in the body for both adults and children. The fractures are often the result of sporting activities in the young and may occur from a simple fall in the elderly - especially those with osteoporosis. Motor vehicle accidents are another common cause of tibial fractures. Open or complex tibial fractures are sometimes associated with compartment syndromes, infection and neurovascular compromise. The femur is the strongest of the long bones and generally only sustains fractures when exposed to extreme stress, such as that experienced in a motor vehicle collision or industrial accident. Fibular fractures commonly occur with a direct below to the lateral lower leg or with extreme ankle rotational forces or excessive inversion. A high percentage of ankle fractures involve the fibula, especially in older women. Humerus fractures are relatively rare in adults, but are the second most common fractures to occur at birth - behind only the clavicle in frequency. The radius is the most commonly fractured bone in the upper extremity, but still less common in frequency than the tibia. Falls on an outstretched arm are a common mechanism for the injury.

A 72-year-old female presents with a several month history of symmetric pain to the hands, wrists, and feet. She also is noticing a change to the appearance of the joints of the hands and feet, with a more "nobby" appearance than before. When the patient wakes up in the morning, the joints are particularly stiff and red at times. Out of all the areas that are painful, it is the hands that are the most painful. Based on this history, what physical exam findings would you expect to find on the hands that are classic findings of this disease? A Radial deviation and subluxation of the DIP joints B Radial deviation and subluxation of the MCP joints C Radial deviation and subluxation at the PIP joints D Ulnar deviation and subluxation at the MCP joints E Ulnar deviation and subluxation at the PIP joints

Ulnar deviation and subluxation at the MCP joints

A 34-year-old woman presents to your clinic with a 3-month history of a mildly tender mass on the dorsocentral aspect of her dominant right wrist. She says the "lump comes and goes," but this time, it has stayed and become tender. She denies any known trauma of the wrist or hand. What examination technique would help to show that this mass is a ganglion cyst? Answer Choices 1 Checking the active range of motion of the wrist 2 Palpating the mass for size & shape 3 Obtaining radiographs of the wrist 4 Transilluminating the mass 5 Palpating the mass for tenderness

Transilluminating the mass A prominent ganglion cyst of the dorsal wrist will transilluminate while a solid tumor will not. The active range of motion is usually not affected with a ganglion cyst except for tenderness that may limit the motion. Although ganglions are usually smooth on the surface, tumors can be too. Radiographs are taken only to rule out any bony pathology, but they won't reveal the ganglion. MRI is used for locating the mass and its origin. Most ganglions and tumors are tender to firm palpation or pressure.

Active elbow extension is primarily controlled by which muscle(s)? A Anconeus B Biceps C Brachialis D Brachioradialis E Triceps

Triceps E The triceps are the primary muscles that produce active elbow extension. The anconeus muscle is known as a secondary elbow extensor. The biceps and brachialis muscles are primary muscles of elbow flexion and the brachioradialis is a secondary elbow flexor.

A fracture involving the medial epicondyle will most likely cause damage to which nerve? A Axillary B Median C Peroneal D Radial E Ulnar

Ulnar Because the ulnar nerve passes through the cubital tunnel, which is a groove on the posterior aspect of the medial epicondyle, any fractures involving the medial epicondyle can also cause damage to the ulnar nerve. The median nerve is most susceptible to injury at the carpal tunnel. Branches of the radial nerve can become entrapped on the lateral side of the elbow and the associated symptoms are often confused with lateral epicondylitis. Radial nerve injuries are more commonly associated with humeral shaft fractures. The axillary nerve is significantly proximal to the medial epicondyle and the peroneal nerve is in the leg.

A 20-year-old man presents with right thumb pain after falling during a skiing trip 2 days ago. During the examination, you note gross laxity in the first metacarpophalangeal joint when moving the right thumb into abduction. A radiograph reveals no fracture. Question What structure is injured? Answer Choices 1 Radial collateral ligament of the thumb 2 Ulnar collateral ligament of the thumb 3 Flexor tendons of the thumb 4 Extensor tendons of the thumb 5 Median nerve damage

Ulnar collateral ligament of the thumb Explanation The clinical picture is of a gamekeeper's thumb. With this injury, the ulnar collateral ligament is ruptured, causing laxity when moving the thumb into abduction. There would be no laxity when moving the thumb into abduction with radial collateral ligament damage. The flexor and extensor tendons are not damaged in a gamekeeper's thumb. Median nerve damage (Carpal tunnel syndrome) would present with aching, numbness, or tingling in the thumb, some of the fingers, and sometimes part of the hand.

A 20-year-old man presents after taking a fall during a skiing trip 2 days ago. The chief complaint is pain to the right thumb. During the examination, you note gross laxity in the first metacarpophalangeal joint when moving the right thumb into abduction. An X-ray reveals no fracture. Question What structure is injured? Answer Choices 1 Radial collateral ligament of the thumb 2 Ulnar collateral ligament of the thumb 3 Flexor tendons of the thumb 4 Extensor tendons of the thumb 5 Median nerve damage

Ulnar collateral ligament of the thumb The clinical picture is of a gamekeeper's thumb. With this injury, the ulnar collateral ligament is ruptured, which causes laxity when moving the thumb into abduction. In cases of radial collateral ligament damage, there would be no laxity when moving the thumb into abduction. The flexor and extensor tendons are not damaged in a gamekeeper's thumb. Median nerve damage (Carpal tunnel syndrome) would present with aching, numbness, or tingling in the thumb, some of the fingers, and sometimes part of the hand.

A 24-year-old man presents due to increasing pain in his right groin and buttock and difficulty walking. The pain has been present and worsening for about 1 month. He further reports that he had quite significantly injured himself also near his right hip in a fall while snowboarding about 3 months ago. He did not seek care immediately and had difficulty ambulating for a week, then saw a chiropractor to "put his lower back back in." The pain and gait problems eventually improved. Now he is concerned because the pain has returned with no new history of trauma. He is otherwise healthy and takes no regular medicines. He denies any chronic health conditions and has an unremarkable family history for musculoskeletal and rheumatologic conditions. On physical exam, the patient was noted to walk into the exam room with a slight limp. Examination of the hip, buttock, and groin region reveals no edema, erythema, or ecchymosis. The exam does not produce one particular point of tenderness with palpation, but pain is elicited with right hip motions, particularly internal rotation. Diagnostic testing revealed a "crescent sign" and confirmed the suspected condition. The patient will be scheduled for definitive treatment within a week. Question What approach is best to prevent further complications and maintain his current health, until definitive treatment occurs? Answer Choices 1 Begin aggressive physical therapy 2 Initiate antibiotics 3 Initiate bisphosphonate therapy 4 Initiate high-dose corticosteroids 5 Utilize crutches for any ambulation

Utilize crutches This patient's condition is suspicious for avascular necrosis (or the more preferred term, osteonecrosis) of the hip. Avascular necrosis (AVN) is a relatively common complication, following a traumatic hip dislocation, when the vascular supply to the femoral head is damaged and bone death occurs. There are many theories about the exact mechanism, and other causes, such as vascular, congenital, and autoimmune diseases, have been implicated with osteonecrosis. If the "crescent sign" is seen on plain radiography (x-ray), AVN is confirmed. Definitive treatment is surgical, with a variety of approaches, from core decompression to bone grafting. Until surgery occurs, symptomatic patients should utilize crutches for any ambulation to prevent collapse of the femoral head. The patient should not begin aggressive physical therapy prior to surgical repair. As noted above, weight-bearing on the joint may predispose the patient to bone collapse. There is also no reason to initiate antibiotics prior to surgical treatment. AVN is not caused by infection, in contrast to osteomyelitis. Likewise, the provider should not initiate bisphosphonate therapy, even though the long-term goal is for the patient to regrow new bone. In fact, bisphosphonate therapy has been linked with causing osteonecrosis. There are some studies proposing benefit with using bisphosphonates, but not enough definitive data yet to recommend their use. The provider should not initiate high-dose corticosteroids, either. As with bisphosphonates, corticosteroids have been linked with causation of osteonecrosis. The primary mechanism of AVN does not appear to be inflammatory, so the steroids would not benefit the patient's healing.

A 17-year-old male college freshman presents with chronic fatigue, back pain, and stiffness. The pain has been present for the past several months, but it appears to be worsening. The back symptoms are worst at night and first thing in the morning, and they improve somewhat during the day. However, because he is always tired, he is having difficulty staying productive at school. Question MRI of the lumbosacral spine reveals bilateral inflammatory changes of the sacroiliac joints and fusion of the L3 - L5 vertebrae. What extra-articular condition is associated with this disease? Answer Choices 1 Diabetes mellitus 2 Peptic ulcer disease 3 Psoriatic skin lesions 4 Scleroderma 5 Uveitis

Uveitits The patient is presenting with signs and symptoms indicative of ankylosing spondylitis (AS). Characteristic symptoms include back pain that is worse in the morning and improves with exercise. Back stiffness is also reported, as is extreme fatigue. AS is more common in male patients. As with other inflammatory arthritides, patients usually present at a younger age (18 - 35 years) relative to patients with mechanical causes of back pain. Unlike mechanical back pain, the pain associated with inflammatory spondylitis usually improves with movement. AS is a clinical diagnosis, but many patients will have elevated c-reactive protein and erythrocyte sedimentation rates. Diagnostic imaging classically reveals inflammatory and sclerotic changes affecting the sacroiliac joints and spine. Vertebral fusion is also seen as a result of progressive joint erosion. Extra-articular conditions associated with AS include iritis and uveitis (inflammations of the iris and uvea, respectively) as well as inflammatory bowel disease, enthesitis (inflammation of tendinous and ligamentous tissue connected to bone), and dactylitis (sausage-shaped inflammatory changes affecting the fingers and toes). Heart disease is also more common in patients with inflammatory arthritides. Diabetes mellitus and peptic ulcer disease are relatively more common in patients with rheumatoid arthritis. Psoriatic skin lesions are seen in patients with psoriatic arthritis. Scleroderma is an autoimmune disease that does not coexist with increased frequency in any of the inflammatory arthritides.

The patient is a 16-year-old boy who is currently an inpatient 7 days after a motor vehicle accident that resulted in multiple lower extremity fractures. The fractures were surgically corrected recently. He is currently immobilized below the waist, but casts will not be placed until the majority of his edema has resolved. He is stable, the edema is resolving, and he is recovering well, with the exception of new onset of increased edema and erythema over his left thigh. X-rays show evidence of acute osteomyelitis. The patient does not have any allergies, and a culture from the infected bone has revealed methicillin-resistant Staphylococcus aureus. Question What treatment regimen is the best choice to treat his infection following surgical debridement? Answer Choices 1 Nafcillin (Unipen) 2g IV q 4-6 hours for 6 weeks 2 Vancomycin (Vancocin)1 g IV q 12 hours for 6 weeks 3 Oxacillin (Bactocill) 2 g IV q 4-6 hours for 2 weeks 4 Ciprofloxacin (Cipro) 400mg IV q 12 hours for 2 weeks 5 Linezolid (Zyvox) 600mg IV q 12 hours for 2 weeks

Vancomycin The correct answer is Vancomycin (Vancocin) 1g IV q 12 hours for 6 weeks for several reasons. Methicillin-resistant Staphylococcus aureus (MRSA) has to be treated with an antibiotic that MRSA is sensitive to, such as Vancomycin. Vancomycin is a glycopeptide antibiotic typically used to treat Gram-positive bacteria. The other reason that this answer is correct is due to the length of time noted in the treatment. Patients with acute osteomyelitis need at least 4-6 weeks of antibiotic therapy. Studies have also been done on animals that show optimal treatment length to be 4-6 weeks in duration. Nafcillin (Unipen) 2g IV q 4-6 hours for 6 weeks is not the correct answer. Nafcillin is a narrow spectrum beta-lactamase penicillin that is used to treat Gram-positive bacteria, including staphylococci, that are resistant to other penicillins. If the patient's culture had shown methicillin-sensitive Staphylococcus aureus (MSSA), then Nafcillin would have been an appropriate antibiotic choice. However, Nafcillin is not effective against MRSA. The length of time of 6 weeks was an appropriate length of time if, again, the culture had shown MSSA. Oxacillin (Bactocill) 2 g IV q 4-6 hours for 2 weeks is not the correct answer. Oxacillin is also a narrow spectrum beta-lactamase penicillin that is used to treat Gram-positive bacteria, including staphylococci, that are resistant to other penicillins. If the culture had shown MSSA, then Oxacillin would have been an appropriate antibiotic choice. In addition, the answer is not correct because the length of treatment of 2 weeks would not be long enough. Ciprofloxacin (Cipro) 400mg IV q 12 hours for 4 weeks is not the correct answer. Ciprofloxacin is a fluoroquinolone antibiotic used to treat a wide variety of infections. However, it is typically used to treat Gram-negative organisms rather than the Gram-positive organism in this question. It is possible for osteomyelitis to be caused by Gram-negative bacilli, and this would be a good treatment option in that case. Linezolid (Zyvox) 600mg IV q 12 hours for 2 weeks is not the correct answer. Linezolid is in the oxazolidinone class of medications and is typically used to treat infections that have resistance to other antibiotics, such as vancomycin-resistant enterococci (VRE) or MRSA. Therefore, Linezolid would be a good antibiotic choice in this case. However, the length of treatment of 2 weeks would not be long enough to effectively treat the infection.

The patient is a 16-year-old boy who is currently an inpatient 7 days after a motor vehicle accident that resulted in multiple lower extremity fractures. The fractures were surgically corrected recently. He is currently immobilized below the waist, but casts will not be placed until the majority of his edema has resolved. He is stable, the edema is resolving, and he is recovering well, with the exception of new onset of increased edema and erythema over his left thigh. X-rays show evidence of acute osteomyelitis. The patient does not have any allergies, and a culture from the infected bone has revealed methicillin-resistant Staphylococcus aureus. Question What treatment regimen is the best choice to treat his infection following surgical debridement? Answer Choices 1 Nafcillin (Unipen) 2g IV q 4-6 hours for 6 weeks 2 Vancomycin (Vancocin)1 g IV q 12 hours for 6 weeks 3 Oxacillin (Bactocill) 2 g IV q 4-6 hours for 2 weeks 4 Ciprofloxacin (Cipro) 400mg IV q 12 hours for 2 weeks 5 Linezolid (Zyvox) 600mg IV q 12 hours for 2 weeks

Vancomycin (Vancocin)1 g IV q 12 hours for 6 weeks The correct answer is Vancomycin (Vancocin) 1g IV q 12 hours for 6 weeks for several reasons. Methicillin-resistant Staphylococcus aureus (MRSA) has to be treated with an antibiotic that MRSA is sensitive to, such as Vancomycin. Vancomycin is a glycopeptide antibiotic typically used to treat Gram-positive bacteria. The other reason that this answer is correct is due to the length of time noted in the treatment. Patients with acute osteomyelitis need at least 4-6 weeks of antibiotic therapy. Studies have also been done on animals that show optimal treatment length to be 4-6 weeks in duration. Nafcillin (Unipen) 2g IV q 4-6 hours for 6 weeks is not the correct answer. Nafcillin is a narrow spectrum beta-lactamase penicillin that is used to treat Gram-positive bacteria, including staphylococci, that are resistant to other penicillins. If the patient's culture had shown methicillin-sensitive Staphylococcus aureus (MSSA), then Nafcillin would have been an appropriate antibiotic choice. However, Nafcillin is not effective against MRSA. The length of time of 6 weeks was an appropriate length of time if, again, the culture had shown MSSA. Oxacillin (Bactocill) 2 g IV q 4-6 hours for 2 weeks is not the correct answer. Oxacillin is also a narrow spectrum beta-lactamase penicillin that is used to treat Gram-positive bacteria, including staphylococci, that are resistant to other penicillins. If the culture had shown MSSA, then Oxacillin would have been an appropriate antibiotic choice. In addition, the answer is not correct because the length of treatment of 2 weeks would not be long enough. Ciprofloxacin (Cipro) 400mg IV q 12 hours for 4 weeks is not the correct answer. Ciprofloxacin is a fluoroquinolone antibiotic used to treat a wide variety of infections. However, it is typically used to treat Gram-negative organisms rather than the Gram-positive organism in this question. It is possible for osteomyelitis to be caused by Gram-negative bacilli, and this would be a good treatment option in that case. Linezolid (Zyvox) 600mg IV q 12 hours for 2 weeks is not the correct answer. Linezolid is in the oxazolidinone class of medications and is typically used to treat infections that have resistance to other antibiotics, such as vancomycin-resistant enterococci (VRE) or MRSA. Therefore, Linezolid would be a good antibiotic choice in this case. However, the length of treatment of 2 weeks would not be long enough to effectively treat the infection.

A 64-year-old female who has a history of injectable drug use presents with blood work that reveals leukocytosis with a left shift, and there is suspicion of osteomyelitis based on the patient's prior history. Based on this history, what bone would be most affected by hematogenous osteomyelitis in adults? A Feet B Long bones C Pelvis D Sternoclavicular bones E Vertebrae

Vertebrae Hematogenous osteomyelitis accounts for about 20% of all cases of osteomyelitis in adults. It is more common in males and the prevalence is higher amongst those who are IV drug abusers, patients being treated with dialysis or who have sickle cell disease. Other conditions which may lead to sepsis (i.e. patients with central lines, urinary infections, and urethral catheterization) increase the risk of hematogenous osteomyelitis. Unlike children, the long bones are rarely affected in adults with the vertebrae being the most likely location for the bone infection to occur. Lumbar vertebrae are most often affected, followed by thoracic and cervical vertebrae. Osteomyelitis of the sternoclavicular bones and pelvic bones are not uncommon sites, but these tend to be most frequent amongst IV drug abusers. The feet do not tend to be significantly affected by hematogenous osteomyelitis as frequently as osteomyelitis caused by infected foot ulcers as are often seen in those with diabetes or peripheral vascular disease

A 64-year-old female who has a history of injectable drug use presents with blood work that reveals leukocytosis with a left shift, and there is suspicion of osteomyelitis based on the patient's prior history. Based on this history, what bone would be most affected by hematogenous osteomyelitis in adults? A Feet B Long bones C Pelvis D Sternoclavicular bones E Vertebrae

Vertebrae Hematogenous osteomyelitis accounts for about 20% of all cases of osteomyelitis in adults. It is more common in males and the prevalence is higher amongst those who are IV drug abusers, patients being treated with dialysis or who have sickle cell disease. Other conditions which may lead to sepsis (i.e. patients with central lines, urinary infections, and urethral catheterization) increase the risk of hematogenous osteomyelitis. Unlike children, the long bones are rarely affected in adults with the vertebrae being the most likely location for the bone infection to occur. Lumbar vertebrae are most often affected, followed by thoracic and cervical vertebrae. Osteomyelitis of the sternoclavicular bones and pelvic bones are not uncommon sites, but these tend to be most frequent amongst IV drug abusers. The feet do not tend to be significantly affected by hematogenous osteomyelitis as frequently as osteomyelitis caused by infected foot ulcers as are often seen in those with diabetes or peripheral vascular disease.

Which of the following would be the best type of exercise to do to strengthen bones and prevent osteoporosis? A Ballistic stretching B Recumbent bike C Static stretching D Swimming E Weight training

Weight training Weight bearing and resistance exercises are the most effective at maintaining bone density. Bones respond favorably to regular impact loading exercises such as walking and strength training with either machines or free weights. Adding balance training exercises like those done in tai chi, yoga, or Pilates would be beneficial as well as part of a fracture prevention plan. Ballistic stretching is a type of flexibility exercise that uses bouncing movements to try and increase range of motion. Static stretching is a more gentle approach to stretching where the person stretches as far as they comfortably can and hold that position for several seconds. There are various schools of thought on the benefits and safety of each stretching method. While both likely have a role in maintaining flexibility and function, neither is likely to play a significant role in the prevention of osteoporosis. Exercising on a recumbent bike likely offers some strength and cardiovascular benefits, especially if utilized as part of a strenuous workout, but as a non weight bearing exercise (seeing as the person using the bike is sitting and leaning back with their weight fully supported), this is not the ideal exercise for osteoporosis prevention. Swimming is a wonderful exercise choice since it affords a challenging cardiovascular workout, uses all of the muscles of the body, improves range of motion and is well tolerated by people with various joint ailments, but because it is not a weight bearing exercise, it should be combined with weight training for optimal bone health.

A 27-year-old Caucasian man returns to the emergency department with unbearable left lower leg pain; he does so approximately 6 hours after initial discharge. While playing lacrosse, the patient sustained a closed, mid-shaft tibial fracture. After casting and an anti-inflammatory, his pain was noted to be mild (2 out of 10 on 1 - 10 scale) at time of discharge. He reports his pain is increasing dramatically (it is now rated at 9 out of 10) and is unresponsive to his prescribed narcotic, acetaminophen, icing, and elevating his leg. He also describes a feeling of tingling and numbness throughout the lower left extremity. Question What is the most likely cause of this patient's severe pain? Answer Choices 1 Claudication 2 Compartment syndrome 3 Malingering or drug-seeking behavior 4 Sepsis 5 Unidentified fibula fracture

compartment syndrome The most likely diagnosis is compartment syndrome, which is an emergency. Without prompt identification and treatment, blood flow is reduced and can result in tissue necrosis and permanent nerve and muscle damage. Compartment syndrome can occur after a trauma, such as a fracture or burn. As increasing fluid pressures build in the area of injury, the restrictive fascia prevents outflow. In this patient's case, his cast may have been applied too tightly and should be removed immediately. If his symptoms do not resolve, he may need a surgical fasciotomy. Key features of compartment syndrome include a history of trauma (especially to an extremity), edema, and pain out of proportion to history/findings. Pulselessness is a late finding. Claudication is a condition in which the patient experiences lower extremity pain secondary to poor arterial blood supply. It is associated with peripheral arterial disease. Claudication should classically worsen with movement (such as prolonged walking) and improve with rest. This patient's pain did not improve with rest and his history is inconsistent with peripheral arterial disease. Malingering or drug-seeking behaviors should be evaluated in any patient complaining of pain out of proportion to history, as the patient may be attempting to deceive the provider. This patient's history was not concerning for prior history of psychiatric illness or substance abuse. The objective physical exam findings on this patient (reduced capillary refill and sensation, edema and shiny, taut skin) help substantiate a diagnosis of compartment syndrome. Sepsis is concerning condition that could cause edema and pain in an extremity. However, the patient had a history of a closed fracture (much less likely to develop sepsis than with an open fracture) and is afebrile. This patient could be at a future risk of sepsis, but in this scenario, he does not present with systemic symptoms of profound infection. Depending upon the mechanism of injury, this patient may have fractured both his tibia and fibular. Sometimes, a 2nd fracture can be missed in initial evaluation. However, a fibular fracture would not explain this patient's severe pain. In contrast, a fibular fracture would be expected to respond to immobility (with the cast), ice, analgesics, and elevation.

A 35-year-old man presents with a 2-week history of constant burning and throbbing pain in his left hand. The pain also seems to affect his distant forearm. Contact with normal clothing and bed sheets worsen his hand pain. He also reports that his hand is swollen. He denies trauma to his hand, but does report an uncomplicated left wrist fracture 3 months ago, which resulted from a sports accident. By the time of cast removal at 7 weeks post-injury, patient denied any pain or edema in the affected limb. He was instructed to return to normal activities, as tolerated. He was initially treated with ibuprofen and opioid pain pills. He discontinued all pain medicines within 2 weeks of the fracture. He now describes his hand pain as an 8 out of 10, on a 1 - 10 pain scale. On physical exam, the left hand and forearm are noted to have some localized edema, warmer temperature, and increased hair growth, compared to his right hand and forearm. No rash or skin lesions are noted. With even light palpation of the affected region, the patient cries out in pain. Range of motion is decreased, and reflexes are increased the left upper extremity (in comparison to the right). The rest of his exam is normal. X-ray of the left wrist and hand are normal, with good fracture resolution. Question What is the most likely diagnosis? Answer Choices 1 Complex regional pain syndrome 2 Factitious disorder 3 Herpes zoster (shingles) 4 Nerve root impingement at cervical spine 5 Thoracic outlet syndrome

complex regional pain syndrome This patient most likely has complex regional pain syndrome (CRPS). CRPS is indeed a complex syndrome often occurring in a limb; patients exhibit vasomotor and neurologic symptoms out of proportion with objective findings. It often occurs following some type of event, such as a surgery, fracture, stroke, or myocardial infarction. Most of the time, there is no nerve lesion. CRPS is classified into 3 stages, and symptoms can vary depending on the stage. It is characterized by persistent burning and/or throbbing pain in an extremity. Swelling, redness, and increased hair/nail growth occur in stage I; it may progress to cool, pale skin, and even osteoporosis in the latter 2 stages. Allodynia, the phenomenon in which a normally painless stimulus produces significant pain response, is a common finding. No single exam finding or test is diagnostic of CRPS, which is a clinical diagnosis. Factitious disorder is a psychiatric illness in which the patient feigns symptoms in order to be in the "sick role". The careful practitioner will need to be aware that CRPS exists. Otherwise, it may be tempting to assume the patient is exaggerating his symptoms, as his/her pain seems out of proportion to objective findings. Herpes zoster (shingles) can cause inflammation in a single, unilateral nerve root, leading to burning pain sensation in a limb (or other body part). The skin lesions may not appear immediately. This patient's affected region is not consistent with 1 dermatome, and herpes zoster is quite uncommon in this age group. Nerve root impingement at the cervical spine (cervical radiculopathy) may occur following neck injury or degenerative changes. It can be associated with unilateral limb pain and paresthesias, but it typically affects the limb more proximally. The distal edema and allodynia are not classically associated with cervical radiculopathy. Thoracic outlet syndrome is a condition in which the blood vessels and/or nerves become compressed as they exit the chest wall and travel down the arms. It may be unilateral or bilateral, and it can cause pain and paresthesias in the neck, arms and hands. Thoracic outlet syndrome would be more likely if the patient described repetitive upper body movements. With more localized, unilateral symptoms following a fracture, CRPS seems much more likely.

A 49-year-old, right-hand dominant woman presents with a 2-week history of progressive pain in her right thumb and wrist area. She says that her thumb seems to "stick" in place upon movement. She states that she has never experienced this before. You ask her if she has been using her hands more often than normal, and she tells you she recently began to crochet a sweater for a family member. On physical examination, her pain is markedly exacerbated when she places her thumb into the palm of her hand, and you passively move her hand in an ulnar direction. Question What is the most likely diagnosis? Answer Choices 1 Wrist arthritis 2 Scaphoid fracture 3 Dorsal wrist ganglion 4 Thumb arthritis 5 de Quervain's tenosynovitis

de Quervain's tenosynovitis The clinical picture is suggestive of de Quervain's tenosynovitis. It occurs commonly in middle-aged women, precipitated by repetitive use of the thumb. The inflammation thickens the tendon sheath abductor pollicis longus and extensor brevis on the thumb side of the wrist. This can cause pain, swelling, and may cause the thumb to lock in place upon movement. The Finkelstein test (full flexion of thumb into palm followed by ulnar deviation of the wrist) will produce pain and is diagnostic of de Quervain's tenosynovitis. Wrist arthritis causes pain on movement with evidence on radiographs, which is not seen in this patient. A scaphoid fracture would have tenderness in the anatomical snuffbox, which is not described in this patient. Dorsal wrist ganglion presents with a palpable mass. Thumb arthritis presents with swelling over the carpometacarpal joint and pain on joint compression.

A 67-year-old presents with a 2-year history of numbness in the lower extremities. The most relevant complaint is gait impairment and instability due to loss of deep tendon sensation. Neurologic examination shows a disturbed sense of position with preserved tactile and temperature sense, normal muscular force, and normal reflexes. Electrophysiologic evaluation excludes a sensitive polyneuropathy Question What is the most likely cause of the patient's symptoms? Answer Choices 1 Lipoma 2 Astrocytoma 3 Hemangioblastoma 4 Oligodendroglioma 5 Ependymoma

ependymoma This patient has an ependymoma. Ependymomas make up approximately 60% of spinal cord tumors. They are relatively slow-growing and may develop anywhere along the entire length of the spinal cord. However, a large percentage of these tumors are found at the lower sections of the cord. They develop from ependymal cells and can be seen in all age groups; however, the most common age group is between 20 and 40 years. Men and women are affected equally. Tumors may metastasize here from other parts of the body or they may be made of the same types of cells. Spinal tumors may be intradural, extramedullary, intramedullary, or extradural. Astrocytomas are the 2nd most common type of spinal cord tumor. They are found more often in children than in adults. Other rare types include developmental tumors, oligodendogliomas, lipomas, and hemangioblastomas. The usual presentation is pain, and classically it is aggravated in supine or flat position. As the tumor advances, abnormal sensation and eventually weakness follow, until finally the normal bladder and bowel function is lost. Surgery is the treatment of choice. Radiation therapy is sometimes given if the total surgical removal of the tumor is not possible or if the tumor appears to grow rapidly.

A 45-year-old man presents a 2-day history of left-sided chest pain. He describes it as an aching pain that is aggravated by movement, and he denies any history of trauma to the chest. On examination, there is tenderness on palpation over his left chest wall. Question What is the most likely cause of his chest pain? Answer Choices 1 Musculoskeletal pain 2 Herpes zoster 3 Spontaneous pneumothorax 4 Angina 5 Pericarditis

musculoskeletal pain Musculoskeletal chest pain, which can be unilateral or bilateral, is usually described as an aching pain. It is aggravated by movement. On examination, tenderness may be elicited on application of pressure over the affected chest wall. Herpes zoster pain is usually described as sharp or burning. On examination, there is usually a unilateral vesicular rash with a dermatomal distribution. In some cases, the pain may precede the lesions. Angina pain is usually described as tightness or pressure on the chest and lasts for less than 10 minutes. It is usually left sided or retrosternal and may radiate to the jaw, neck, and shoulder. It is precipitated by physical exertion or emotional stress. It is relieved by rest and sublingual nitroglycerin. Spontaneous pneumothorax pain is of sudden onset and patients present within a few hours of its onset. It is usually pleuritic in nature. On examination, the patients may be dyspneic with decreased breath sounds on the affected side. Pericarditis pain is usually described as a sharp retrosternal pain that is often worse when the patient is supine and is relieved by sitting upright. It is aggravated by deep inspiration and changes in position. It may be episodic and last for hours to days. On examination, a pericardial friction rub may be auscultated

A 54-year-old man presents with a 2-month history of left non-dominant shoulder pain. There is no history of trauma, but the pain began about a week after shoveling wet heavy snow from his 100-foot driveway. At first, the pain seemed to come and go from day to day, but has gotten progressively worse and more constant. Pain is worse with overhead use, and he cannot sleep on his left side; he wakes up if he rolls over onto his left shoulder. On exam, he is tender to palpation over the left humerus greater tuberosity. He has pain with Neer and Hawkins tests, but has a negative drop arm test. You prescribe physical therapy and non-steroidal anti-inflammatories, and you also advise taking a break from using the arm for heavy work.. Question At what point would you recommend a steroid injection? Answer Choices 1 Right away, as pain relief will make stretching more effective 2 Steroid injections should never be used for this problem 3 Only after failure of a home therapy program of at least 6 weeks 4 Only after 3 weeks of formal physical therapy treatment 5 Once a week for 6 weeks to reduce the inflammation

once after failure of a home therapy program of at least 6 weeks When a home therapy program performed 3 to 4 times a day for 6 weeks fails to improve the symptoms of impingement syndrome, a subacromial injection of steroid could be considered. Posterior capsule stretching exercises should be performed at home prior to any steroid injection. Some patients require the supervision of a physical therapist to assure adequate stretching, but they still require at least 6 weeks of therapy prior to an injection. Steroid injections should not be repeated if the first one does not improve the symptoms for at least 4 to 6 weeks. Repeated steroid injections can weaken the tendons of the rotator cuff and potentially cause a rupture of the tendon.

A 6-year-old boy has gradually increasing sharp pain to his left anterior hip that seems worse at night. He is moderately overweight, but he remains active and plays baseball. There is no history of trauma. There is no redness, swelling, or fever; there is no involvement of the other hip. Ibuprofen does seem to help. There is also no family history of any bone or joint disease. His weight is at the 95th percentile with height at the 50th percentile. On exam, there is mild tenderness to palpation over his left anterior hip, but the rest of the exam is normal. Laboratory studies show a normal complete blood count, hemoglobin, sedimentation rate, platelets, and C reactive protein. Plain X-rays show a small round lytic lesion surrounded by mild reactive bone formation. Based on these findings, what is the most likely diagnosis? Answer Choices 1 Legg-Calve-Perthes disease 2 Osteomyelitis 3 Primary lymphoma of bone 4 Osteoid osteoma 5 Ewings sarcoma

osteoid osteoma Osteoid osteoma is a common benign bone tumor consisting of an ovoid or spherical nidus of osteoid-rich tissue with interconnected bone trabeculae on highly vascularized connective tissue. It ranges from 0.5 to 2 cm in size. It does not progress or have malignant potential. In fact, some lesions may regress spontaneously. Multinucleated giant cells, osteoclasts, and axons are frequently observed. There may be bone sclerosis around the central nidus. Most are cortical or periosteal, but 20% arise within the marrow. Most patients are 5 to 20 years old. Most commonly, the proximal femur and tibia are involved, although any bone may be affected. Pain is usually the presenting symptom and is frequently relieved with non-steroidal anti-inflammatory drugs (NSAIDs). Tenderness upon palpation may be elicited on exam; patients may also have disuse atrophy, painful scoliosis, or limb-length discrepancy. X-rays are the initial examination of choice to reveal the central nidus as a small spherical or oval lytic lesion surrounded by soft-tissue edema and possible reactive bone formation. Computed Tomography (CT) is also used for precise localization and as guidance for biopsy and ablation. The primary goal of treatment is pain control with NSAIDs; however, if pain is not controlled or complications such as neuropathy, synovitis, growth disturbance, or scoliosis develop, the lesion should be removed. Several techniques are available for ablation, including percutaneous radiofrequency, ethanol, laser, or thermocoagulation therapy under CT guidance. Surgical options include en bloc excision, curettage, or CT guided removal. Although small lytic lesions can be caused by infections or other benign neoplasms of the bone and leukemia, the history, physical findings, small size of the lesion, and location in the diaphysis make osteoid osteoma the most likely diagnosis. Ewing sarcoma is a highly malignant primary bone tumor derived from bone marrow. Males are affected more frequently than females, and it most commonly presents in children and adolescents aged 4-15 years. It is especially infrequent in African Americans and Asians. Metaphyseal or diaphyseal areas of long and flat bones are both affected. The earliest symptom is intermittent pain that becomes intense and may radiate to the limbs, particularly with tumors in the vertebral or pelvic region. X-rays will show a long, permeative lytic lesion in the metadiaphysis and diaphysis of the bone with a prominent soft tissue mass extending from the bone, or it will show a lesion with poorly defined margins that is destroying the bone. Occult metastatic disease is usually present, requiring multi-drug chemotherapy as well as local disease control with surgery and/or radiation. Length of treatment varies depending on location and staging, but most patients undergo chemotherapy for 6-12 months and radiation for 5-8 weeks. Legg-Calve-Perthes disease, a childhood hip disorder, results from infarction with idiopathic avascular necrosis of the bony epiphysis of the femoral head. It occurs bilaterally in 10-20% of children, and they usually present at 4-8 years of age. Children present with limp, pain, and reduced hip range of motion. Possible causes include trauma, developmental, inflammatory, and coagulation abnormalities. Risk factors include male gender, low socioeconomic group, presence of inguinal hernia or genitourinary tract anomalies, and low birth weight. Early radiological signs most commonly show a small femoral epiphysis, sclerosis of the femoral head with sequestration and collapse with fragmentation, slight widening of the joint space due to thickening of the cartilage, failure of epiphyseal growth, and the presence of joint fluid or joint laxity. Magnetic Resonance Imaging is more sensitive in detecting early disease. Treatment aims at maintaining containment of the femoral head within the acetabulum with abduction splints, casts, or surgically with an osteotomy of the proximal femur. Osteomyelitis is an infective process that may involve all osseous components including the metaphyses of long bones, particularly the tibia or femur. Hematogenous spread most commonly occurs in infants and young children. Neonates have group B streptococci and Escherichia coli as the most commonly involved organisms, with 50% also having a septic joint. Beyond the neonatal period, the most common organisms involved are Staphylococcus aureus, group A streptococci, and Haemophilus influenzae type b. Infections in this age group may gain access from traumatic injuries or surgical procedures. Joint involvement is rare beyond the neonatal period. Untreated or inadequately treated infections can become chronic, with staphylococcus isolated most frequently. Clinical presentation of hematogenous osteomyelitis can be variable, ranging from acute pain with decreased movement of the affected limb and adjacent joint to possible swelling and redness. Systemic signs can include fever, malaise, and irritability. Following trauma onset may be insidious; despite localized pain, edema, and redness, the frequent absence of systemic signs can misleadingly suggest a superficial infection. Laboratory studies may show an elevated or normal leukocyte count where sedimentation rate and C-reactive protein are typically elevated. A blood culture is positive in 50% of cases. Diagnosis is based on the history and physical exam findings supported by a positive technetium-99 methylene diphosphonate bone scan. X-ray changes show swelling of tissue around the infection and periosteal reaction or new bone formation, but these may not be evident until 1 to 2 weeks after onset of symptoms. Eventually, bone destruction with lytic lesions will be visible. Successful treatment requires an appropriate antibiotic at sufficiently high dose and duration. For staphylococcal infections, the most common cause of osteomyelitis, a penicillinase-resistant semisynthetic penicillin is the drug of choice. Primary lymphoma of bone (PLB) is a rare malignant neoplastic disorder. Most cases result from non-Hodgkin lymphoma. PLB tumors produce osteoclast-stimulating factors that cause lytic bone destruction. It is rare in children younger than 10 years of age and fairly evenly distributed in the 2nd through 8th decades. The usual presenting symptom is prolonged pain with possible swelling. Diagnostic criteria include a primary focus in a single bone, histologic confirmation, and no evidence of distant soft tissue or lymph node involvement. The most common location is the thoracic spine, but diffuse spinal involvement is also quite frequent. Treatment involves radiation therapy and chemotherapy. Surgery is indicated only when a large mass causes acute cord compression.

A 47-year-old man presents with a history of worsening burning left heel pain for the last few weeks. He states that it is worse in the morning after getting out of bed and beginning to walk; it improves with exercise. He denies any foot numbness, tingling, or known injury. He states that he recently began a jogging regimen. On examination, Thompson test is negative. Question What is the most likely diagnosis? Answer Choices 1 Heel contusion 2 Calcaneal fracture 3 Tarsal tunnel syndrome 4 Achilles tendon rupture 5 Plantar fasciitis

plantar fasciitis The correct answer is plantar fasciitis, as it occurs as a result of excess stress on the Achilles tendon that attaches to the plantar fascia. Classic presentation is burning heel pain, which is worse when bearing weight after prolonged periods of rest (e.g., sleep). Heel contusion is a soft tissue injury resulting from a trauma. Heel contusions cause pain, but the pain is unlikely to be as severe as what is described in this case. Calcaneal fracture typically occurs after an injury, and the pain would likely be more consistent with weight bearing. Tarsal tunnel syndrome is due to entrapment of the posterior tibial nerve, and it presents with medial malleolus pain that may radiate. Pain is typically worse during sleep, walking, and dorsiflexion. Paresthesias are common. Achilles tendon rupture is often described as the feeling of being kicked or shot in the area of the Achilles tendon; subsequently, jogging and walking normally would not be possible, as the tendon is crucial for this movement.

An anxious-looking young mother brings a 6-week-old female infant to your office wrapped in many blankets. When your nurse unwraps her, she appears lethargic; she has dusky mottled skin and very weak pulses. As your staff quickly provides ventilation and compressions, you prepare to establish vascular access. What is the best site for immediate vascular access for this infant? Answer Choices 1 Peripheral scalp vein 2 Proximal anteromedial tibia 3 Saphenous vein 4 Distal radius 5 Sternum

proximal anteromedial tibia Insertion of an intraosseous needle directly into the anterior tibia is relatively easy and will assure rapid vascular access. This infant's clinical presentation of decreased activity, mottled, dusky skin, and weak pulses suggests severe volume depletion. Whatever the cause, and in this emergent situation that can be assessed later, rapid fluid resuscitation and possibly pressors are indicated. A peripheral scalp vein would be very difficult to cannulate in this volume-depleted infant. Blood would be shunted away from the periphery to more vital organs. Also, these small caliber veins would not allow a large bore needle for rapid fluid resuscitation. The saphenous vein, though larger than scalp veins, would also be difficult to cannulate due to shunting. The distal radius is a site chosen for intraosseous access in the adolescent. In an infant, it is not yet fully developed. The sternum is a site used for intraosseous access in the adult and is part of a system called First Access for Shock and Trauma (FAST) and works by a hand driven, push-pull mechanism.

A 10-year-old boy presents with a possible arm fracture. While playing baseball at school, he slid into another student and hit his upper arm on the other student's leg. An X-ray of the patient's right arm reveals that he has a fracture at the distal 1/3 of the humerus. You also note that the patient cannot extend his wrist. Question What structure has most likely been injured? Answer Choices 1 Brachial artery 2 Musculocutaneous nerve 3 Median nerve 4 Ulnar nerve 5 Radial nerve

radial nerve Explanation The clinical picture is suggestive of injury to the radial nerve. The characteristic sign of injury to the radial nerve is wrist-drop. Wrist-drop is the inability to extend the wrist and the digits of the metacarpophalangeal joints. Injury to the brachial artery would result in contraction of the digits and sometimes the wrist. Since there is collateral circulation, symptoms of ischemia to the distal nerves and muscles may take up to 6 hours to present. Injury to the musculocutaneous nerve would result in weakening of elbow flexion and supination of the forearm. Injury to the median nerve would result in loss of flexion of the PIP joints in digits 1 to 3. Injury to the ulnar nerve can result in sensory loss in the hand, and patients may have difficulty making a fist because they cannot flex their 4th and 5th digits at the DIP joint.

A 73-year-old man presents with inability to actively raise his left non-dominant arm to retrieve plates from the kitchen cabinet. This began a month ago after his shoulder pain improved. He had a history of pain in that shoulder for over 6 months that kept him from sleeping on the left side and the pain would wake him often. There was no specific injury he can recall although he felt a pop a month ago while taking out the trash. On inspection you notice the back of the shoulder appears sunken when compared to the other shoulder. Question What is the most likely diagnosis? Answer Choices 1 Rotator cuff tear 2 Impingement Syndrome 3 Adhesive capsulitis 4 Glenohumeral dislocation 5 Proximal Humerus Fracture

rotator cuff tear The key here is the inability to actively raise the arm. Typically elderly patients with rotator cuff tears report a prodrome of chronic shoulder pain for several months and maybe a episode where they feel a pop when performing some heavier than usual activity (taking the trash out). A sunken shoulder indicates atrophy due to the tear. Impingement syndrome would be painful, but active motion is usually normal. Adhesive capsulitis is limited active and passive range of motion limitation. Rotator cuff tear has no passive range of motion limitation. Glenohumeral dislocation is painful with any motion, but the disorder is not with the rotator cuff, but the joint congruity. A proximal humerus fracture would have a history of trauma.

A 56-year-old insulin dependent diabetic has been under your evaluation for his diabetes for several years. The patient has a 3-year history of diabetic neuropathy to the right foot, and may have suffered an injury to the foot without knowing due to loss of sensation. The patient now presents with a tender, reddened, and swollen right foot for the last 10 days that is also warm to the touch. You suspect that this patient may have an acute case of osteomyelitis. Based on this history, what bacterial organism is most commonly the cause of osteomyelitis? A Group A beta-hemolytic streptococci B Hemophilus influenzae C Mycoplasma D Pseudomonas aeruginosa E Staphylococcus aureus

staph aureus

A forearm fracture just distal to the elbow damages the radial nerve. What will the patient experience? Answer Choices 1 Wrist drop from the loss of wrist extension 2 Weakness of the pronator muscles 3 Loss of sensations on the ventral surface of the forearm 4 Loss of thumb opposition 5 No ability to abduct the fingers

wrist drop form the loss of wrist extension the radial nerve innervates the wrist extensors, so damage to the nerve will not allow wrist extension, which leads to wrist drop. Pronator and opposition muscles, as well as sensory aspects of the ventral forearm, are all mediated by the median nerve. Finger abduction is controlled by the ulnar nerve.


Ensembles d'études connexes

Why Profit Appears as a Credit on a Company's Balance Sheet

View Set

Chapter 57: Drugs Affecting Gastrointestinal Secretions

View Set

Government Types: Types of Government

View Set

BIOL 1030 Chapter 5 homework questions

View Set

Chapter 10: Fiscal Policy & Debt

View Set

Muscles of the Neck and Vertebral Column: Head Movements and Trunk Extension

View Set